You are on page 1of 118

Bu ürünün bütün hakları

ÇÖZÜM DERGİSİ YAYINCILIK SAN. TİC. LTD. ŞTİ.’ne aittir.

Tamamının ya da bir kısmının ürünü yayımlayan şirketin

önceden izni olmaksızın fotokopi ya da elektronik, mekanik

herhangi bir kayıt sistemiyle çoğaltılması, yayımlanması ve

depolanması yasaktır.

Çözüm Yayınları Grafik Birimi

Çözüm Yayınları Dizgi Birimi

2017 Ankara

Başak Matbaacılık
(0312) 397 16 17

ÖNSÖZ

Sayın Meslektaşlarım ve Sevgili Öğrenciler,


Üniversite eğitimi, kişinin çalışma hayatına ve yaşantısına yön veren önemli bir unsurdur. Bu eğitime katılmanın
severek isteyerek bir bölümde okumanın yolu ise bilindiği gibi sınavlardan geçer. Son açıklanan sınav sistemi
YKS'de (Yüksek Öğretim Kurumları Sınavı) matematiğin rolü daha da artmıştır.

Bu durum göz önünde bulundurularak farklı tarzdaki sorulardan oluşturduğum YKS 3D Matematik Soru Bankası
(2. oturum) kitabımın siz değerli öğrencileri başarıya ulaştıracağına inancım sonsuzdur.

Bu kitabın oluşumunda;

• Bir bölüme ait çok sayıda alt başlık oluşturularak hazırlanan testlerle konu içerikleri eksiksiz hazırlanmıştır.

• Bir test içerisindeki sorular kolaydan zora 3D tekniğine uygun olarak hazırlanmıştır.

• Her testin soruları genelden özele bilgi düzeyinizi artırmak üzere tasarlanmıştır.

• Özgün ve hedefe uygun sorular kullanılmıştır.

• ÖSYM soruları titizlikle analiz edilerek her bölüme ait bire bir ÖSYM testleri hazırlanmıştır.

• TÜMEVARIM testleriyle öğrencilerin konuyu geriye doğru dinamik bir şekilde taraması sağlanmıştır.

Kitabın hazırlık aşamasında emeği geçen yayın ekibine, fikirler ile desteklerini esirgemeyen meslektaşlarıma ve
Çözüm Yayınları yöneticilerine teşekkürlerimi sunarım.

Kitapla ilgili her türlü fikrinizicelalsaltas@cozumyayinlari.com.tr mail adresinden tarafıma iletmeniz beni mutlu eder.

Üniversiteye giriş sınavında ve hayatın her alanında başarı ve mutluluk dileklerimle…

Celal SALTAŞ
İÇİNDEKİLER

01. BÖLÜM: POLİNOMLAR 06. BÖLÜM: DİZİLER


Polinomlar......................................................................................7 Dizi Kavramı...............................................................................151

Çarpanlara Ayırma........................................................................15 Aritmetik Dizi...............................................................................155

Rasyonel İfadeler........................................................................19 Geometrik Dizi............................................................................159

Bire Bir ÖSYM.............................................................................25 Bire Bir ÖSYM...........................................................................163

07. BÖLÜM: LİMİT VE SÜREKLİLİK


02. BÖLÜM: MANTIK Limit Kavramı.............................................................................165
Önerme ve Bileşik Önermeler......................................................29 Süreklilik.....................................................................................173
Açık Önermeler............................................................................37 Bire Bir ÖSYM...........................................................................177
Bire Bir ÖSYM.............................................................................41 TÜMEVARIM - III........................................................................181

08. BÖLÜM: TÜREV


03. BÖLÜM: MODÜLER ARİTMETİK Türev Alma Kuralları...................................................................191
Bölme ve Bölünebilme..................................................................43 Teğet ve Normal Denklemleri.....................................................211
Modüler Aritmetikte İşlemler.........................................................55 Artan - Azalan Fonksiyon, Mutlak ve Yerel, Maksimum ve
Bire Bir ÖSYM.............................................................................63 Minimum Noktaları.....................................................................221

TÜMEVARIM - I............................................................................67 Maksimum ve Minimum Problemleri...........................................231

Fonksiyonun Grafikleri................................................................237

Bire Bir ÖSYM...........................................................................241

TÜMEVARIM - IV.......................................................................247
04. BÖLÜM: DENKLEM VE EŞİTSİZLİK SİSTEMLERİ
Doğrusal Denklem Sistemlerinin Çözümleri.................................73

İkinci Dereceden Denklemler.......................................................79 09. BÖLÜM: İNTEGRAL

Karmaşık Sayılar..........................................................................85 Belirli ve Belirsiz İntegral, Belirli İntegralin Özellikleri.................257

İkinci Dereceden Fonksiyonlar.....................................................87 İntegral Alma Kuralları, Belirsiz İntegralin Özellikleri..................263

İkinci Dereceye Dönüştürülebilen Denklemler..............................93 İntegral Alma Yöntemleri............................................................267

İkinci Dereceden Bir Bilinmeyenli Eşitsizlikler..............................97 Belirli İntegralin Uygulamaları.....................................................275

Denklem ve Eşitsizlik Sistemleri.................................................109 Bire Bir ÖSYM...........................................................................285

Bire Bir ÖSYM...........................................................................113


10. BÖLÜM: SAYMA VE OLASILIK
Permütasyon..............................................................................289

05. BÖLÜM: ÜSTEL VE LOGARİTMİK FONKSİYONLAR Seçme (Kombinasyon)...............................................................291

Üstlü Sayılarda İşlemler, Üstel Fonksiyon..................................119 Tekrarlı Permütasyon.................................................................295

Logaritma Fonksiyonu................................................................123 Dairesel Permütasyon................................................................297

Üstel ve Logaritmik Denklem ve Eşitsizlikler..............................131 Deneysel ve Teorik Olasılık........................................................299

Bire Bir ÖSYM...........................................................................137 Bire Bir ÖSYM...........................................................................305

TÜMEVARIM - II.........................................................................141 TÜMEVARIM - V........................................................................309


Polinomlar BÖLÜM 01 Test 01
1. Bir P(x) polinomu için, 5. Bir P(x) polinomu için,

P(x – 1) = 2x2 – 4x + 5 P(x + 2) = x3 + 4x2 + 2x + m – 3

olduğuna göre, P(2) kaçtır? tür. P(x) polinomunun sabit terimi 5 olduğuna göre, m
kaçtır?
A) 8 B) 9 C) 10 D) 11 E) 12
x = 3 için A) 4 B) 5 C) 6 D) 7 E) 8
P(2) = 18 – 12 + 5 = 11 bulunur. P(0) = 5
(Cevap D) x = –2 için P(0) = –8 + 16 – 4 + m – 3 = 5
m+1=5
m = 4 bulunur. (Cevap A)

2. P(x) = 2x4 – 3x2 + x + 5


6. P(x) = (a – 2)x3 + 4x2 + (b + 1)x + 1
polinomunun baş katsayısı a, derecesi b ve sabit terimi c
Q(x) = 4x2 + (a + 3)x + 1
olduğuna göre, a + b + c toplamı kaçtır?
polinomları için P(x) = Q(x) olduğuna göre, a + b toplamı
A) 8 B) 9 C) 10 D) 11 E) 12
kaçtır?
a = 2, b = 4 ve c = 5 tir.
a + b + c = 11 bulunur. (Cevap D) A) 5 B) 6 C) 7 D) 8 E) 9
a – 2 = 0 ⇒ a = 2 ve
b + 1 = a + 3 ⇒ b + 1 = 2 + 3
⇒ b = 4 bulunur.
a + b = 2 + 4 = 6 dır. (Cevap B)
Çözüm Yayınları

3. Bir P(x) polinomu için,

P(x – 1) = 2x3 – 3x2 + 4x – 2 7. P(x) = x2 + 3x – 2


olduğuna göre, P(x + 2) polinomunun sabit terimi kaçtır? olduğuna göre, P(x2 – 1) polinomu aşağıdakilerden
A) 24 B) 27 C) 32 D) 35 E) 37 hangisidir?

x = 0 için P(x + 2) polinomunun sabit terimi P(2) dir. A) x4 – x + 2 B) x4 – 2x – 4


P(x – 1) polinomunda x yerine 3 yazılır.
C) x4 + 2x – 2 D) x4 + x – 2
x = 3 için P(2) = 54 – 27 + 12 – 2
= 27 + 10 E) x4 + x2 – 4
= 37 (Cevap E)
(x2 – 1)2 + 3(x2 – 1) – 2 = x4 – 2x2 + 1 + 3x2 – 3 – 2
= x4 + x 2 – 4 (Cevap E)

4. P(x) ve Q(x) birer polinom olmak üzere,

der P( x 2 ) ⋅ Q( x 3 ) = 14
 
 P( x )  8. Her x gerçek sayısı için,
der  =2
Q( x ) 
 6x2 + 6x – 12 = (2x + a) (bx – 3)

olduğuna göre, der[P(x) + Q(x)] kaçtır? olduğuna göre, a + b toplamı kaçtır?

A) 3 B) 4 C) 5 D) 6 E) 7 A) 6 B) 7 C) 8 D) 9 E) 10
der[P(x)] = a ve der [Q(x)] = b olsun. 6x2 + 6x – 12 = 2bx2 – 6x + abx – 3a –3a = –12 ⇒ a = 4 ab – 6 = 6
2a + 3b = 14  = 2bx2 + (ab – 6)x – 3a 2b = 6 ⇒ b = 3 4b = 12
 ⇒ a = 4 ve b = 2 bulunur.
a−b = 2  b = 3 sağlar.
der[P(x) + Q(x)] = 4 (Toplamının derecesi yüksek dereceli polinomun derecesi olur.) a + b = 7 dir. (Cevap B)
(Cevap B)
Test 01 1. D 2. D 3. E 4. B 5. A 6. B 7. E 8. B 9. C 10. C 11. A 12. C 13. B 14. D 15. B 16. E

9. P(x) = xn+3 + 4x7–n + x2 + 3 13. Bir P(x) polinomu için,

ifadesi bir polinom olduğuna göre, n nin alabileceği tam P(x + 1) + P(2x) = 6x – 4
sayı değerlerinin toplamı kaçtır?
olduğuna göre, P(5) kaçtır?
A) 20 B) 21 C) 22 D) 23 E) 24
A) 6 B) 7 C) 8 D) 9 E) 10
n + 3 ≥ 0 ⇒ n ≥ –3 P(x) birinci dereceden bir polinomdur.
7 – n ≥ 0 ⇒ –n ≥ –7 P(x) = ax + b olsun.
⇒n≤7 a(x + 1) + b + a(2x) + b = 6x – 4
Buna göre, –3 ≤ n ≤ 7 olmalıdır. n nin alabileceği tam sayı değerlerinin toplamı 3ax + a + 2b = 6x – 4
–3 – 2 – 1 + 0 + 1 + 2 + 3 + 4 + 5 + 6 + 7 = 22 dir. (Cevap C) 3a = 6 ⇒ a = 2 ve
a + 2b = – 4 ⇒ b = –3 bulunur.

P(x) = 2x –3 ⇒ P(5) = 7 dir. (Cevap B)

14. Bir P(x) polinomu için,


b−4 P(x) – P(x – 1) = x3
10. P(x) = x3 + (a – 2) x + +1
x
ifadesi bir polinom olduğuna göre, a + b toplamı kaçtır? tür. P(2x + 1) polinomunun x – 2 ile bölümünden kalan,
P(x) polinomunun katsayıları toplamından kaç fazladır?
A) 2 B) 5 C) 6 D) 8 E) 10
A) 204 B) 210 C) 215 D) 224 E) 230

x = 2, P(2) – P(1) = 8
a−2 =0 ⇒ a = 2 
 ⇒ a + b = 6 bulunur. x = 3, P(3) – P(2) = 27
b−4 =0 ⇒ b = 4 
(Cevap C) x = 4, P(4) – P(3) = 64
x = 5, P(5) – P(4) = 125
Çözüm Yayınları

+ +
P(5) – P(1) = 224 bulunur.
(Cevap D)

15. P(x) = x4 + 4x3 + 6x2 + 4x + a


11. P(x – 2) polinomunun x – 1 ile bölümünden kalan 5,
x – 4 ile bölümünden kalan 8 dir. polinomu için P( 3 − 1) = 2 olduğuna göre, a kaçtır?

Buna göre, P(x) polinomunun x2 – x – 2 ile bölümünden A) –8 B) –6 C) –4 D) 2 E) 4


kalan kaçtır?
P(x) = (x + 1)4 –1 + a
A) x + 6 B) x + 4 C) 2x + 1 ( 3 –1 + 1)4 – 1 + a = 2

D) x – 3 E) 2x – 3 8+a=2

a = –6 bulunur.
P(x – 2) = (x – 1) (x – 4) . Q(x) + ax + b (Cevap B)
x = 1 için, a + b = 5
⇒ a = 1 ve b = 4 bulunur.
x = 4 için, 4a + b = 8

x yerine x + 2 yazılarak
P(x) = (x + 1) (x–2) . Q(x + 2) + b 16. P(x) = (x4 – 3x2 – 4x + 3)3
8 = 1 . (x + 2) + 4
polinomu x – 2 ile bölündüğünde, elde edilen bölümün
= x + 6 elde edilir. (Cevap A)
katsayıları toplamı kaç olur?

A) 9 B) 16 C) 21 D) 24 E) 26
24 P(x) polinomunun x – 2 ile bölümünden kalan P(2) = –1 dir. P(x) + 1 polinomu x – 2 ile
12. P( x ) = x n−3 + 2x n − 3x + 4 tam bölünür.

ifadesi bir polinom olduğuna göre, n nin alabileceği kaç Bu bölüm Q(x) olsun.

farklı doğal sayı değeri vardır? P(x) + 1 P(1) + 1


Q(x) = ⇒ Q(1) =
x–2 –1
A) 4 B) 5 C) 6 D) 7 E) 8 (–3)3 +1
⇒ Q(1) =
–1
24
n – 3 ≥ 0 ⇒ n ≥ 3 ve doğal sayı olmalıdır.
n ⇒ Q(1) = 26 bulunur.
n ! {3, 4, 6, 8, 12, 24} olur. n nin alabileceği 6 farklı değer vardır. (Cevap E)

(Cevap C)
Polinomlar BÖLÜM 01 Test 02
1. P(x) = 2x3 – 4x2 + 5x – a 5. P(x) = 2x4 + x3 – 3x2 – 1
polinomunun bir çarpanı x – 1 olduğuna göre, P(–1) polinomunun (x – 1) (x + 1) çarpımına bölümünden kalan
kaçtır? aşağıdakilerden hangisidir?

A) –15 B) –14 C) –13 D) –12 E) –11 A) x – 2 B) x – 1 C) x + 1


P(1) = 0 ⇒ 2 – 4 + 5 – a = 0
D) x + 2 E) 2x + 1
⇒ a = 3 bulunur. P(x) = (x – 1) (x + 1) . Q(x) + ax + b
P(–1) = –2 – 4 – 5 – a ⇒ P(–1) = –11 – 3 P(1) = a + b ⇒ a + b = –1 ⇒ a = 1, b = –2
⇒ P(–1) = –14 P(–1) = –a + b ⇒ –a + b = –3
(Cevap B) ax + b = x – 2 bulunur.
(Cevap A)

2. Bir P(x) polinomu için,


P(x – 2) = x2 – x + 7
6. P(x) = ax5 – 3x2 + b
dir.
polinomunun x2 + 1 ile bölümünden kalan 2x + 8
Buna göre, P(x + 1) polinomunun (x – 3) ile bölümünden olduğuna göre, a + b toplamı kaçtır?
kalan kaçtır?
A) 5 B) 6 C) 7 D) 8 E) 9
A) 27 B) 30 C) 33 D) 35 E) 37
x2 2
+ 1 = 0 ⇒ x = –1 yazalım.
P(x + 1) polinomunun x – 3 ile bölümünden kalan x = 3 için P(4) tür. a(x2)2 . x – 3x2 + b = 2x + 8 ⇒ ax + 3 + b = 2x + 8
P(x – 2) polinomunda x yerine 6 yazılarak P(4) = 36 – 6 + 7 = 37 bulunur. ⇒ a = 2, b + 3 = 8
(Cevap E) ⇒ a = 2, b = 5
a + b = 7 bulunur.
(Cevap C)
Çözüm Yayınları

3. P(x) = 2x3 – ax2 + bx – 2


polinomu x2 – 1 ile kalansız bölünebildiğine göre, x + 3 ile
bölümünden kalan kaçtır? 7. a ve b birer doğal sayı olmak üzere,
A) –32 B) –30 C) –24 D) 12 E) 20 P(x) = (x – 1)2a+1 + (x – 1)2a+2 – 2b+1
x2 = 1 için P(x) = 0 olur. polinomu x + 1 ile kalansız bölünebildiğine göre, a ile b
2 . 1 . x – a + bx – 2 = 0 ⇒ (b + 2) x – a – 2 = 0 arasındaki bağıntı aşağıdakilerden hangisidir?
⇒ b + 2 = 0 ve –a – 2 = 0
⇒ b = –2 ve a = –2 A) a + b = 2 B) a + 2b = 0 C) a = 2b
bulunur. D) 2a = b E) 2a = b + 1
P(x) = 2x3 + 2x2 – 2x – 2 olur.
P(–1) = (–2)2a+1 + (–2)2a+2 – 2b+1 = 0
P(–3) = –54 + 18 + 6 – 2 = –32 dir.
–22a+1 + 22a+2 = 2b+1
(Cevap A)
22a+1 = 2b+1
2a + 1 = b + 1
2a = b
(Cevap D)

4. Bir P(x) polinomu için,


P(x) + P(x + 2) = –2x2 + 6x + 6 9
olduğuna göre, P(1) – P(–1) ifadesinin değeri kaçtır?
8. Bir P(x) polinomu için,
A) –3 B) –1 C) 8 D) 10 E) 11
P(x + 1) + P(x – 2) = 4x – 6
P(x) ikinci dereceden bir polinomdur.
olduğuna göre, P(3) kaçtır?
P(x) = ax2 + bx + c olsun.
ax2 + bx + c + a (x + 2)2 + b(x + 2) + c = –2x2 + 6x + 2 A) –4 B) –2 C) 2 D) 4 E) 6
2ax2 + (4a + 2b) x + 4a + 2b + c = –2x2 + 6 x + 2 P(x) birinci dereceden bir polinomdur.
2a = –2 ⇒ a = –1 P(x) = ax + b olsun.
4a + 2b = 6 ⇒ b = 5 a(x + 1) + b + a(x – 2) + b = 4x – 6
4a + 2b + c = 2 ⇒ c = –7 bulunur. 2ax + 2b – a = 4x – 6
P(x) = x2 + 5 x –7 ⇒ P(1) – (P–1) = 10 dur. 2a = 4 ⇒ a = 2,
(Cevap D) 2b – a = –6 ⇒ b = –2 bulunur.
P(x) = 2x – 2 ⇒ P(3) = 4 tür.
(Cevap D)
Test 02 1. B 2. E 3. A 4. D 5. A 6. C 7. D 8. D 9. B 10. D 11. B 12. A 13. D 14. A 15. B 16. D

9. P(x) polinomunun x2 – 2 ile bölümünden elde edilen bölüm 13. P(x) polinomunun x + 1 ile bölümünden kalan 6,
x2 + 4 ve kalan 2x + 1 dir. Q(x) polinomunun x – 2 ile bölümünden kalan 3 tür.

Buna göre, P(x) polinomunun katsayılar toplamı kaçtır? P( x − 2)


= ax 2 + 3 x − 5
Q( x + 1)
A) –3 B) –2 C) –1 D) 1 E) 2
P(x) = (x 2 – 2) (x2 + 4) + 2x + 1 olduğuna göre, a kaçtır?
P(1) = (–1) . (5) + 2 + 1 = –2
(Cevap B) A) –3 B) –2 C) 3 D) 4 E) 5
P(–1) = 6 ve Q(2) = 3 tür. Verilen eşitlikte x yerine 1 yazılır.
P(–1) 6
=a+3–5⇒ =a–2
Q(2) 3
⇒ a = 4 tür.
(Cevap D)

10. P(x) polinomunun x2 + 2x – 3 ile bölümünden kalan


5x + 2 olduğuna göre, 3x2 + P(x) polinomunun x – 1 ile
bölümünden kalan kaçtır?
14. P(x) polinomunun x4 – 1 ile bölümünden kalan
A) 4 B) 7 C) 8 D) 10 E) 12 x3 + ax2 + 2x – 3 ve x2 + 1 ile bölümünden kalan x + 1
olduğuna göre, a kaçtır?
P(x) = (x2 + 2x –3) . Q(x) + 5x + 2 ⇒ P(1) = 7 dir.
Q(x) = 3x2 + P(x) olsun. A) –4 B) –2 C) 1 D) 3 E) 5
Q(1) = 3 + P(1) = 3 + 7 = 10 bulunur. P(x) = (x 2 –1) (x2 + 1) . Q(x) + x3 + ax2 + 2x – 3
(Cevap D)
x2 = –1 için –x – a + 2x –3 = x + 1
–a = 4
Çözüm Yayınları

a = –4 bulunur.
(Cevap A)

11. P(x) ve Q(x) polinomları için,

P(x + 1) = x · Q(–x) + 5 15. Baş katsayısı 2 olan üçüncü dereceden bir P(x) polinomunun
bir çarpanı x2 – 2x + 4 tür.
tir. P(x) polinomunun sabit terimi 15 olduğuna göre, Q(x)
polinomunun x – 1 ile bölümünden kalan kaçtır? P(x) polinomunun x – 2 ile tam bölünebildiğine göre, P(x)
polinomunun katsayıları toplamı kaçtır?
A) –11 B) – 10 C) –9 D) –8 E) –7
A) –9 B) –6 C) –2 D) 3 E) 5
P(0) = 15 tir. Verilen eşitlikte x yerine –1 yazılır.
P(0) = –1 . Q(1) + 5 ⇒ 15 = –Q(1) + 5 P(x) = 2(x2 – 2x + 4) (x – 2)

⇒ Q(1) = –10 P(1) = 2 . (1 – 2 + 4) . (–1) = –6 bulunur.


(Cevap B)
bulunur.
(Cevap B)

10

12. P(x) = x3 + ax + b
16. P(x) polinomunun x2 + 1 ile bölümünden kalanı x – 1
polinomunun bir çarpanı x2 – 3x + 1 olduğuna göre, olduğuna göre, P3(x) polinomunun x2 + 1 ile bölümünden
(a, b) ikilisi aşağıdakilerden hangisidir? kalan kaçtır?
A) (–8, 3) B) (–8, 5) C) (–4, 3) A) 2x – 3 B) 2x – 1 C) 2x + 1

D) (–3, 4) E) (–3, 8) D) 2x + 2 E) 2x + 3
x3 + ax + b = (x + c) (x2 –3x + 1) Kalanın küpünün x2 + 1 ile bölümünden kalan bulunur.
x3 + ax + b = x3 + (c – 3) x2 + (1 – 3c) x + c (x – 1)3 = x3 – 3x2 + 3x –1 polinomunda
c – 3 = 0 ⇒ c = 3, x2 + 1 = 0 ⇒ x2 = –1 yazalım.
a = 1 – 3 c ⇒ a = – 8 ve –1 . x + 3 + 3x –1 = 2x + 2 elde edilir.
(Cevap D)
b = c = 3 bulunur. (a, b) = (–8, 3) (Cevap A)
Polinomlar BÖLÜM 01 Test 03
1. P(x) = x7 – 2x6 + 7x – 5 5. P(x) = x4 + x3 – 3x2 + ax + b
polinomunun x2 – x – 2 ile bölümünden kalan polinomunun x2 + x – 2 ile bölümünden kalan 3x – 1
aşağıdakilerden hangisidir? olduğuna göre, a + b toplamı kaçtır?

A) 2x – 13 B) 4x – 5 C) 5x – 3 A) –3 B) –2 C) –1 D) 2 E) 3
P(x) = (x2 + x – 2) . Q(x) + 3x – 1
D) 6x – 1 E) 8x – 7
P(1) = ⇒ 1 + 1 – 3 + a + b = 2 ⇒ a + b = 3 bulunur.
P(x) = (x – 2) (x + 1) . Q(x) + ax + b
(Cevap E)
P(2) = 27 – 2 . 26 + 14 – 5 = 2a + b ⇒ 2a + b = 9
P(–1) = –1 –2 –7 –5 = –a + b ⇒ –a + b = –15
denklem sisteminin çözümünden a = 8 ve b = –7 bulunur.
ax + b = 8x –7 elde edilir.
(Cevap E)

2. P(x) ve Q(x) birer polinom olmak üzere,


P(x) Q(x)
x6 – 4x 6. P(x) · P(x + 2) = x2 – 6x + 8
x2 + 3x – 2 olduğuna göre, P(2) kaçtır?

bölme işlemine göre, P(x) polinomunun derecesi en az A) –2 B) –1 C) 0 D) 1 E) 2


kaçtır?
P(x) . P(x + 2) = (x – 4) (x – 2)
A) 7 B) 8 C) 9 D) 15 E) 18 P(x) = x – 4 alınırsa, P(x + 2) = x – 2 olur.
Kalan 2. dereceden olduğundan, bölenin derecesi en az 3 olur. P(x) polinomunun derecesi P(2) = –2 dir.
(Cevap A)
de en az 3 + 6 = 9 olur.
(Cevap C)
Çözüm Yayınları

3. Bir P(x) polinomunun x2 – 2x – 3 ile bölümünden kalan


2x + 3 ve x + 3 ile bölümünden kalan –13 olduğuna
göre, x2 + 4x + 3 ile bölümünden kalan aşağıdakilerden
hangisidir? 7. Bir P(x) polinomunun x – 1 ile bölümünden kalan 7,
A) 2x + 3 B) 4x – 1 C) 7x + 8 x + 2 ile bölümünden kalan 1 olduğuna göre, x2 + x – 2 ile
bölümünden kalan aşağıdakilerden hangisidir?
D) 5x + 5 E) 3x + 4
A) 2x – 5 B) 2x – 3 C) 2x + 1
P(x) = (x2 – 2x –3) . Q(x) + 2x + 3
P(–1) = 1 dir. D) 2x + 3 E) 2x + 5
P(x) polinomunun x2 + 4x + 3 ile bölümümden kala ax + b olsun.
P(x) = (x2 + 4x + 3) . Q(x) + ax + b P(x) = (x2 + x –2) . Q(x) + ax + b

P(–3) = –13 ⇒ –3a + b = –13 P(1) = 7 ⇒ a + b = 7


⇒ a = 7 ve b = 8 ⇒ a = 2 ve b = 5
P(–1) = 1 ⇒ –a + b = 1 P(–2) = 1 ⇒ –2a + b = 1

bulunur. Kalan ax + b = 7x + 8 dir. bulunur. Kalan ax + b = 2x + 5 tir.


(Cevap C) (Cevap E)

4. P(x) = x3 + x2 – 4x – 4
Q(x) = x3 – 2x2 – x + 2

olduğuna göre, 11
EKOK [P( x ), Q( x )]
EBOB [P( x ), Q( x )] 8. P(x) ve Q(x) polinomları için,

der P 2 ( x ) ⋅ Q( x ) = 10
ifadesinin sadeleştirilmiş biçimi aşağıdakilerden  
hangisidir?  P3 ( x) 
der   = 10
A) x2 – x – 2 B) x2 + x – 2 C) x2 – 4 Q( x ) 

D) x2 – 1 E) x2 – 2x olduğuna göre, der [P(x) + Q(x)] kaçtır?
P(x) = x2(x + 1) – 4(x + 1) = (x + 1) (x – 2) (x + 2)
A) 4 B) 5 C) 6 D) 7 E) 8
Q(x) = x2(x – 2) – (x – 2) = (x – 2) (x – 1) (x + 1)
der[P(x)] = a, der [Q(x)] = b olsun.
EKOK [P(x) , Q(x)] (x – 1) (x – 2) (x + 1) (x + 2)
= 2a + b = 10
EBOB [P(x) , Q(x)] (x – 2) (x + 1) ⇒ a = 4 ve b = 2
3a – b = 10
= (x – 1) (x + 2)
der[P(x) + Q(x)] = 4 olur.
= x2 + x – 2 (Cevap B) (Cevap A)
Test 03 1. E 2. C 3. C 4. B 5. E 6. A 7. E 8. A 9. C 10. A 11. C 12. D 13. E 14. A 15. E 16. E

24 13. P(x) ve Q(x) birer polinom olmak üzere,


9. P( x ) = x 7 −n + 4 x n −3
P(2x + 3)
= x2 + x + 5
ifadesi bir polinom olduğuna göre, n nin alabileceği tam Q( x )
sayı değerlerinin toplamı kaçtır?
veriliyor.
A) 12 B) 15 C) 16 D) 18 E) 20
Q(x) polinomunun x – 2 ile bölümünden kalan 4 olduğuna
7–n≥0⇒n≤7
göre, P(x) polinomunun x – 7 ile bölümünden kalan
n, 24 ün bölenleri ve n ≤ 7 olmalıdır. n nin alabileceği tam sayı değerlerinin toplamı
kaçtır?
1 + 2 + 3 + 4 + 6 = 16 dır.
(Cevap C)
A) 27 B) 30 C) 35 D) 42 E) 44
P(7) P(7)
x = 2 için =4+2+5⇒ = 11
Q(2) 4
⇒ P(7) = 44
elde edilir.
(Cevap E)

10. Bir P(x) polinomu için,


14. P(x) = x4 – 4x3 + ax2 + bx + 4
P(2x + 3) = x2 – mx + 5
polinomu (x – 2)2 ile tam bölünebildiğine göre, a · b
tir.
çarpımı kaçtır?
P(x – 2) polinomunun x – 3 ile bölümünden kalan
A) –20 B) –18 C) –15 D) 12 E) 16
7 olduğuna göre, m kaçtır?
Bölüm 2. dereceden bir polinomdur.
A) 1 B) 2 C) 3 D) 4 E) 5 P(x) = (x – 2)2 (x2 + cx + d)
P(x – 2) polinomunun x – 3 ile bölümün kalanı bulmak için x yerine 3 yazılır. P(1) dir. P(x) = x4 + (c – 4) x3 + (d – 4c + 4) x2 + (4c – 4d) x + 4d
P(2x + 3) polinomunda x yerine –1 yazılarak c – 4 = –4 ⇒ c = 0
P(1) = 7 ⇒ 1 + m + 5 = 7 4d = 4 ⇒ d = 1
Çözüm Yayınları

⇒ m = 1 bulunur. a = d – 4c + 4 ⇒ a = 5 ve
(Cevap A)
b = 4c – 4d ⇒ b = –4 bulunur.
a. b = –20 dir. (Cevap A)

15. n pozitif tam sayı ve


P(x – 1) = (x + 1)n – x2n + 7

11. Bir P(x) polinomu için, olduğuna göre, P(x) polinomunun x2 + 3x + 2 ile
bölümünden kalan kaçtır?
(x – 2) · P(x) = x2 – mx – 10
A) –2x + 10 B) –2x + 6 C) 2x + 4
olduğuna göre, P(2) kaçtır?
D) 2x + 6 E) 2x + 10
A) 5 B) 6 C) 7 D) 8 E) 9
P(x) = (x2 + 3x + 2) . Q(x) + ax + b
x = 2 için 0 = 4 – 2m – 10 ⇒ m = – 3 bulunur.
P(x) = (x + 1) (x + 2) . Q(x) + ax + b
(x – 2) . P(x) = x2 + 3x – 10 ⇒ (x – 2) . P(x) = (x – 2) (x + 5)
P(–1) = –a + b = 1n – 0 + 7 ⇒ –a + b = 8
⇒ P(x) = x + 5
P(–2) = –2a + b = 0 – (–1)2n + 7 ⇒ –2a + b = 6
⇒ P(2) = 7
(Cevap C) denklem sisteminin çözümünden a = 2 ve b = 10 bulunur.
ax + b = 2x + 10 elde edilir.
(Cevap E)
12
16. P(x) polinomunun x2
– x + 1 ile bölümünden kalan
x + 1 olduğuna göre, P3(x) polinomunun x2 – x + 1 ile
bölümünden kalan kaçtır?

A) 3x – 1 B) 3x + 1 C) 6x – 1
12. P(x) polinomunun x2 – 3x ile bölümünden kalan x + 2
D) 6x – 2 E) 6x – 3
olduğuna göre, P2(x) polinomunun x2 – 3x ile
bölümünden kalan aşağıdakilerden hangisidir? P3(x) polinomunun x2–x + 1 ile bölümünden kalan, (x + 1)3 nün x2 – x + 1 ile bölümünden
kalana eşittir.
A) 4x + 2 B) 4x + 4 C) 7x – 4 (x + 1)3 =x3 + 3x2 + 3x + 1

D) 7x + 4 E) 9x + 7 polinomunda x2 – x + 1 = 0 ⇒ x2 = x – 1 yazılır.
K(x) = (x – 1) x + 3 (x – 1) +3x + 1
P(x) = [(x2 –3x) . Q(x) + x + 2]2
=x2 – x + 3x –3 + 3x + 1
Kare alındığında (x + 2)2 dışındaki terimler x2 – 3x ile tam bölündüğünden (x + 2)2 nin x2 – 3x
=(x–1) + 5x – 2 = 6x –3
ile bölümümden kalan bulunur.
(Cevap E)
(x + 2)2 = x2 + 4x + 4 polinomunda x2 – 3x = 0 ⇒ x2 = 3x yazılır.
Kalan 3x + 4x + 4 = 7x + 4 tür. (Cevap D)
Polinomlar BÖLÜM 01 Test 04
1. P(x) = x3 + 4x + m 5. P(x) polinomu birinci dereceden bir polinomun küpüdür.

polinomu x – 1 ile bölündüğünde bölüm Q(x) ve kalan 6 dır. P(x) polinomunun x ile bölümünden kalan 8 ve
x – 1 ile bölümünden kalan 1 olduğuna göre, P(x – 3)
Buna göre, Q(x) polinomunun x + 1 ile bölümünden kalan
polinomunun x – 9 ile bölümünden kalan kaçtır?
kaçtır?
A) –64 B) –27 C) –8 D) 16 E) 64
A) 3 B) 4 C) 5 D) 6 E) 7
P(x) = (ax + b)3
P(1) = 6 ⇒ 1 + 4 + m = 6
P(0) = 8 ⇒ b3 = 8
⇒ m = 1 bulunur.
⇒b=2
x3 + 4x + 1 = (x – 1) . Q(x) + 6
P(1) = 1 ⇒ (a + b)3 = 1
x = –1 için –1 – 4 + 1 = –2 . Q(–1) + 6 ⇒ Q(–1) = 5 bulunur.
(Cevap C) =a+2=1
= a = –1
P(x) = (–x + 2)3 bulunur.
P(x–3) polinomunun x–9 ile bölümünden kalan P(6) dır.
P(6) = (–6 + 2)3 = –64 (Cevap A)

6. n ∈ N olmak üzere,
2. P(x) = (x3 + 2x2 – 3x + 4)2
P(x) = x2n+1 + 5x2n – 162
polinomunda x4 lü terimin katsayısı kaçtır?
polinomunun çarpanlarından biri x + 3 olduğuna göre,
A) –2 B) –1 C) 4 D) 6 E) 8
derecesi kaçtır?
(x3 + 2x2 – 3x + 4) (x3 + 2x2 – 3x + 4)
A) 3 B) 4 C) 5 D) 6 E) 7
x3(–3x) + (2x2) (2x2) – (3x) x3 = –3x4 + 4x4 – 3x4
= –2x4 P(–3) = (–3)2n+1 + (–3)2n – 162 = 0
(Cevap A) –3(–3)2n + 5 (–3)2n = 162
Çözüm Yayınları

2(–3)2n = 162
32n = 34
n = 2 bulunur.
der[p(x)] = 2n + 1 = 5 tir.
(Cevap C)

3. Her x gerçek sayısı için,

ax3 + bx2 + cx + 2 = (2x2 + 1) (x + 2) + 3x 7. Bir P(x) polinomunun x3 – 8 ile bölümünden kalan


olduğuna göre, a + b + c toplamı kaçtır? x2 + 2x – 3 olduğuna göre, x – 2 ile bölümünden kalan
kaçtır?
A) 10 B) 9 C) 8 D) 7 E) 6
A) 1 B) 3 C) 5 D) 6 E) 8
x = 1 için, a + b + c + 2 = (3) . (3) + 3
P(x) = (x3 – 8) . Q(x) + x2 + 2x –3
a + b + c = 10 bulunur.
(Cevap A) P(2) = 0 + 4 + 4 – 3 = 5 bulunur.
(Cevap C)

13

8. P(x) = 3x3 + 4x2 + mx + n

polinomu (x + 2)2 ile tam bölünebildiğine göre, m · n


4. Bir P(x) polinomunun x2 – 4 ile bölümünden kalan 2x + 7 dir. çarpımı kaçtır?

Buna göre, (x + 4) · P(x) polinomunun x + 2 ile A) 360 B) 420 C) 480 D) 520 E) 640
bölümünden kalan kaçtır? 3x3 2 2
+ 4x + mx + n = (x + 2) . (3x + b)
= (x2 + 4x + 4) (3x + b)
A) –6 B) –4 C) 3 D) 5 E) 6
= 3x3 + (b + 12) x2 + (4b + 12) x + 4b
P(x) = (x2 – 4) . Q(x) + 2x + 7 ⇒ P(–2) = 3
b + 12 = 4 ⇒ b = – 8
(x + 4) . P(x) polinomunun x + 2 ile bölümünden kalan x = –2 için
m = 4b + 12 ⇒ m = –20
(–2 + 4) . P(–2) = 2 . 3 ⇒ P(–2) = 6 dır. ⇒ m . n = 640
(Cevap E) n = 4b ⇒ n = –32
(Cevap E)
Test 04 1. C 2. A 3. A 4. E 5. A 6. C 7. C 8. E 9. B 10. A 11. E 12. E 13. C 14. A 15. A 16. C

9. P(x) = x10 + 2x5 + 3x – 1 13. (x2 – 1) · P(x – 2) = x4 + ax2 + bx + 6

polinomunun x5 + 1 ile bölümünden kalan olduğuna göre, P(x) polinomunun x + 4 ile bölümünden
aşağıdakilerden hangisidir? kalan kaçtır?

A) 3x – 1 B) 3x – 2 C) x2 – 3 A) –4 B) –3 C) –2 D) 1 E) 2
x = 1 için, 0 = 1 + a + b + 6 ⇒ a + b = –7
D) x2 + x E) x3 – 1
x = –1 için 0 = 1 + a – b + 6 ⇒ a – b = –7
P(x) = (x5 + 1) . Q(x) + K(x)
a = –7 ve b = 0 bulunur.
x5 + 1 = 0 ⇒ x5 = –1 yazılırsa K(x) elde edilir.
(x2 – 1) . P(x – 2) = x4 – 7x2 + 6 ⇒ P(x – 2) = x2 – 6 olur.
P(x) = (x5)2 + 2x5 + 3x –1
x = –2 için P(–4) = 4 – 6 = –2 bulunur.
x5 = –1 için K(x) = (–1)2 + 2(–1) + 3x – 1
(Cevap C)
K(x) = 3x – 2 bulunur.
(Cevap B)

10. P(x) ve Q(x) polinomları için, 14. x5 – 3x3 + 2x2 – 5 ≡ ax + b [mod (x2 + 1)]
P( x + 2) olduğuna göre, a + b toplamı kaçtır?
= 2x + 5
( x − 2)
Q
A) –3 B) –2 C) 2 D) 3 E) 4
x2 + 1 = 0 ⇒ x2 = –1 yazılır.
tir.
(x2)2 . x – 3x2 . x + 2x2 – 5 = ax + b [mod(x2 + 1)]
P(x) polinomunun katsayıları toplamı 12 olduğuna göre, x + 3x – 2 – 5 = ax + b ⇒ 4x –7 = ax + b
Q(x) polinomunun x + 3 ile bölümünden kalan kaçtır? ⇒ a = 4 ve b = –7
a + b = 4 – 7 = –3 bulunur.
A) 4 B) 5 C) 6 D) 7 E) 8
(Cevap A)
P(1) = 12 dir.
Verilen eşitlikte x yerine –1 yazılır.
P(1) 12
Çözüm Yayınları

x = –1 için, = –2 + 5 ⇒ =3
Q(–3) Q(–3)

⇒ Q(–3) = 4
(Cevap A)

15. P(x) = x9 – x6 + ax + b
olmak üzere,

P(x) = (x2 – x + 1) · Q(x) + x + 2


11. P(x) = 2x3 + ax2 + bx + 3
eşitliğini sağlayan Q(x) polinomunun sabit terimi kaçtır?
polinomunun x2 + 2 ile bölümünden kalan x – 1 olduğuna
A) 2 B) 3 C) 4 D) 5 E) 6
göre, a + b toplamı kaçtır?
x ≠ –1 olmak üzere,
A) –4 B) –2 C) 1 D) 5 E) 7 (x + 1) . P(x) = (x3 + 1) . Q(x) + (x + 1) (x + 2)
P(x) = (x2 + 2) . Q(x) + x – 1 x3 = –1 için (x + 1) . P(x) = (x + 1) . (x + 2)
x2 + 2 = 0 ⇒ x2 = –2 yazılır. P(x) = x + 2 olur.
2(–2) x + a (–2) +bx +3 = x – 1 (–1)3 – (–1)2 + ax + b = x + 2 ⇒ ax + b –2 = x + 2
(b – 4) x –2a + 3 = x – 1 ⇒ a = 1 ve b = 4
b – 4 = 1 ⇒ b = 5 ve P(0) = Q(0) + 2 ⇒ 4 = Q(0) + 2
–2a + 3 = –1 ⇒ a = 2 bulunur. ⇒ Q(0) = 2 bulunur.
a + b = 2 + 5= 7 dir. (Cevap E) (Cevap A)
14

12. Katsayıları toplamı 4 olan bir P(x) polinomu veriliyor.


16. Üçüncü dereceden bir P(x) polinomu x – 2, x + 1 ve x + 3 ile
(x – 2) · P(x) = x3 – 4x2 + ax + b
ayrı ayrı bölündüğünde 7 kalanını vermektedir.
olduğuna göre, a kaçtır?
P(x) polinomunun x – 1 ile bölümünden kalan 15
A) –7 B) –6 C) –4 D) 3 E) 9 olduğuna göre, x + 2 ile bölümünden kalan kaçtır?

x = 2 için , 8 – 16 + 2a + b = 0 ⇒ 2a + b = 8 A) 1 B) 2 C) 3 D) 4 E) 5
x = 1 için , –P(1) = 1 – 4 + a + b ⇒ –4 = –3 + a + b P(x) = a(x – 2) (x + 1) (x + 3) +7 biçimindedir.

⇒ a + b = –1 P(1) = 15 ⇒ a(–1) (2) (4) +7 = 15

2a + b = 8 ⇒ a = –1 bulunur.
⇒ a = 9 bulunur.
a + b = –1 (Cevap E) P(x) = –(x – 2) (x + 1) (x + 3) +7 dir.
P(–2) = –(–4) (–1) (1) +7 = 3 elde edilir. (Cevap C)
Çarpanlara Ayırma BÖLÜM 01 Test 05
1. a2 – b2 + 4b – 4 5. x3 + 2x2 – 5x – 6 = 0
ifadesinin çarpanlarından biri, aşağıdakilerden denkleminin çözüm kümesi aşağıdakilerden hangisidir?
hangisidir?
A) {–3, –1, 2} B) {–3, 1, 2} C) {–2, –1, 3}
A) a – 2b B) a + 2b C) a + b – 2
D) {–1, 2, 3} E) {–2, 1, 3}
D) a – b – 2 E) a + b + 2 x = –1 için –1 + 2 + 5 – 6 = 0 olduğundan x = –1 denklemin bir kökü ve x + 1 , x3 + 2x2 –5x – 6

a2 – (b2 – 4b + 4) = a2 – (b – 2)2 polinomunun bir çarpanımdır.

= [a – (b – 2)] [a + (b – 2)] x3 + 2x2 – 5x – 6 x+1

= (a – b + 2) (a + b – 2) x3 + x2 x2 + x – 6
(Cevap C) x2 – 5x – 6
x2 + x
–6x – 6

–6x – 6

0
x2 + x – 6 = 0
(x + 3) (x–2) = 0
x = –3 x = 2
Ç = {–3, –1, 1} (Cevap A)
2. a2 – 4b = 40

a3 – 4ab = 320
6. a + b + c = 11
olduğuna göre, a + b toplamı kaçtır?
ab + ac + bc = 36
A) 10 B) 12 C) 14 D) 16 E) 18
olduğuna göre, a2 + b2 + c2 işleminin sonucu kaçtır?
a3 – 4ab = 320 ⇒ a(a2 – 4b) = 320 A) 42 B) 49 C) 51 D) 57 E) 64
⇒ a . 40 = 320
⇒ a = 8 bulunur. (a + b + c)2 = 112 ⇒ a2 + b2 + c2 + 2 (ab + ac + bc) = 121
Çözüm Yayınları

a2 – 4b = 40 ⇒ 64 – 4b = 40 36
⇒ b = 6 olur. ⇒ a2 + b2 + c2 = 49
a + b = 8 + 6 = 14 elde edilir. (Cevap B)
(Cevap C)

1 4
3. a – =4 7. x2 + = 32
a x2
1 2
olduğuna göre, a3 – ifadesinin değeri kaçtır? olduğuna göre, x + ifadesinin değeri kaçtır?
a3 x
A) 52 B) 64 C) 76 D) 80 E) 84 A) 4 2 B) 6 C) 2 10 D) 4 3 E) 7

3 3
1 1 1 1 2 2 4
a3 – = a– + 3a . a– x+ = x2 + 2 . x . + 2
a3 a a a x x x
4
= 64 + 3 . 4 = x2 + +4
x2
= 76 = 36
(Cevap C) 2
x+ = 6 bulunur.
x
(Cevap B)
15

8. x2 – 4x – 1 = 0
4. a + b = 8
1
olduğuna göre, x3 – ifadesinin değeri kaçtır?
a+c=5 x3

olduğuna göre, ab + b2 – ac – bc ifadesinin değeri kaçtır? A) 60 B) 72 C) 76 D) 80 E) 84


1
A) 16 B) 20 C) 24 D) 32 E) 40 x2 – 1 = 4x ⇒ x – = 4 olur.
x
b(a + b) – c(a + b) = (a + b) (b – c) 3
1 1 1 1
=8.3 x3 – = x– +3.x. x–
x3 x x x
= 24
= 43 + 3 . 1 . 4
(Cevap C)
= 76 bulunur. (Cevap C)
Test 05 1. C 2. C 3. C 4. C 5. A 6. B 7. B 8. C 9. D 10. E 11. A 12. C 13. B 14. B 15. E 16. B

9. a – b = 3 13. x2 + 2x = 7
b+c=7 olduğuna göre, x4 + 4x3 – 8x + 3 ifadesinin değeri kaçtır?

olduğuna göre, a2 – ab + ac – bc ifadesinin değeri kaçtır? A) 21 B) 24 C) 28 D) 30 E) 35


(x2 + 2x)2 = 49 ⇒ x4 + 4x3 + 4x2 = 49
A) 18 B) 21 C) 28 D) 30 E) 35
⇒ x4 + 4x3 = 49 – 4x2
a–b=3
⇒ a + b = 10 ⇒ x4 + 4x3 = 49 – 4 (7 – 2x)
b+c=7
⇒ x4 + 4x3 – 8x + 3 = 49 – 28 + 8x – 8x + 3
a(a – b) + c (a – b) = (a – b) (a + c)
= 24 (Cevap B)
= 3 . 10
= 30 (Cevap D)

10. (x2 – 3x)2 – 2(x2 – 3x) – 8


ifadesi veriliyor. 14. x2 – x + 1 = 0

Aşağıdakilerden hangisi bu ifadenin çarpanlarından biri olduğuna göre, x4 – 2x3 + x + 3 ifadesinin değeri kaçtır?
değildir?
A) 3 B) 5 C) 7 D) 9 E) 11
A) x – 4 B) x – 2 C) x – 1 x2 = x – 1
(x – 1)2 – 2x (x – 1) + x + 3 = x2 – 2x + 1 – 2x2 + 2x + x + 3
D) x + 1 E) x + 2
= –x2 + x + 4
x2 – 3x = t olsun.
= – (x – 1) + x + 4
t2 – 2t – 8 = (t – 4) (t + 2)
= –x + 1 + x + 4
= (x2 – 3x – 4) (x2 – 3x + 2)
= 5 (Cevap B)
Çözüm Yayınları

= (x – 4) (x + 1) (x – 2) (x – 1) (Cevap E)

11. x2 – y2 = 4xy
2 y2
olduğuna göre, x + ifadesinin değeri kaçtır?
y 2 x2
A) 18 B) 16 C) 14 D) 12 E) 8
15. 310 + 4 · 65 + 2x
x2 y2
x2 – y2 = 4xy ⇒ – =4 ifadesi bir tam kare olduğuna göre, x tam sayısı kaçtır?
xy xy
x y
⇒ – =4 A) 4 B) 6 C) 8 D) 10 E) 12
y x
(35)2 + 2 . 2 . 35 . 5 + 2x = (35)2 + 2 . 35 . 6 + (26)2
2 2
x2 y2 x y 2 x y
2 + 2 = – +2. . = (35 + 26)2
y x y x y x
2x = (26)2 = 212 ⇒ x = 12 (Cevap E)
= 16 + 2 = 18 bulunur.
(Cevap A)

16 12. a > 0 olmak üzere,


1
a2 + = 14
a2
1
olduğuna göre, a3 + ifadesinin değeri kaçtır?
a3
16. a+b+c = 6
A) 42 B) 48 C) 52 D) 64 E) 76
1 + 1 + 1 = 13
1 2 1 1 1 2
a+ – 2a . = a2 + 2 ⇒ a + = 16 a b c a ⋅b ⋅c
a a a a
1
olduğuna göre, a2 + b2 + c2 ifadesinin değeri kaçtır?
⇒a+ =4
a
A) 9 B) 10 C) 12 D) 15 E) 18
1 1 3 1 1
a3 + 3 = a+ – 3a . a+ bc + ac + ab 13
a a a a = ⇒ bc + ac + ab = 13
a.b.c a.b.c
= 64 – 3 . 4 (a + b + c)2 = a2 + b2 + c2 + 2(ab + ac + bc)
= 52 (Cevap C) 36 = a2 + b2 + c2 + 2 . 13
a2 + b2 + c2 = 10 bulunur. (Cevap B)
Çarpanlara Ayırma BÖLÜM 01 Test 06
1. a · b · c = 24 olduğuna göre, a+b =4 3
5.
b a
a − 4  ⋅ b − 6  ⋅ c − 8 
      2 2
 bc   ac   ab  olduğuna göre, a + b + 1 ifadesinin değeri kaçtır?
2
b a2
işleminin sonucu kaçtır?
A) 45 B) 47 C) 51 D) 53 E) 55
A) 10 B) 15 C) 18 D) 20 E) 30
a b 2
2 a2 a b b2
abc – 4 abc – 6 abc – 8 20 . 18 . 16 + = 4 3 ⇒ +2. . + = 48
. . = b a b2 b a a2
bc ac ab (a . b . c)2
a2 b2
20 . 18 . 16 ⇒ + = 46
= b2 a2
24 . 24
2 2
a b
=10 (Cevap A) ⇒ + + 1 = 47 (Cevap B)
b2 a2

6. a – b + c = 6
2. x4 + 5x2 +9 a2 + b2 + c2 = 50
ifadesinin çarpanlarından biri aşağıdakilerden hangisidir? olduğuna göre, ab – ac + bc ifadesinin değeri kaçtır?
2 2 2
A) x – 3 B) x + 3 C) x + 1 A) 7 B) 9 C) 12 D) 14 E) 21
2 2
D) x – x + 3 E) x + 2x + 3 (a – b + c)2 = 62 ⇒ a2 + b2 + c2 – 2ab + 2ac – 2bc = 36
x4 + 6x2 + 9 –x2 = (x2 + 3)2 – x2 ⇒ 50 – 2 (ab – ac + bc) = 36
= (x2 + 3 – x) (x2 + 3 + x) (Cevap D)
⇒ ab – ac + bc = 7 bulunur. (Cevap A)
Çözüm Yayınları

7. a – b = 2

3. a = 3 − 2 a3 – b3 = 26

olduğuna göre, a · b çarpımı kaçtır?


b =3+ 2
A) 3 B) 4 C) 5 D) 6 E) 7
olduğuna göre, a3 + b3 ifadesinin değeri kaçtır?
a3 – b3 = (a – b)3 + 3ab (a – b)
A) 48 B) 60 C) 72 D) 90 E) 120
26 = 23 + 3ab (2)
a + b = 6 ve a . b = 7 dir.
a . b = 3 bulunur. (Cevap A)
a3 + b3 = (a + b)3 – 3ab(a + b)

= 216 – 3 . 7 . 6

= 90
(Cevap D)
17

8. x2 – 6x + 1 = 0
1
olduğuna göre, x3 + ifadesinin değeri kaçtır?
x3
4. x3 – 3x2 – 4x + 12 = 0
A) 176 B) 180 C) 194 D) 198 E) 208
denkleminin çözüm kümesi aşağıdakilerden hangisidir?
x2 – 6x + 1 = 0 ⇒ x2 + 1 = 6x
A) {–3, –2, 2} B) {–2, 2, 3} C) {–2, 1, 6} 1
⇒x+ =6
x
D) {–1, 3, 4} E) {1, 3, 4} 1 1 3 1 1
3
x + = x+ –3·x· x+
x2(x–3)–4(x–3)=0 ⇒(x–3) (x2 – 4) = 0 x x x
a3
⇒ x = 3 , x = –2 , x = 2 = 216 – 3 · 6
(Cevap B)
= 198 (Cevap D)
Test 06 1. A 2. D 3. D 4. B 5. B 6. A 7. A 8. D 9. C 10. B 11. C 12. A 13. C 14. A 15. B 16. C

9. x3 + y3 = x + y = 2 13. a2 + b2 – 2ab – 6a + 6b + 9 = 0
olduğuna göre, x · y çarpımı kaçtır? olduğuna göre, a – b farkı kaçtır?

A) 5 B) 4 C) 1 D) –1 E) –2 A) 1 B) 2 C) 3 D) 6 E) 9
x3 + y3 = (x + y)3 –3 xy (x + y) (a – b)2 – 6 (a – b) + 9 = 0
2 = 8 – 3 xy · 2 ⇒ x · y = 1 bulunur. (Cevap C) (a – b – 3)2 = 0 ⇒ a – b = 3 (Cevap C)

10. x2 – 3x – 5 = 0 14. x + y = z

25 x · y · z = 12
olduğuna göre, x2 + ifadesinin değeri kaçtır?
x2
olduğuna göre, x3 + y3 – z3 ifadesinin değeri kaçtır?
A) 17 B) 19 C) 20 D) 25 E) 36
A) –36 B) –27 C) –18 D) –9 E) –6
5
x2 – 5 = 3x ⇒ x – =3
x (x + y)3 = z3 ⇒ x3 + 3xy (x + y) + y3 = z3
5 2 ⇒ x3 + y3 – z3 = –3xy (x + y)
⇒ x– =9
x
z
25
⇒ x2 – 10 + =9 =x3 + y3 – z3 = –3xyz
x2
25 = –3 · 12
Çözüm Yayınları

⇒ x2 + = 19 bulunur. (Cevap B) = –36 (Cevap A)


x2

1
11. x – =3 15. a pozitif tam sayı olmak üzere,
x
1 a3 + 2a2 + 11a + 10
olduğuna göre, x4 + ifadesinin değeri kaçtır?
x4
ifadesi bir tam sayının küpüdür.
A) 109 B) 116 C) 119 D) 121 E) 123
Buna göre, a kaçtır?
1 2 1 1 1
x– = 9 ⇒ x2 – 2 · x · + = 9 ⇒ x2 + = 11 A) 8 B) 9 C) 10 D) 11 E) 12
x x x2 x2
3 2 2
1 1 2 1 a + 3a + 3a + 1 –a + 2a + 9
⇒ x4 + = x2 + – 2 · x2 ·
x4 x2 x2 (a + 1)3 – (a2 –8a –9) = (a + 1)3 – (a + 1) (a – 9)

= 121 – 2 = (a + 1)[(a + 1)2 – (a – 9)]

= 119 (Cevap C) = (a + 1) (a2 + a + 10)


2 2
a + 2a + 1 = a + a + 10 ⇒ a = 9 (Cevap B)

18

12. x + 1 = a
16. x – 3 = 10
x2 – x + 1 = b x

olduğuna göre, x3 – ab – 19 işleminin sonucu kaçtır? olduğuna göre, x – 3 x ifadesinin değeri kaçtır?

A) –20 B) –19 C) –18 D) 18 E) 19 A) –3 B) –1 C) 1 D) 2 E) 3


(x + 1) (x2 – x + 1) = ab ⇒ x3 + 1 = ab x x – 3 = 10 x ⇒ x x – 9 x – x – 3 = 0
⇒ x3 – ab + 1 = 0 ⇒ x (x – 9) – ( x + 3) = 0
⇒ x3 – ab – 19 = –20 ⇒ x ( x – 3) ( x + 3) – ( x + 3) = 0
(Cevap A)
⇒ ( x + 3) (x – 3 x – 1) = 0
⇒ x – 3 x = 1 bulunur. (Cevap C)
Rasyonel İfadeler BÖLÜM 01 Test 07
1. 2ab − 2ax + xy − by
2 2
5. a − b − 2a + 2b = 2
x −b 2 2 a −b 3
ifadesinin sadeleştirilmiş biçimi aşağıdakilerden olduğuna göre, a + b toplamı kaçtır?
hangisidir?
A) 4 B) 6 C) 8 D) 9 E) 10
A) a B) b C) y – 2x
(a – b) [a + b – 2] 2
=
D) y – 2a E) y – 2b (a – b) (a + b) 3
2a(b – x) –y (b – x) (b – x) (2a – y) 3(a + b) – 6 = 2(a + b)
=
x–b x–b
(a + b) = 6 (Cevap B)
= y – 2a (Cevap D)

6. a 3 + b 3 : a 2 − ab + b 2
x 3 − 9 x : x 2 − x − 12 a 2 − b 2 a 2 − 2ab + b 2
2.
x 2 + 2x − 15 x 2 + x − 20 ifadesinin sadeleştirilmiş biçimi aşağıdakilerden
ifadesinin sadeleştirilmiş biçimi aşağıdakilerden hangisidir?
hangisidir? A) a – b B) a2 – b2 C) a + b
x x
A) x – 3 B) C) a+b a−b
x+3 x−4 D) E)
a−b a+b
x+3
D) x E) (a + b) (a2 – ab + b2) (a – b)2
x−4 = = a – b
(a – b) (a + b) a2 – ab + b2
(x – 4) (x + 5) (Cevap A)
x(x – 3) (x + 3)
= = x (Cevap D)
(x – 3) (x + 5) (x – 4) (x + 3)
Çözüm Yayınları

3. x 2 + 2x + m x − 12 x− 1
x2 + x − 6 7. x : x
x2 + x + 1 x + 1
x+4
ifadesinin sadeleştirilmiş biçimi olduğuna göre, m
x+3 ifadesinin sadeleştirilmiş biçimi aşağıdakilerden
kaçtır?
hangisidir?
A) –10 B) –9 C) –8 D) 6 E) 8 1
A) x – 1 B) x + 1 C)
x
x2 + 2x + m x+4 x2 + 2x + m x2 + 2x – 8
= ⇒ = x −1 x +1
(x + 3) (x – 2) (x + 3) (x + 3) (x – 2) (x + 3) (x – 2) D) E)
(x – 2) x x
m = –8 dir. (Cevap C) x3 – 1 x3 – 1
x2 x (x – 1) (x2 + x + 1) . (x + 1) x 1
: = =
2
x +x+1 x+1 x2 (x2 + x + 1) (x – 1) (x + 1) x
(Cevap C)

19

3 2
4. a + 8 : a − 2a + 4
a2 − 4 a 3 − 2a 2 8. ( x 2 + x − 2)( x 2 − 2x − 3)
=0
x2 − 1
ifadesinin sadeleştirilmiş biçimi aşağıdakilerden
hangisidir? denkleminin çözüm kümesi aşağıdakilerden hangisidir?

A) a – 2 B) a + 2 C) a – 4 A) {–2, 3} B) {–2, 1} C) {–1, 3}

D) a2 E) a + 4 D) {–2, –1, 3} E) {–2, 1, 3}


(a + 2) (a2 – 2a + 4) 2
a (a – 2) (x + 2) (x – 1) (x + 1)(x – 3)
= = a2 (Cevap D) = 0 ⇒ x = –2 , x = 1 , x = –1 , x = 3
(a – 2) (x + 2) a2 – 2a + 4 (x – 1) (x + 1)
x = 1 ve x = – 1 paydası sıfır yaptığından alınmaz. Çözüm kümesi {–2, 3} tür.
(Cevap A)
Test 07 1. D 2. D 3. C 4. D 5. B 6. A 7. C 8. A 9. D 10. E 11. E 12. E 13. C 14. C 15. C 16. B

2x − 7 13. x2 + x + 1 = 0
9. = A + B
x 2 − 7 x + 12 x − 4 x − 3 10
olduğuna göre, x − 1 ifadesi aşağıdakilerden hangisine
olduğuna göre, A + B toplamı kaçtır? x −1
eşittir?
A) –2 B) –1 C) 1 D) 2 E) 3
A) –1 B) –x C) 1 D) x E) x + 1
2x –7 = (x – 3) A + (x – 4) B
x2 + x + 1 = 0 ⇒ (x – 1) (x2 + x + 1) = 0, x ≠ 1
x = 4 için 1 = A ⇒ A = 1 ,
⇒ x3 – 1 = 0
x = 3 için –1 = –B ⇒ B = 1 buluruz.
⇒ x3 = 1
A + B = 2 dir. (Cevap D)
x10 – 1 (x3)3 . x – 1 x–1
= = =1
x–1 x–1 x–1
(Cevap C)

14. a = a +4

10. x3 − 1 x − 1 olduğuna göre,


:
x 2 − 1 1− 1
x x a a +1
işleminin sonucu aşağıdakilerden hangisidir? a + 1

A) x2 B) x C) –x D) –1 E) 1 ifadesinin değeri kaçtır?

1 A) 3 B) 4 C) 5 D) 6 E) 7
x–
x3 – 1 x x3 – 1 x x–1
. . . . =1 a = t olsun. t2 – t = 4 olur.
x3 – 1 x–1 1 x3 – 1 x(x – 1)
t3 + 1
x (Cevap E) = t2 – t + 1
t+1
=4+1
=5
Çözüm Yayınları

(Cevap C)

15. a, b ve c birer pozitif tam sayı olmak üzere,


11. (a −2 − b −2 ) ab
a + 2b − 3 a2 – b = 11
b a
b – c2 = 6
ifadesinin sadeleştirilmiş biçimi aşağıdakilerden
hangisidir? olduğuna göre,

a+b a+b a+b a 2 − b 2 + ac − bc


A) B) C) a−b
a b a−b
a−b a+b ifadesinin değeri kaçtır?
D) E)
a+b 2b − a
A) 80 B) 84 C) 87 D) 90 E) 95
1 1 b2 – a 2
– ab . ab (a – b) (a+ b) + c (a – b)
a2 b2 a2 b2 =a+b+c
= a–b
2 2 (a – b) (a – 2b)
a + 2b – 3ab
a2 – c2 = 17 ⇒ (a – c) (a + c) = 17
ab ab
⇒a–c=1
(b–a) (b + a) ab a+b ⇒ a = 9 ve b = 8
= . = a + c = 17
20 ab (a – b) (a – 2b) 2b – a (Cevap E) a2 – b = 11 ⇒ 81 – 11 = b
⇒ b = 70 , a + b + c = 9 + 70 + 8 = 87
(Cevap C)

12. ( x 2 − xy )2 abx − aby 16. a 9 + a 8 + a 7 + ... + a 2 + a



ay − ax byx 2 − bx 3 a −9 + a −8 + a −7 + ... + a −2 + a −1
ifadesinin sadeleştirilmiş biçimi aşağıdakilerden ifadesinin sadeleştirilmiş biçimi aşağıdakilerden
hangisidir? hangisidir?

A) x + y B) y – x C) a(x + y) A) a10 – 1 B) a10 C) a10 + 1

D) b(x + y) E) x – y D) a18 + 1 E) a18


[x(x – y)]2 ab (x – y) x2 (x – y)2 ab (x – y) a(a8 + a7 + a7 + ... + 1)
. = . =x–y = a10
a(y – x) bx2 (y – x) a(y – x) bx2(y – x) a–9 (1 + a + a2 + ... a8 )
(Cevap E) (Cevap B)
Rasyonel İfadeler BÖLÜM 01 Test 08
3 2
5. 5x + 2 = A + B
1. x − 9 x : 2x − 5 x − 3
x 2 + 3x 2x 2 − x − 1 x 2 + 2x − 8 x − 2 x + 4
ifadesinin sadeleştirilmiş biçimi aşağıdakilerden olduğuna göre, A + B toplamı kaçtır?
hangisidir? A) –3 B) –2 C) 3 D) 4 E) 5
2
A) x – 3 B) x – 1 C) x – x 5x + 2 = (x + 4)A + (x – 2)B

D) x + 3 E) x + 1 x = 2 için 12 = 6A ⇒ A = 2 ve

x (x – 3) (x + 3) (2x + 1) (x – 1) x = –4 için –18 = –6B ⇒ B = 3 bulunur.


. =x–1
x (x + 3) (x – 3) (2x + 1) A + B = 5 tir.
(Cevap B)
(Cevap E)

a 3 − b 3 : a 2 + ab + b 2
6.
a + 2b a 2 + ab − 2b 2
x6 − 1 :  x + 1 + 1 
2.
x3 − x2 + x  x ifadesinin sadeleştirilmiş biçimi aşağıdakilerden
hangisidir?
ifadesinin sadeleştirilmiş biçimi aşağıdakilerden
hangisidir? A) a – b B) a + b C) (a – b)2

A) x – 1 B) x + 1 C) x2 – x D) (a + b)2 E) a2 – b2

D) x2 + x E) x2 – 1 (a – b) (a2 + ab + b2) (a + 2b) (a – b)


. = (a – b)2
a + 2b a2 + ab + b2
(x3 – 1) (x3 + 1) x (Cevap C)
. = x2 – 1
x (x2 – x + 1 x2 + x + 1
(Cevap E)
Çözüm Yayınları

 
  2
3 (a + 1)2 + 3 7.  a 2 − a  : a
3. a − 82 : 2 1
4−a a −a−6  1+  a +1
 a
ifadesinin sadeleştirilmiş biçimi aşağıdakilerden
ifadesinin sadeleştirilmiş biçimi aşağıdakilerden
hangisidir?
hangisidir?
A) a + 1 B) a + 2 C) a + 3
1
A) B) a C) a2 D) a – 1 E) a + 1
D) 3 – a E) 2 – a a
a a+1 a2 a+1
(a – 2) (a2 + 2a + 4) (a – 3) (a + 2) a2 – . = a2 – .
. =3–a a+1 a2 a+1 a2
(2 – a) (2 + a) a2 + 2a + 4 a
(Cevap D) a2 (a + 1 –1) a+1
= .
a+1 a2
= a (Cevap B)
21

2
4. x + 6 x + a 1+ a
x 2 + bx − 12 8. a +1 = 9
x+2 1− a
ifadesinin sadeleştirilmiş biçimi olduğuna göre, a +1
x −3
a + b toplamı kaçtır?
olduğuna göre, a kaçtır?

A) 8 B) 9 C) 10 D) 11 E) 12 A) 3 B) 4 C) 6 D) 8 E) 9
x2 + 6x + a x+2 x2 + 6x + 8 a+1+a
= =
x2 + bx – 12 x–3 x2 + x – 12 a+1
(x + 4) = 9 ⇒ 2a + 1 = 9
a + 1 –a
a = 8 ve b = 1 dir. a + b = 9 bulunur. (Cevap B) ⇒ 2a = 8
a+1
⇒ a = 4 (Cevap B)
Test 08 1. B 2. E 3. D 4. B 5. E 6. C 7. B 8. B 9. B 10. A 11. A 12. C 13. C 14. A 15. B 16. A

9. a + b = 6 olduğuna göre, a 4 + a2 + 1 : 1+ 1 + 1 


13.  
a 2 − 6a + 9 − b 2 a3 − a2 + a  a a2 
2 2
− b − 3a − 3b
a ifadesinin sadeleştirilmiş biçimi aşağıdakilerden
hangisidir?
ifadesinin değeri kaçtır?
1 1
1 1 3 4 5 A) B) C) a D) a2 E) a + 1
A) B) C) D) E) a2 a
3 2 4 5 6
a4 + 2a2 + 1 – a2 a2 + a + 1
(a – 3)2 – b2 (a – 3 – b) (a – 3 + b) :
= a(a2 – a + 1) a2
(a – b) (a + b) –3 (a + b) (a + b) (a – b – 3)
a+b–3 (a2 + 1)2 – a2 a2
= = :
a+b a(a2 – a + 1) a2 + a + 1

6–3 1 (a2 + 1 – a) (a2 + 1 + a) a2


== = bulunur. (Cevap B) = : = a (Cevap C)
6 2 a(a2 – a + 1) a2 + a + 1

14. a ≠ 2 olmak üzere,

10. a 2 − 1 : a 2 − 4a + 3 a 2 + 12 = 10
a2 + 3a + 9 a 3 − 27 a

işleminin sonucu aşağıdakilerden hangisidir? olduğuna göre, a2 + 2a ifadesinin değeri kaçtır?

A) a + 1 B) a – 1 C) a + 2 A) 6 B) 8 C) 10 D) 12 E) 14

2
D) a + a + 3 E) a2 –a+3 a3 – 10a + 12 = 0 ⇒ a3 – 8 – 10a + 20 = 0

⇒ (a – 2) (a2 + 2a + 4) –10 (a – 2)
(a – 1) (a + 1) (a – 3) (a2 + 3a + 9)
= =a+1
a2 + 3a + 9 (a – 1) (a – 3) ⇒ (a – 2) (a2 + 2a – 6) = 0
(Cevap A)
⇒ a2 + 2a = 6 (a≠2) (Cevap A)
Çözüm Yayınları

15. x7 − x
(x2 − 1)[( x 2 + 1)2 − x 2 ]
ifadesinin sadeleştirilmiş biçimi aşağıdakilerden
a + a hangisidir?
11.
1+ x y 1+ x −y
A) x – 1 B) x C) x + 1
ifadesinin sadeleştirilmiş biçimi aşağıdakilerden
x x
hangisidir? D) E)
x −1 x +1
A) a B) x C) y D) xy E) x–y x(x6 – 1) x(x3 – 1) (x3 + 1)
= =x
(x2 2
– 1) (x + 1 – x) (x + 1 + x)2 (x3 – 1) (x3 + 1)
a a a axy (Cevap B)
+ = +
1 + xy 1+
1 1 + xy xy + 1
xy
a(1 + xy)
=
1 + xy
=a (Cevap A)

22
16. x ≠ y olmak üzere,

(a2x + y + a x +2y )  aa y + aa x − 1


x y

3
12. x − 6 x − 2 : 6 x +1 a3x + a3y
3 6
x −4 x +2
ifadesinin sadeleştirilmiş biçimi aşağıdakilerden
işleminin sonucu kaçtır?
hangisidir?
6 3
A) x B) x C) 1 A) 1 B) ax–y C) ax+y

D) 6
x + 1 E) 3
x +1 D) ax – ay E) ax + ay
6
x = t olsun. ax ay
(ax + ay) . aa + y . + –1
t2 – t – 2 t+1 (t – 2) (t + 1) t + 2 ay ax (ax + ay) (a2x + a2y – ax + y)
: = . = 1 bulunur. = =1
t2 – 4 t+2 (t – 2) (t + 2) t + 1) 3x 3y (ax + ay) (a2x – ax . ay + a2y)
a +a
(Cevap C) (Cevap A)
Rasyonel İfadeler BÖLÜM 01 Test 09
2 5. a – b = 10 olduğuna göre,
1. a − 9 :  1 − 2 
2 1− a  a − 1 a + 1 a 2 − b 2 + 4b − 4
2 2
işleminin sonucu aşağıdakilerden hangisidir? − b − 2a + 2b
a

A) a – 3 B) a – 1 C) a + 1 ifadesinin değeri kaçtır?


D) a + 3 2
E) a – 1 5 3 6
A) 3 B) 2 C) D) E)
3 2 5
(a – 3) (a + 3) a + 1 – 2a + 2
: a2 – (b2 – 4b + 4) a2 – (b – 2)2
(1 – a) (1 + a) (a – 1) (a + 1) =
(a – b) (a + b) –2 (a – b) (a – b) (a + b – 2)
(a – 3) (a + 3) (a – 1) (a + 1)
= . [a – (b – 2)] [a + (b – 2)]
(1 – a) (1 + a) –a + 3 =
(a – b) (a + b – 2)
= a + 3 (Cevap D) a–b+2
=
a–b

10 + 2 6
= = (Cevap E)
10 5

x 3 − 2x 2 − x + 2
2. x 3 − 3x 2 + 3x − 1 − x 2 − x − 6
x2 − x − 2 6.
x 2 − 2x + 1 x+2
ifadesinin sadeleştirilmiş biçimi aşağıdakilerden
işleminin sonucu aşağıdakilerden hangisidir?
hangisidir?
A) 2 B) 4 C) x – 3
A) x + 2 B) x + 1 C) x – 1
D) x –1 E) x + 1
D) x – 2 E) –x + 2
(x – 1)3 (x + 2) (x – 3)
x2(x – 2) – (x – 2) (x – 2) (x2 – 1) = = x – 1 – (x – 3)
= (x – 1)2 x+2
(x – 2) (x + 1) (x – 2) (x + 1)
= 2 (Cevap A)
(x – 1) (x + 1)
Çözüm Yayınları

=
x+1
= x – 1 (Cevap C)

3. x4 −1 :  x + 1 
 
x3 + x2  x
2
ifadesinin sadeleştirilmiş biçimi aşağıdakilerden 7. x − 1 + x+2 +x − x +1+ 3 = 2
2 x −1 x2 + 3x + 2 x3 + 1 x +1
hangisidir?
olduğuna göre, x kaçtır?
1 x −1 x +1
A) B) C)
x x x A) 1 B) 2 C) 3 D) 4 E) 5
D) x – 1 E) x + 1 1 1 1 3 6
+ + + =2⇒ =2
(x2 – 1) (x2 + 1) x (x – 1) (x + 1) x–1 x+1 x+1 x+1 x+1 x+1
. = =
x2 (x + 1) x2 + 1 x (x + 1) x ⇒ 2x + 2 = 6
(Cevap B) ⇒ x = 2 (Cevap B)

23

(a − b)3 − 3(b − a)3  


4. 8.  1 −  a +1
:
1
(a − b)2 + (b − a)2
 1− 1  a − 1
ifadesinin sadeleştirilmiş biçimi aşağıdakilerden  a  a
hangisidir? ifadesinin sadeleştirilmiş biçimi aşağıdakilerden
A) a – b B) 2(a – b) C) b – a hangisidir?

D) 2(b – a) E) a + b 1 1 a −1
A) − B) C) a – 1 D) a + 1 E)
a a a
(a – b)3 + 3(a – b)3 4(a – b)3 1 a a(a + 1)
= = 2(a – b) a+1
(a – b)2 + (a – b)2 2(a – b)2 1–
a–1
: = 1–
a–1
:
a2 – 1
a2 – 1
(Cevap B)
a a
a–1–a (a – 1) (a + 1) 1
= . =– (Cevap A)
a–1 a(a + 1) a
Test 09 1. D 2. C 3. B 4. B 5. E 6. A 7. B 8. A 9. B 10. A 11. A 12. A 13. A 14. D 15. C 16. B

2 13. a, b, c, d tam sayılar ve a < b < c < d olmak üzere,


a − a −1
9. a : 3a (x – a) (x – b) (x – c) (x – d) = 9
a− a − 1 a +1
a denkleminin bir kökü 4 tür.
ifadesinin sadeleştirilmiş biçimi aşağıdakilerden Buna göre, a + b + c + d toplamı kaçtır?
hangisidir?
A) 16 B) 17 C) 18 D) 19 E) 20
a −1 a +1
A) B) C) a – 1 9 un çarpanları 3, 1 –1, –3 tür.
a a
4–a = 3 ⇒ a = 1,
1
D) a + 1 E) 4 – b = 1 ⇒ b = 3,
a
a2 – a 2 + 1 4 – c = – 1 ⇒ c = 5,
a (a + 1) (a2 – a + 1) 4 – d = –3 ⇒ d = 7 bulunur.
.
a2 – a + 1 a a + b + c + d = 1 + 3 + 5 + 7 = 16 dır. (Cevap A)
a

1 a (a + 1) (a2 – a + 1) a+1
= . . = (Cevap B)
a a2 – a + 1 a a

14. a 3 − 3a + 3 − 1 = 64
a a3
10. x 2 + 4 x + 4 : x 2 + 2x
olduğuna göre, a 2 +
1 ifadesinin değeri kaçtır?
x 2 + x − 2 x 2 − 2x + 1
a2
ifadesinin sadeleştirilmiş biçimi aşağıdakilerden
hangisidir? A) 12 B) 14 C) 16 D) 18 E) 20
1 3 1
x −1 x a– = 64 ⇒ a – =4
A) B) C) x a a
x x +1 1 2 1
a– = 42 ⇒ a2 –2 + = 16
x −1 a a2
D) x – 1 E) 1
x +1
Çözüm Yayınları

⇒ a2 + 2 = 18
a
(x + 2)2 (x – 1)2 x–1
. = (Cevap A) (Cevap D)
(x + 2) (x – 1) x (x + 2) x

15. (x + 1)2 = 4x + 10

a−b − a olduğuna göre,

11. a a+b 1
a−b + b ( x + 2)2 +
( x + 2) 2
b a+b

ifadesinin sadeleştirilmiş biçimi aşağıdakilerden ifadesinin değeri kaçtır?


hangisidir? A) 34 B) 36 C) 38 D) 40 E) 42
b3 b a a3 x2 + 2x + 1 = 4x + 10
A) − B) − C) − D) − E) ab
a3 a b b3 x2+ 4x + 4 = 6x + 13
a2 – b 2 – a 2 (x + 2)2 = 6(x + 2) + 1
a(a + b) – b2 b(a + b) 1
= . (x + 2) – =6
a2 – b 2 + b 2 a(a + b) a2 (x + 2)
b(a + b) 1
(x + 2)2 + = 38
b3 (x + 2)2
24 =– (Cevap A) (Cevap C)
a3

9x − 4 ⋅ 3x + 3 16.  x 4 + x 3 + x 2 + x + 1 + 1  : x
5
12.
3x − 3  2 x − 1 x − 1

ifadesinin sadeleştirilmiş biçimi aşağıdakilerden ifadesinin sadeleştirilmiş biçimi aşağıdakilerden


hangisidir? hangisidir?

A) 3x – 1 B) 3x C) 3x + 1 A) x – 1 B) x + 1 C) x2 – 1
x −1 x +1
D) 3x + 2 E) 3x + 3 D) E)
x x
t2 – 4t + 3 (t – 3) (t – 1) x5 – 1 + 1 x2 – 1
3x = t, = =t–1 . =x+1
t–3 t–3 x–1 x5
= 3x – 1 (Cevap A) (Cevap B)
BİRE BİR BÖLÜM 01 Test 10
1. P(x) üçüncü dereceden bir polinom olmak üzere, 5. P(x) bir polinom ve
P(–2) = P(1) = P(3) = 0 P(x – 1) + x2 · P(x + 1) = x3 + 4x2 + x + 2

P(2) = 8 P(2) = 5

olduğuna göre, P(–1) kaçtır? olduğuna göre, P(x) polinomunun sabit terimi kaçtır?

A) –20 B) –16 C) –12 D) 12 E) 16 A) 2 B) 3 C) 4 D) 5 E) 6


P(x) = a (x + 2) (x – 1) (x – 3) x = 1 için P(0) + 1 . P(2) = 1 + 4 + 1 + 2
P(2) = 8 ⇒ a(4) (1) (–1) = 8 P(0) + 5 = 8
⇒ a = –2 bulunur. P(0) = 3 bulunur.
P(x) = –2 (x + 2) (x – 1) (x – 3) ⇒ P(–1) = –16 (Cevap B)
(Cevap B)

6. Her x gerçek sayısı için


2. P(x) = x3 + 4x2 + ax + b
x2 + ax – 4 = (x – 1) (bx + c)
polinomu x2 – x – 2 ile tam bölünebildiğine göre, a + b
toplamı kaçtır? olduğuna göre, a + b + c toplamı kaçtır?

A) –17 B) –15 C) –3 D) 15 E) 17 A) 3 B) 5 C) 6 D) 8 E) 10
x = 1 için 1 + a – 4 = 0 ⇒ a = 3 elde edilir.
x2 – x – 2 = (x + 1) (x – 2)
x2 + 3x – 4 = (x – 1) (bx + c)
P(–1) = 0 ⇒ –1 + 4 – a + b = 0 ⇒ –a + b = –3
(x – 1) (x + 4) = (x – 1) (bx + c)
P(2) = 0 ⇒ 8 + 16 + 2a + b = 0 ⇒ 2a + b = –24
x + 4 = bx + c (x ≠ 1 için)
Çözüm Yayınları

3a = –21 b = 1 ve c = 4 bulunur.
a = –7 a + b + c + = 3 + 1 + 4 = 8 dir. (Cevap D)
b = – 10
a + b = –17 dir. (Cevap A)

3. a ve b birer pozitif tam sayı olmak üzere,


P(x) = (x + a) (x + b) 7. Baş katsayısı 2 olan ikinci dereceden bir P(x) polinomu için
polinomunun katsayılarının toplamı 21 olduğuna göre, P(1) – P(0) = 5
a + b toplamı kaçtır?
olduğuna göre, P(2) – P(1) işleminin sonucu kaçtır?
A) 4 B) 6 C) 8 D) 10 E) 12
A) 6 B) 8 C) 9 D) 10 E) 12
P(1) = 21 ⇒ (1 + a) (1 + b = 21)
P(x) = 2x2 + bx + c
1 + a = 3 ve 1 + b = 7
P(1) – P(0) = 5 ⇒ (2 + b + c) – c = 5
(a = 2 ve b = 6) veya (a = 6 ve b = 2)
⇒ b = 3 bulunur.
olabilir. a + b = 8 dir. (Cevap C)
P(2) – P(1) = (8 + 6 + c) – (2 + 3 + c) = 9 (Cevap C)

25

4. Baş katsayısı 1 olan üçüncü dereceden P(x) polinomu


x2 + 2 ile kalansız bölünmektedir.

P(2x) polinomunun 2x – 1 ile bölümünden elde edilen


kalan 12 olduğuna göre, P(2) kaçtır? 8. a – 2b = 4 olduğuna göre,
P(x) = (x2 + 2) (x + b) a2 + 4b2 – 4ab + a – 2b – 1
1
2x – 1 = ⇒ x =
2 ifadesinin değeri kaçtır?
1
x= için P(2x) P(1) = 12 3 0
2
3.(1 + b) = 12 A) 17 B) 18 C) 19 D) 20 E) 21
b=3 (a – 2b)2 + (a – 2b) – 1 = 16 + 4 – 1
P(2) = 6 . (2 + b) = 6 . 5 = 30 = 19 (Cevap C)
Test 10 1. B 2. A 3. C 4. (30) 5. B 6. D 7. C 8. C 9. B 10. C 11. D 12. (18) 13. (200) 14. C 15. B 16. A

9. a ve b birer gerçek sayı olmak üzere, 13. a – b = b – c = 10


a2 – 2b = 6 olduğuna göre, a2 + c2 – 2b2 ifadesinin değeri kaçtır?

b2 – 4a = –11 a2 – b2 + c2 – b2 = (a – b) (a + b) + (c – b) (c + b)
= 10 (a + b) – 10(b + c)
olduğuna göre, a + b toplamı kaçtır?
= 10a + 10b – 10b –10c 2 0 0
A) 2 B) 3 C) 4 D) 5 E) 6 = 10(a – c)
Eşitlikler taraf taraf toplanarak düzenlediğinde, Ayrıca;
a2 – 4a + 4 + b2 – 2b + 1 = 0 a – b = 10
⇒ a – c = 20 dir.
(a – 2)2 + (b – 1)2 = 0 b – c = 10
a = 2 , b = 1 ve a + b = 3 elde edilir. (Cevap B) 10(a – c) = 10 . 20 = 200 bulunur.

2 2 3
10. f ( x ) = (1 − x ) (1 + x + x + x )
2 3
1− x − x + x
14. x bir gerçek sayı olmak üzere,
olduğuna göre, f( 3) değeri kaçtır?
( 5 + 3 )x = 2
A) 2 B) 3 C) 4 D) 5 E) 6
f(x) =
(1 – x) (1 – x4)
=
(1 – x) (1 – x2) (1 + x2) olduğuna göre, ( 5 − 3) x ifadesi aşağıdakilerden
(1 – x) –x2(1 – x) (1 – x) (1 -x2) hangisine eşittir?
= 1 + x2 dir. A) 2–x B) 21–x C) 2x–1
f 3 = 1 + 3 = 4 bulunur. (Cevap C)
D) 2x+1 E) 4x

x
5 + 3 = 2
Çözüm Yayınları

x
5 – 3 = k

[( 5 + 3) ( 5 + 3)x = 2 k ⇒ 2x = 2k
11. a ve b pozitif tam sayılar, p bir asal sayı olmak üzere,
⇒ k = 2x–1 bulunur. (Cevap C)
a2 – b2 = p

olduğuna göre, a2 + b2 toplamının p türünden değeri


a − a −2
aşağıdakilerden hangisidir? 15.
1 + a −1 + a −2
p −1 p +1 p2 − 1
A) B) C) ifadesinin sadeleştirilmiş biçimi aşağıdakilerden
2 2 2
hangisidir?
p2 + 1
D) E) p2 + 1
2 A) 2a B) a – 1 C) a + 1
a–b=1 p+1 p–1 1 1
a+b=p
a= ,b= D) E)
2 2 a a −1
a2 + b2 = (a – b)2 + 2ab a(1 – a–3) a(1 – a–1) (1 + a–1 + a–2)
=
p+1 p–1 1 + a–1 + a–2 1 + a–1 + a–2
=1+2. .
2 2 = a(1 – a–1)
p – 1 p2 + 1
2
=1+ = (Cevap D) = a – 1 (Cevap B)
2 2

26

2
1 1
12.  x +  − 6  x +  + 9 = 0 a3 − b3
 x  x 16.
(a + b)2 − ab
1
olduğuna göre, x3 + ifadesinin değeri kaçtır?
x3 ifadesinin sadeleştirilmiş biçimi aşağıdakilerden
1 hangisidir?
x + = t olsun.
x
t2 – 6t + 9 = 0 ⇒ (t – 3)2 = 0 A) a – b B) a + b C) b – a
⇒ t = 3 bulunur. 1 8 2
D) a + ab E) ab + b2
1 1 3
x+ =3⇒ x+ = 27
x x
(a – b) (a2 + ab + b2)
1 1 3 1 1 = a – b
x3 + = x+ – 3x . . x+ a2 + 2ab + b2 – ab
x3 x x x (Cevap A)
= 27 – 3 . 3

= 18
BİRE BİR BÖLÜM 01 Test 11
1. P(x) bir polinom ve 5. Katsayılarının toplamı 4 olan bir P(x) polinomunun x + 2 ile
bölümünden kalan –5 tir.
x3 + ax + 8 = (x + 2) · P(x)
Buna göre, P(x) polinomunun x2 + x – 2 ile bölümünden
olduğuna göre, P(–2) kaçtır?
kalan aşağıdakilerden hangisidir?
x = –2 için –8 – 2a + 8 = 0 ⇒ a = 0 bulunur.
x3 + 8 = (x + 2) . P(x) A) x + 3 B) 2x + 2 C) 3x + 1
(x + 2) (x2 – 2x + 4) = (x + 2) . P(x)
1 2 D) 4x – 1 E) 5x – 2
P(x) = x2 – 2x + 4 ⇒ P(–2) = 4 + 4 + 4
⇒ P(–2) = 12 dir. P(1) = 4 , P(–2) = –5 tir.
P(x) = (x – 1) (x + 2) . Q(x) + ax + b
P(1) = 4 ⇒ a + b = 4
⇒ a = 3 ve b = 1
P(–2) = 5 ⇒ –2a + b = –5
Kalan: ax + b = 3x + 1 dir. (Cevap C)
2. P(x) ve Q(x) polinomları için
P(x + 2) = (x2 – x – 3) · Q(x) + x + 5

bağıntısı sağlanmaktadır. 6. P(x) ikinci dereceden bir polinom ve Q(x) = k sabit polinom
olmak üzere,
Q(x) in sabit terimi 4 olduğuna göre, P(x) polinomunun
x – 2 ile bölümünden kalan kaçtır? P(x) + Q(x) = 2x2 – x + 4
A) –7 B) –3 C) 1 D) 4 E) 6 P(Q(x)) = 5
x = 0 için, P(2) = (–3) . Q(0) + 5
olduğuna göre, k nin alabileceği değerlerin toplamı
P(2) = (–3) . 4 + 5
kaçtır?
P(2) = –7 bulunur.
(Cevap A) A) 1 B) 2 C) 3 D) 4 E) 5
P(x) = ax2 + bx + c
P(x) + Q(x) = ax2 + bx + c + k = 2x2 – x + 4
a = 2 , b = –1 , c + k = 4 ⇒ c = 4 – k bulunur.
Çözüm Yayınları

P(x) = 2x2 – x + 4 – k
P(k) = 2k2 – k + 4 – k
3. P(x) = (x – 3)n + (x – 2)n – 1 = 2k2 – 2k + 4
2
polinomu x2 – 5x + 6 ile tam bölünebildiğine göre, n için k1 + k2 = = 1 bulunur. (Cevap A)
2
aşağıdakilerden hangisi doğrudur?
7. P(x) = x3 + ax +3
A) Pozitif tek sayıdır.
olmak üzere, P(x – 1) polinomunun x + 1 ile bölümünden
B) Pozitif çift sayıdır. kalan, P(x + 1) polinomunun x – 1 ile bölümünden kalana
eşittir.
C) Negatif tek sayıdır.
Buna göre, a kaçtır?
D) Negatif çift sayıdır.
A) –8 B) –4 C) –2 D) 4 E) 8
E) Herhangi bir pozitif tam sayıdır.
P(x – 1) polinomunun x + 1 ile bölümünden kalan x = –1 için P(–2) ve P(x + 1) polinomunun
x2 – 5x + 6 = (x – 2) (x – 3)
x – 1 ile bölümünden kalan x = 1 için P(2) dir.
P(2) = 0 ve P(3) = 0 dır.
P(–2) = P(2) ⇒ –8 –2a + 3 = 8 + 2a + 3
P(2) = 0 ⇒ (–1)n + 0n – 1 = 0
⇒ –4a = 16
⇒ (–1)n = 1 n pozitif çift sayıdır.
⇒ a = –4 bulunur. (Cevap B)
P(3) = 0 ⇒ 0n + 1n – 1 = 0 ⇒ 1n = 1
(Cevap B)

8. Üçüncü dereceden baş katsayısı 1 olan gerçek katsayılı P(x) 27


polinomu için,
4. P(x) = 2x3 – ax2 + (b + 1)x + a + 3
P(–1) = P(2) = P(5) = 9
polinomu x2 – 2x ile tam bölünebildiğine göre, a + b
olduğuna göre, P(x) polinomunun katsayıları toplamı
toplamı kaçtır?
kaçtır?
A) –18 B) –12 C) 12 D) 15 E) 18
P(x) = (x + 1) (x – 2) (x – 5) + 9
x2 – 2x = 0 ⇒ x(x – 2) = 0 P(1) = 2 . (–1) . (–4) + 9
P(0) = 0 ⇒ a + 3 = 0 = 17 1 7
⇒ a = –3
P(2) = 0 ⇒ 16 – (–3) . 4 + 2b + 2 – 3 + 3 = 0
⇒ 30 + 2b = 0
⇒ b = –15
a + b = –18 dir. (Cevap A)
Test 11 1. (12) 2. A 3. B 4. A 5. C 6. A 7. B 8. 17 9. D 10. A 11. B 12. C 13. D 14. B 15. (3) 16. D

9. a ve b birer gerçek sayı olmak üzere, 13. a ≠ b olmak üzere,


a3 – 3a2b = –25 2a + 6 = 2b + 6
a b
b3 – 3ab2 = –52
olduğuna göre, a · b çarpımı kaçtır?
olduğuna göre, a – b farkı kaçtır?
1 1
A) –5 B) –3 C) 2 D) 3 E) 5 A) B) C) 2 D) 3 E) 4
3 2
Eşitlikler taraf tarafa çıkarılır. 2 2
3 3 a +3 b +3
a3 – 3a2b + 3ab2 – b3 = –25 + 52 2a+ =2b+ ⇒ – =0
a b a b
(a – b)3 = 27 ⇒ a – b = 3 (Cevap D)
a2b + 3b – ab2 –3a
⇒ =0
ab
ab(a – b) –3 (a – b)
⇒ =0
ab
⇒ (a – b) (ab – 3) = 0
⇒ ab = 3 (a ≠ b) (Cevap D)

10. a3 – 2 = 0 olduğuna göre,


1 14. a = b + 2
2 − a +1
a 2 2
olduğuna göre, a − b ifadesinin b cinsinden değeri
(a − b) 3
ifadesinin a cinsinden değeri aşağıdakilerden hangisidir?
aşağıdakilerden hangisidir?
a +1 a +1
A) B) C) a + 1 b +1 b +1
3 2 A) B) C) b + 1
4 2
a −1 a −1
D) E)
2 3 D) 2b + 4 E) 2b + 6
1 a+1 a+1 a+1
= = = (a – b) (a + b) (b + 2) + b 2b + 2 b+1
a2 – a + 1 a3 + 1 2+1 3 = = =
(a + 1) (Cevap A) (a – b) (a – b)2 [(b + 2) – b]2 4 2
Çözüm Yayınları

(Cevap B)

11. a ve b pozitif gerçek sayılar olmak üzere,


a2 – ab – 6b2 = 0

a+b
olduğuna göre, ifadesinin değeri kaçtır?
a −b
3x + 7 = A + B
15.
A) 1 B) 2 C) 3 D) 4 E) 5 x 2 − 2x − 3 x − 3 x + 1
(a + 2b) (a – 3b) = 0 ⇒ a + 2b = 0 veya a – 3b =0 olduğuna göre, A + B toplamı kaçtır?
⇒ a = –2b veya a = 3b
a ve b pozitif gerçek sayı olduğundan a = –2b olamaz. a = 3b dir. 3x + 7 = (x + 1) . A + (x – 3) . B

a+b 3b + b 4b x = 3 için 16 = 4A ⇒ A = 4,
= = = 2 bulunur. 3
a–b 3b – b 2b (Cevap B) x = –1 için 4 = –4B ⇒ B = –1 bulunur.

A + B = 4 – 1 = 3 tür.

28
12. a + b = 2
7
a3 + b3 =
2
olduğuna göre, a · b çarpımı kaçtır?
a 2 − a −1
1 1 3 16.
A) B) C) D) 1 E) 2 a − a −2
3 2 4
ifadesinin sadeleştirilmiş biçimi aşağıdakilerden
a + b = 2 ⇒ (a + b)3 = 8
hangisidir?
⇒ a3 + 3a3b + 3ab2 + b3 = 8
⇒ a3 + b3 + 3ab(a + b) = 8 A) –1 B) –a C) 1 D) a E) a2
7
⇒ + 3ab . 2 = 8 1 a3 – 1
2 a2 –
7 a a
⇒ 6ab = 8 – = =a
2 1
3
a– a3 – 1
⇒ ab = bulunur. (Cevap C)
4 a2 a2 (Cevap D)
Önerme ve Bileşik Önermeler BÖLÜM 02 Test 01
1. p ⇒ q 5. p ⇒ q′
bileşik önermesi aşağıdakilerden hangisine denktir? bileşik önermesinin olumsuzu (değili) aşağıdakilerden
hangisidir?
A) p ∨ q′ B) p′ ∨ q C) p ∧ q
A) p′ ∨ q B) p ∨ q′ C) p′ ∧ q
D) p′ ∧ q E) p ∧ q′
p ⇒ q ≡ pı ∨ q
D) p ∧ q′ E) p ∧ q
(p ⇒ qı)ı ≡ (pı ∨ qı)ı
(Cevap B)
≡p∧q
(Cevap B)

2. Aşağıdaki bileşik önermelerden hangileri p ⇒ q′ bileşik 6. (p ⇒ q) ∨ (p ⇔ r) ≡ 0


önermesine denktir?
olduğuna göre; p, q ve r örenmelerinin doğruluk değerleri
I. p′ ∧ q′ sırasıyla aşağıdakilerden hangisidir?
II. p ∧ q A) 0, 1, 0 B) 1, 1, 0 C) 1, 0, 0
III. p′ ∨ q′ D) 1, 0, 1 E) 1, 1, 1
A) Yalnız I B) Yalnız II C) Yalnız III p⇒q≡0
⇒p≡1,q≡0,r ≡0
p⇔r≡0
D) I ve III E) II ve III
(Cevap C)
p ⇒ q ı ≡ pı ∨ q ı
Çözüm Yayınları

(Cevap C)

7. p ≡ 1, q ≡ 0, r ≡ 0
olduğuna göre, (p ∧ r′)′ ⇒ q önermesi aşağıdakilerden
3. p′ ⇒ q hangisine denktir?
önermesinin değili aşağıdakilerden hangisidir? A) p ∧ r B) p′ C) 1 D) 0 E) q
A) p ∧ q B) p ∨ q C) p′ ∧ q′ (p ∧ rı)ı ⇒ q ≡ (p ∧ rı) ∨ q
≡ (1 ∧ 1) ∨ 0
D) p′ ∨ q′ E) p′ ∨ q ≡ 1∨0
(pı ⇒ qı)ı ≡ (p ∨ q)ı
≡1
≡ pı ∧ q ı
(Cevap C)
(Cevap C)

29

8. p ∧ 1 ≡ 0
q ∨ 0 ≡ 1

olduğuna göre, aşağıdaki önermelerden hangisinin


4. (p ∧ q′) ∨ (p′ ∨ q) doğruluk değeri 1 dir?

bileşik önermesi aşağıdakilerden hangisine denktir? A) p ∨ q′ B) p′ ∧ q C) p ∧ q

A) 0 B) 1 C) p D) q E) p′ D) p ∧ q′ E) p′ ∧ q′
(p ∧ qı) ∨ (pı ∨ q) ≡ ∨ (p ∧ q) ∨ (p ∧ qı)ı p∧1≡0
⇒p≡0,q≡1
r r q∨0≡1
≡1 olduğundan pı ∧ q = 1 ∧ 1 ≡ 1
(Cevap B) (Cevap B)
Test 01 1. B 2. C 3. C 4. B 5. E 6. C 7. C 8. B 9. C 10. C 11. E 12. C 13. D 14. C 15. C 16. E

9. Aşağıdaki ifadelerden hangileri birer önermedir? 13. (p ⇒ q) ⇒ (r ∧ r′)


I. Türkiye′nin başkenti Antalya′dır. bileşik önermesi aşağıdakilerden hangisine denktir?

II. Ayakkabın çok güzelmiş. A) p ∨ q B) p ∨ q′ C) p ∧ q

III. Bir saat 60 dakikadır. D) p ∧ q′ E) p′ ∧ q


(pı ∨ q)ı ∧ 0 ≡ (p ∧ qı)
A) Yalnız I B) I ve II C) I ve III
(Cevap D)
D) II ve III E) I, II ve III
Ayakkabın çok güzelmiş hüküm bildirmediğinden önerme değildir.
(Cevap C)

14. p′ ∨ q ≡ 0
olduğuna göre, aşağıdaki önermelerden hangisinin
doğruluk değeri 0 dır?
10. Aşağıdakilerden hangisi bir çelişkidir?
A) p B) q′ C) p′ ∧ q
A) p ⇒ p′ B) p ∨ p′ C) p ∧ p′
D) p ∨ q E) p′ ∨ q′
D) p′ ⇒ p E) (p ∧ p′)′ pı ∨ 0 ⇒ p ≡ 1
p ∧ qı daima yanlış olduğundan çelişkidir. q ≡ 0 olduğundan
(Cevap C) pı ∧ q ≡ 0 dır.
Çözüm Yayınları

(Cevap C)

15. p ∨ (q ∧ r)′ ≡ 0
11. p ∧ (p ∨ q)′ olduğuna göre p, q ve r önermelerinin doğruluk değerleri
sırasıyla aşağıdakilerden hangisidir?
bileşik önermesi aşağıdakilerden hangisine denktir?
A) 1, 0, 1 B) 1, 1, 1 C) 0, 1, 1
A) p B) p′ C) q D) q′ E) 0
p ∧ (p ∨ q)ı ≡ p ∧ (pı ∧ qı) D) 0, 1, 0 E) 1, 1, 0
≡ (p ∧ pı) ∧ qı p≡0

≡ 0 ∧ qı (q ∧ rı)ı ≡ 0 , q ∧ r ≡ 1 , q ≡ 1 ve r ≡ 1
≡0 (Cevap C)
(Cevap E)

30

12. Aşağıdakilerden hangisi çelişkidir? 16. [(p ⇒ q) ∨ q′] ∨ p


A) p ⇒ 1 B) p ⇒ p′ C) p ⇔ p′ önermesi aşağıdakilerden hangisine denktir?
D) p ∨ p′ E) 0 ⇒ p A) p B) q C) p′ D) q′ E) 1
(p ⇔ pı) önermesi, p veya pı den biri doğru iken oligeri yanlış olduğundan çelişkidir. [(pı ∨ q) ∨ qı] ∨ p ≡ [p ∨(q ∨ qı)] ∨ p
(Cevap C) ≡ (p ∨ 1) ∨ p
≡1∨p
≡1
(Cevap E)
Önerme ve Bileşik Önermeler BÖLÜM 02 Test 02
1. [p ⇒ (p ∧ q)]′ 5. p ⇒ (q ∨ r′) = 0
önermesi aşağıdaki önermelerden hangisine denktir? olduğuna göre, p, q, r önermelerinin doğruluk değerleri
sırasıyla aşağıdakilerden hangisidir?
A) p ∨ q B) p ∨ q′ C) p ∧ q
A) 0, 0, 0 B) 0, 1, 0 C) 1, 0, 0
D) p′ ∧ q E) p ∧ q′
[pı ∨ (p ∧ q)]ı ≡ [(pı ∨ q) ∧ (pı ∨ q)] D) 1, 0, 1 E) 0, 1, 1
≡ [1 ∧ (pı ∨ q)]ı p≡1
≡ (pı ∨ q)ı q ∨ rı ≡ 0 olduğundan

≡p∧ qı q ≡ 0 ve r ≡ 1 olur.

(Cevap E) (Cevap D)

2. Aşağıdaki önermelerden hangisi bir totolojidir?


6. (q ⇒ p)′ ∧ p
A) p ∧ p′ B) p′ ⇒ p C) p ∨ p′
bileşik önermesi aşağıdakilerden hangisine denktir?
D) p ⇔ p′ E) p ⇒ p′
p ∨ pı totolojidir.
A) p B) q C) q′ D) 1 E) 0
(qı ∨ p)ı ∨ p ≡ q ∧ pı ∧ q
(Cevap C)
≡ q ∧ (pı ∧ p)

q∧0
≡0
(Cevap E)
Çözüm Yayınları

3. (p ⇒ q) ∧ r ≡ 1
7. p′ ∧ q ⇒ q′
olduğuna göre, aşağıdakilerden hangisi yanlıştır?
bileşik önermesi aşağıdakilerden hangisine denktir?
A) p ≡ 1 iken q ≡ 1 dir.
A) p B) q C) p′
B) p ≡ 0 iken q ≡ 0 veya q ≡ 1 dir.
D) p ∨ q′ E) p′ ∨ q
C) r ≡ 1 dir. ı ı ı
(p ∧ q) ∨ q ≡ p ∨ q ∨ q ı ı

D) (p ∨ q′) ∨ r′ ≡ 0 olabilir. ≡ p ∨ (qı ∨ qı)



≡ p ∨ qı
E) r′ ⇒ (p ∨ q) ≡ 0 dır.
(Cevap D)
p ⇒ q ≡ 1 ve r ≡ 1 dir.
rı ⇒ (p ∨ q) ≡ 0 ⇒ p ∨ q ≡ 1
olduğundan E şıkkı yanlıştır.
(Cevap E)

31

8. ‟Bugün Pazar ise yarın pazartesidir.ˮ bileşik önermesinin


karşıt tersi aşağıdakilerden hangisidir?

A) Yarın pazartesi ise bugün pazardır.

B) Bugün pazar değilse yarın pazartesi değildir.

4. (p ⇒ q)′ ∨ (q′ ⇒ p)′ C) Bugün pazar değilse yarın pazartesidir.

örenmesi aşağıdakilerden hangisine denktir? D) Yarın pazartesi değilse bugün pazar değildir.

A) 0 B) 1 C) q′ D) p′ E) p ∨ q E) Bugün pazar ise yarın pazartesidir.


(pı ∨ q)ı ∨ (q ∨ p)ı ≡ (p ∧ qı) ∨ (qı ∨ pı) p ⇒ q önermesinin karşıt tersi qı ⇒ pı dir.
≡ (p ∨ pı) ∧ qı Buna uygun seçenek D seçeneğidir.
≡ 1 ∧ qı (Cevap D)
≡ qı
(Cevap C)
Test 02 1. E 2. C 3. E 4. C 5. D 6. E 7. D 8. D 9. C 10. A 11. D 12. A 13. A 14. E 15. D 16. B

9. Aşağıdaki bileşik önermelerden hangisi ya da hangileri 13. p ⇒ [p′ ∨ (q ∧ r)′] ≡ 0


(p′ ⇒ q)′ bileşik önermesine denktir?
olduğuna göre, p, q ve r önermelerinin doğruluk değerleri
I. p′ ∨ q aşağıdakilerden hangisidir?

II. p ∨ q′ A) 1, 1, 1 B) 1, 0, 1 C) 1, 1, 0

III. p′ ∧ q′ D) 0, 1, 1 E) 0, 1, 0

A) Yalnız I B) Yalnız II C) Yalnız III p≡1


[pı ∨ (q ∧ r)ı] ≡ 0
D) I ve II E) II ve III pı ≡ 0 , (q ∧ r)ı ≡ 0
(pı ⇒ q)ı ≡ (p ∨ q)ı p≡1 , q∧r≡1
≡ pı ∧ q ı q≡1 , r≡1
(Cevap C) (Cevap A)

14. p, yanlış bir önerme


q, doğru bir önerme

10. p ∧ (q′ ⇒ r)′ r, doğru bir önerme

bileşik önermesinin doğruluk değeri 1 olduğuna göre, olduğuna göre, aşağıdakilerden hangileri doğru bir
p, q, r önermelerinin doğruluk değerleri sırasıyla önermedir?
aşağıdakilerden hangisidir? I. q ∧ r
A) 1, 0, 0 B) 1, 0, 1 C) 0, 1, 1 II. (p ∧ q) ∧ r
D) 0, 0, 1 E) 1, 1, 0 III. (p′ ∧ q) ∧ r
Çözüm Yayınları

p ∧ (qı ⇒ r)ı ≡ 1
A) Yalnız I B) Yalnız II C) Yalnız III
p ≡ 1 ve (qı ⇒ r)ı ≡ 1
qı ⇒ r ≡ 1 D) II ve III E) I ve III
qı ≡ 1 ve r ≡ 0 I. q ∧ r ≡ 1 ∧ 1 ≡ 1 (doğru)

q ≡ 0 II. (p ∧ q) ∧ r ≡ (0 ∧ 1) ∧ 1

((Cevap A)) ≡ 0 (yanlış)


III. (pı ∧ q) ∧ r ≡ (1 ∧ 1) ∧ 1
≡1
(Cevap E)

11. p ⇒ (p′ ∨ q)
bileşik önermesinin değili aşağıdakilerden hangisidir?

A) p′ B) p C) q 15. p ⇒ (p′ ∨ q)

D) p ∧ q′ E) p′ ∧ q bileşik önermesinin değili aşağıdakilerden hangisidir?


[p ⇒ (pı ∨ q)]ı ≡ [pı ∨ (pı ∨ q]ı
A) p′ B) p C) q
≡ [(pı ∨ pı) ∨ q]ı
≡ (pı ∨ q)ı D) p ∧ q′ E) p′ ∧ q
ı ı ı ı ı ı
≡p∧q [p ∨ (p ∨ q)] ≡ (p ∨ q)
(Cevap D) ≡ p ∧ pı

32 (Cevap D)

12. p ⇔ (p ∨ q)
bileşik önermesi aşağıdakilerden hangisine denktir?

A) p ∨ q′ B) p′ ∨ q C) p ∧ q′ 16. (p′ ⇒ q′) ⇒ (q ∨ q′)′

D) p′ ∧ q E) p ∧ q bileşik önermesi aşağıdakilerden hangisine denktir?

A) p ∧ q′ B) p′ ∧ q C) p ⇒ q
p q qı p∨q p ⇔ (p ∨ q) p∨q
D) p′ E) q′
1 1 0 1 1 1
(p ∨ q)ı ∨ (q ∨ qı)ı ≡ (pı ∧ q) ∨ 0
1 0 1 1 1 1 ≡ pı ∧ q
0 1 0 1 0 0 (Cevap B)
0 0 1 0 1 1

denk (Cevap A)
Önerme ve Bileşik Önermeler BÖLÜM 02 Test 03
1. (p ∧ q′)′ ∨ q′ 5. (p′ ∨ q) ∨ r′ ≡ 0
önermesi aşağıdakilerden hangisine denktir? olduğuna göre, aşağıdakilerden hangisi doğrudur?

A) p B) q C) q′ A) p ∧ q ≡ 1 B) q ∧ r ≡ 1 C) p ∧ r ≡ 1

D) Totoloji E) Çelişki D) p ⇒ q ≡ 1 E) r ⇒ q ≡ 1
(pı ∨ q)ı ≡ p ∨ (q ∨ qı) (pı ∨ q) ∨ rı ≡ 0
≡p∨1 pı ∨ q ≡ 0 ve rı ≡ 0
≡ 1 (Totoloji) p ≡ 1 , q ≡ 0 ve r ≡ 1
(Cevap D) p ∧ q = 1 ∧ 1 = 1 doğrudur.
(Cevap C)

6. (p ∧ r′)′ ⇒ q
bileşik önermesinin doğruluk değeri 0 iken aşağıdaki
önermelerden hangisinin doğruluk değeri 1 dir?

2. p ⇔ q′ A) p ∨ q B) p ∧ r C) p ⇔ q

bileşik önermesi aşağıdaki önermelerden hangisine D) p ∧ r′ E) r ⇒ q


denktir? (q ∨ rı)ı ⇒ p ≡ 0
(q ∨ rı)ı ≡ 1 ve p ≡ 0
A) p ∧ q′ B) p ∨ q C) p′ ⇒ q q ∨ rı ≡ 0 ve q ≡ 0 , r ≡ 0 olur.
D) p′ ⇔ q E) p ∨ q′ p ⇔ q ≡ 0 ⇔ 0 ≡ 1 dir. (Doğru)
(Cevap C)
p q pı qı p ⇔ qı pı ⇔ q
1 0 0 0 0 0
1 0 0 1 1 1
7. p, doğru bir önerme
Çözüm Yayınları

0 1 1 0 1 1
0 0 1 1 0 0 q, yanlış bir önerme
denk (Cevap D) olduğuna göre, aşağıdakilerden hangileri her zaman
doğru bir önermedir?

I. p ∧ q

II. p′ ∧ q

3. p′ ∧ q ≡ 1 III. p ∧ q′

olduğuna göre, aşağıdakilerden hangisi yanlıştır? A) Yalnız I B) Yalnız II C) Yalnız III

A) p ≡ 0 B) q ≡ 1 C) p ⇒ q ≡ 1 D) I ve II E) I ve III
p ∧ qı ≡ 1 ∨ 1 ≡ 1 doğru önermedir.
D) p ∨ q′ ≡ 1 E) p′ ∨ q′ ≡ 1 (Cevap C)
ı
p ∧ q ≡ 1 , p ≡ 0 ve q ≡ 1 dir.
p ∨ qı ≡ 0 ∨ 0 ≡ 0 olur. D şıkkı yanlıştır.
(Cevap D)

8.
p
r 33
q
4. (p ∧ q) ∨ q
bileşik önermesine denk olan önerme aşağıdakilerden
hangisidir?

A) p ∧ q B) p ⇒ q C) q D) p′ E) p
Aşağıdaki bileşik önermelerden hangisi şekildeki elektrik
devresine karşılık gelir?

A) (p ∧ q) ∨ r B) (p ∧ r) ∨ q C) (p ∧ q) ∧ r
p q p∧q (p ∧ q) ∨ q
1 1 1 1
D) (p ∨ q) ∧ r E) (p ∨ q) ∨ r
(p ∨ q) ∧ r
1 0 0 0
(Cevap D)
0 1 0 1
0 0 0 0

denk (Cevap C)
Test 03 1. D 2. D 3. D 4. C 5. C 6. C 7. C 8. D 9. B 10. A 11. A 12. E 13. A 14. A 15. D 16. A

9. p′ ∨ (q ⇒ r) 13. (p ∨ q) ⇒ r′
bileşik önermesinin doğruluk değeri 0 olduğuna önermesinin karşıt tersi aşağıdakilerden hangisidir?
göre; p, q, r önermesinin doğruluk değerleri sırasıyla
A) r ⇒ (p′ ∧ q′) B) r ⇒ (p ∨ q)
aşağıdakilerden hangisidir?
C) r′ ⇒ (p ∧ q) D) r′ ⇒ (p ∨ q)
A) 1, 0, 0 B) 1, 1, 0 C) 0, 1, 0
E) r ⇒ (p ∧ q)
D) 0, 1, 1 E) 0, 0, 0 Karşıt tersi r ⇒ (p ∨ q)ı ≡ r ⇒ pı ∧ qı dir.
p ≡ 0 olduğundan p ≡ 1 dir. (Cevap A)
q ⇒ r ≡ 0 olduğundan q ≡ 1 ve r ≡ 0 dır.
(Cevap B)

10. p: ‟Ankara, Türkiye′nin en kalabalık kentidir.ˮ


14. [(p ∨ q′) ∧ (p′ ∧ q)]′ ∧ p
q: ‟Ankara, Türkiye′nin başkentidir.ˮ
bileşik önermesi aşağıdakilerden hangisine denktir?
önermeleri veriliyor.
A) p B) q C) r
‟Ankara, Türkiye′nin başkentidir fakat en kalabalık kenti
D) p ∧ q D) p′ ∧ q
değildir.ˮ
[(p ∨ qı ) ∧ (p ∨ qı)] ∧ p ≡ [0]ı ∧ p
önermesi aşağıdakilerden hangisiyle ifade edilebilir? r r

A) q ∧ p′ B) p ∧ q C) p′ ∧ q ≡1∧p
≡p
Çözüm Yayınları

D) p ∧ q′ E) p ∨ q′ (Cevap A)
q ∧ pı
(Cevap A)

15. p′ ⇒ (q ∧ r′) ≡ 0
olduğuna göre, aşağıdaki bileşik önermelerden
11. (p ∧ q′)′ ∨ q′ hangisinin doğruluk değeri 1 dir?

önermesi aşağıdakilerden hangisine denktir? A) p ∧ r B) q ∧ r C) p ∧ q

A) Totoloji B) Çelişki C) q D) p ⇒ r E) r ⇒ q
pı ≡ 1
D) p E) q′
q ∨ rı ≡ 0
(p ∧ qı)ı ∨ qı ≡ (pı ∨ q) ∨ qı
p ≡ 1 , q ≡ 0 , r ≡ 1 olur.
≡ pı ∨ (q ∨ qı)
p ⇒ 1 ≡ 1 dir.
≡ pı ∨ 1
(Cevap D)
≡1
(Cevap A)
34

16. (p′ ∨ q) ⇔ q
bileşik önermesi aşağıdakilerden hangisine denktir?
12. [(p′ ∨ q) ⇒ (p ∧ q′)] ∧ q
A) p ∨ q B) p′ ∨ q C) p′ ∨ q′
bileşik önermesinin olumsuzu aşağıdakilerden
D) p ∧ q E) p′ ∧ q′
hangisidir?

A) p B) p′ C) q′
p q pı pı ∨ q (pı ∨ q) ⇔ q p∨q
D) Çelişki E) Totoloji
1 1 0 1 1 1
[(pı ∨ q)ı ∨ (p ∨ qı)] ∧ q ≡ [(p ∨ qı) ∨ (p ∧ qı)] ∧ q
≡ p ∧ (qı ∧ q) 1 0 0 1 1 1

≡p∧0 0 1 1 1 1 1

≡0 0 0 1 0 0 0
olduğundan olumsuzu 1 olur. Totolojidir. denk (Cevap A)
(Cevap E)
Önerme ve Bileşik Önermeler BÖLÜM 02 Test 04
1. [(p ⇒ q′)′ ∨ (p′ ∧ q)]′ 5. (q ⇒ p)′ ∧ r ≡ 1
bileşik önermesi aşağıdakilerden hangisine denktir? olduğuna göre, (p ∧ r)′ ⇔ q′ bileşik önermesinin doğruluk
değeri, aşağıdakilerden hangisinin doğruluk değerine
A) p B) q C) q′
denktir?
D) p ∨ q′ E) p ∨ q
A) p B) q C) r
[(pı ∨ qı)ı ∨ (pı ∧ q)]ı ≡ [(p ∧ q) ∨ (pı ∧ q)]ı
≡ [(p ∨ pı) ∧)]ı D) p ∨ q E) p ∨ r
(q ⇒ p)ı ≡ 1 , q ⇒ p ≡ 0 , q ≡ 1 ve p ≡ 0 , r ≡ 1 dir.
≡ [1 ∧ q]ı
(p ∧ r)ı ⇔ qı ≡ (0 ∧ 1)ı ⇔ 0
≡ qı
≡1⇔0
(Cevap C)
≡0
≡p
(Cevap A)

2. (p ∧ q′) ∨ q
bileşik önermesi aşağıdakilerden hangisine denktir?

A) p ∧ q B) p ∨ q C) p 6. (p′ ∨ q′) ∨ r ≡ 0

D) q E) p′ ∨ q′ olduğuna göre; p, q, r önermelerinin doğruluk değerleri


(p ∧ qı ∨ ≡ (p ∨ q) ∧ (qı ∨ q) sırasıyla aşağıdakilerden hangisidir?
≡ (p ∨ q) ∧ 1
A) 1, 1, 0 B) 1, 0, 0 C) 1, 0, 1

≡p∨q
(Cevap B) D) 0, 1, 0 E) 1, 1, 1
pı ≡ 0 qı ≡ 0 r≡0
p ≡ 1 q≡1
Çözüm Yayınları

(Cevap A)

3. p ⇒ q ≡ 0
olduğuna göre, [(p ⇒ q) ∨ r] bileşik önermesi
aşağıdakilerden hangisine denktir?

A) p B) p′ C) q D) r E) r′
[(p ⇒ q) ∨ r] ≡ 0 ∨ r 7. [p ⇒ (q ∨ r)] ≡ 0
≡r
olduğuna göre, aşağıdaki önermelerden hangisinin
(Cevap D)
doğruluk değeri 1 dir?

A) p ∧ r B) q ∨ r C) q ⇒ p

D) p ⇒ r E) p ∧ q
p ≡ 1 , q ∨ r ≡ 0
q ⇒ p ≡ 1 olur. (p ≡ 1)
(Cevap C)

4. (p ∨ q′) ∧ r ≡ 1 35

olduğuna göre, aşağıdaki bileşik önermelerden hangisi ya


da hangileri daima doğru bir önermedir?

I. (p′ ∧ q) ∨ r′

II. p ∨ (q ∨ r) 8. q′ ⇒ p′

III. p′ ∧ q önermesinin olumsuzu aşağıdakilerden hangisidir?

A) Yalnız I B) Yalnız II C) Yalnız III A) p ∨ q B) p ∧ q′ C) p′ ∨ q

D) I ve II E) II ve III D) p ∧ q E) p′ ∧ q
(p ∨ qı) ∧ r ≡ 1 olduğundan
(qı ⇒ pı)ı ≡ (q ∨ pı)ı dir.
p ∨ qı ≡ 1 ve r ≡ 1 olur.
≡ qı ∧ p
r ≡ 1 olduğundan II. öncül daima doğru bir önerme olur.
(Cevap B)
(Cevap B)
Test 04 1. C 2. B 3. D 4. B 5. A 6. A 7. C 8. B 9. E 10. A 11. D 12. D 13. B 14. B 15. A 16. D

9. (p ⇒ q)′ 13. q
önermesinin dengi aşağıdakilerden hangisidir? p
r
A) p B) p′ ∨ q C) p′ ∧ q

D) p ∨ q E) p ∧ q′
ı ı ı
(p ⇒ q) ≡ [(p ∨ q)
Şekildeki elektrik devresine göre, aşağıdaki durumların
≡ p ∧ qı
hangisinde devreden akım geçer?
(Cevap E)
A) p ≡ 1, q ≡ 0, r ≡ 0 B) p ≡ 1, q ≡ 0, r ≡ 1

C) p ≡ 0, q ≡ 1, r ≡ 1 D) p ≡ 0, q ≡ 1, r ≡ 0

E) p ≡ 0, q ≡ 0, r ≡ 0
??????????????????????????????????????????????

(Cevap B)

10. p ⇒ r′ ≡ 0
olduğuna göre, aşağıdakilerden hangisi kesinlikle
yanlıştır?

A) p′ ⇒ r ≡ 0 B) r ⇒ p′ ≡ 0 C) p ∨ r ≡ 1
14. [(p′ ⇒ q) ∧ q′] ∧ r′ ≡ 1
D) p ⇒ r ≡ 1 E) p′ ∨ r′ ≡ 0
olduğuna göre; p, q, r önermelerinin doğruluk değerleri
p≡1 , r≡0
sırasıyla aşağıdakilerden hangisidir?
r ≡ 1
pı ⇒ r ≡ 0 ⇒ 1 A) 0, 0, 1 B) 1, 0, 0 C) 1, 0, 1
≡ 1 dir. pı ⇒ r ≡ 0 yanlıştır.
Çözüm Yayınları

D) 1, 1, 0 E) 1, 1, 1
(Cevap A)
(pı ⇒ q) ∧ qı ≡ 1 ve rı ≡ 1 olmalıdır.
pı ⇒ q ≡ 1
pı ⇒ q ≡ 1
qı ≡ 1 r≡0
pı ≡ 0
q≡0
p≡1

(Cevap B)

11. (p ∧ q′) ⇒ p
bileşik önermesi aşağıdaki önermelerden hangisine 15. Aşağıdaki bileşik önermelerden hangisi
denktir?
(p ⇒ q) ∨ r′
A) p ∨ q B) p ∨ q′ C) p′ ∨ q
bileşik önermesine denktir?
D) p ∨ p′ E) q ∧ q′
A) (p ∧ r) ⇒ q B) p ⇒ q C) p ⇒ r
(q ∧ qı) ⇒ p ≡ (p ∧ qı)ı ∨ p
≡ (pı ∨ q) ∨ p D) (p ∧ r) ∨ q E) (p ∧ r) ∧ q
≡ (pı ∨ p) ∨ q pı ∨ q ∨ rı ≡ (pı ∨ rı) ∨ q
≡1∨q ≡ (p ∧ r)ı ∨ q
≡1 ≡ (pı ∧ r) ⇒ q
36 ≡ p ∨ pı (Cevap A)
(Cevap D)

12. p ∨ (p ∨ q′)′ 16. [(p ⇒ 1) ∧ (1 ⇒ p′)] ∧ q


bileşik önermesi aşağıdakilerden hangisine denktir? bileşik önermesi aşağıdakilerden hangisine denktir?
A) p ∧ q B) p′ ∧ q C) p ∧ q′ A) p B) p′ C) p′ ∧ q′
D) p ∨ q E) q D) p′ ∧ q E) p′ ∨ q′
p ∨ (p ∨ qı) ≡ p ∨ (pı ∧ q) [(p ⇒ 1) ∧ (1 ⇒ pı)] ∧ q ≡ [1 ∧ pı] ∧ q
≡ (p ∨ pı) ∧ (p ∨ q) ≡ pı ∧ p
≡1∨p∨q (Cevap D)
≡p∨q
(Cevap D)
Açık Önermeler BÖLÜM 02 Test 05
1. x ∈ N olmak üzere, 5. [(∃x ∈ R, x2 ≤ x) ∧ (∀x ∈ R, x2 > 0)]
p(x): x2 + 1 < 10 önermesinin değili (olumsuzu) aşağıdakilerden
hangisidir?
açık önermesinin doğruluk kümesi aşağıdakilerden
hangisidir? A) [(∀x ∈ R, x2 ≤ x) ∨ (∃x ∈ R, x2 > 0)]

A) {0, 1} B) {0, 2} C) {1, 2} B) [(∀x ∈ R, x2 > x) ∨ (∃x ∈ R, x2 ≤ 0)]

D) {0, 1, 2} E) {1, 2, 3} C) [(∀x ∈ R, x2 > x) ∧ (∃x ∈ R, x2 ≥ 0)]


x = 0 , x = 1 , x = 2 için açık önerme doğru olur
D) [(∃x ∈ R, x2 > x) ∧ (∀x ∈ R, x2 ≤ 0)]
(Cevap D)
E) [(∃x ∈ R, x2 ≤ x) ∨ (∀x ∈ R, x2 > 0)]
(∀x ∈ R , x2 > x) ∨ (∃x ∈ R , x2 ≤ 0)
(Cevap B)

2. x bir gerçek sayı olduğuna göre, aşağıdaki açık


önermelerden hangisinin doğruluk değeri sıfırdır?

∀x, x2 + 1 > 0
A)
6. Aşağıdakilerden hangisi ‟Bazı gerçek sayıların üçüncü kuvveti
∃x, x2 = x
B)
negatiftir.ˮ önermesinin sembolik yazılışıdır?
∃x, x2 < 0
C)
A) ∃x ∈ R, x < 0 B) ∀x ∈ R, x3 < 0
D)
∀x, x2 – 1 = (x – 1) (x + 1)
C) ∃x ∈ R, x3 < 0 D) ∀x ∈ R, x > 0
∃x, x2 = –x
E)
E) ∀x ∈ R, x3 > 0
Karesi sıfırdan küçük olan gerçek sayı yoktur.
∃x ∈ R , x3 < 0
Çözüm Yayınları

(Cevap C)
(Cevap C)

3. x2 = 4 ⇒ (x = –2 ∨ x = 2)
önermesinin karşıt tersi aşağıdakilerden hangisidir?

A) (x2 ≠ 4 ⇒ (x ≠ –2 ∧ x ≠ 2) 7. p(x): ‟x ∈ N, x2 > 19ˮ


B) (x = –2 ∨ x = 2) ⇒ x2 = 4 açık önermesinin doğruluk kümesi aşağıdakilerden
C) (x ≠ –2 ∨ x = 2) ⇒ x2 ≠ 4 hangisidir?

D) x2 ≠ 4 ⇒ (x = –2 ∨ x = 2) A) {2, 3, 4, ...} B) {3, 4, 5, ...} C) {5, 6, 7, ...}

E) (x ≠ –2 ∧ x ≠ 2) ⇒ x2 ≠ 4 D) {7, 8, 9, ...} E) {6, 7, 8, ...}


{5, 6, 7, ...}
(x = –2 ∨ x = 2)ı ⇒ (x2 = 4)ı
(Cevap C)
(x ≠ –2 ∧ x ≠ 2) ⇒ x2 ≠ 4
(Cevap E)
37

4. x bir tam sayı olmak üzere,


p(x): 8 < x2 ≤ 49 8. 1 + 3 + 5 + . . . + 21
açık önermesinin doğruluk kümesi kaç elemanlıdır? işleminin sonucu kaçtır?

A) 5 B) 6 C) 8 D) 9 E) 10 A) 100 B) 121 C) 144 D) 169 E) 196


x = 3 , x = 4 , ........, x = 7 ve 21 – 1
Terim sayısı n = + 1 ⇒ n = 11 dir.
x = –3 , x = –4 , ........, x = –7 2
2
olmak üzere, açık önermenin doğruluk kümesi 10 elemanlıdır. Toplamları n = 121 olur.
(Cevap B)
(Cevap E)
Test 05 1. D 2. C 3. E 4. E 5. B 6. C 7. C 8. B 9. A 10. B 11. A 12. B 13. D 14. D 15. B 16. E

9. (x2 = 16) ⇒ (x = –4 ∧ x = 4) 13. Aşağıdaki ifadelerden hangisi önerme değildir?


önermesinin karşıtı aşağıdakilerden hangisidir? A) 5 < 3 B) 2 < 4 C) 3 · 3 = 9

A) (x = –4 ∧ x = 4) ⇒ (x2 = 16) D) 2x + 4 E) 5 tek sayıdır.


2x + 4 hüküm bildirmediği için önerme değildir.
B) (x ≠ –4 ∧ x ≠ 4) ⇒ (x2 ≠ 16)
(Cevap D)
C) (x = –4 ∧ x = 4) ⇒ (x2 ≠ 16)

D) (x2 ≠ 16) ⇒ (x ≠ –4 ∧ x ≠ 4)

E) (x2 = 16) ⇒ (x ≠ –4 ∧ x ≠ 4)

(x = –4 ∧ x = 4) ⇒ x2 = 16
(Cevap A)
14. x bir gerçek sayı olduğuna göre, aşağıdaki önermelerden
hangisi yanlıştır?

A) ∃x, (x – 1)2 ≤ 0 B) ∀x, (x – 1)2 ≥ 0

C) ∃x, x2 – 4 = 0 D) ∀x, x2 ≥ x

10. p : ∀x ∈ R+ için |x| = –x E) ∃x, x3 < x


0 < x < 1 için x2 < x olduğundan
x
q: ∀x, y ∈ R için ∈Q ∀x , x2 ≥ x yanlıştır.
y
(Cevap D)
önermeleri için aşağıdakilerden hangisi kesinlikle bir
totolojidir?

A) p ∧ q B) q ⇒ p C) p′ ∧ q

D) p ⇔ q′ E) p ∧ q′
15. p: x çift sayı
p ≡ 0 ve q ≡ 0 olduğundan
Çözüm Yayınları

q ⇒ p ≡ 1 dir. q: y tek sayı


(Cevap B)
önermeleri veriliyor.

Buna göre,

I. x + y toplamı tek sayıdır.

II. x · y çarpımı çift sayıdır.


11. ∀n ∈ N+ için,
n(n + 1)(n + 2) III. x + y2 toplamı çift sayıdır.
1 · 2 + 2 · 3 + 3 · 4 + . . . + n(n + 1) =
3
ifadelerinden hangileri doğrudur?
olduğuna göre,
A) Yalnız I B) I ve II C) I ve III
5 · 6 + 6 · 7 + 7 · 8 + . . . + 13 · 14
D) II ve III E) I, II ve III
işleminin sonucu kaçtır?
I ve II doğrudur
A) 870 B) 910 C) 1190 D) 1215 E) 1578 (Cevap B)
(1.2 + 2.3 + 3.4 + ... + 13.14) – (1.2 + 2.3 + 3.4 + 4.5)
13.14.15 4.5.6
= – = 870
3 3 (Cevap A)

38
12. (x2 < 25) ∨ (2x – 3 ≥ 0) 16. (x = 3k ve x = 5m) ⇒ (x = 15n), (k, m, n ∈ Z)
önermesinin olumsuzu aşağıdakilerden hangisidir? önermesinin karşıt tersi aşağıdakilerden hangisidir?

A) (x2 > 25) ∨ (2x – 3) ≤ 0 A) (x = 15n) ⇒ (x = 3k ve x = 5m), (k, m, n ∈ Z)

B) (x2 ≥ 25) ∧ (2x – 3 < 0) B) (x ≠ 15n) ⇒ (x ≠ 3k ve x ≠ 5m), (k, m, n ∈ Z)


2
C) (x > 25) ∨ (2x – 3 < 0) C) (x ≠ 3k veya x ≠ 5m) ⇒ (x ≠ 15n), (k, m, n ∈ Z)

D) (x2 ≥ 25) ∧ (2x – 3 ≤ 0) D) (x ≠ 3k ve x ≠ 5m) ⇒ (x ≠ 15n), (k, m, n ∈ Z)

E) x2 ≥ 25 ∨ 2x – 3 > 0 E) (x ≠ 15n) ⇒ (x ≠ 3k veya x ≠ 5m), (k, m, n ∈ Z)


(x2 ≥ 25) ∧ (2x – 3 < 0) (x = 15n)ı ⇒ (x = 3k ve x = 5m)ı
(Cevap B) ≡ x ≠ 5n ⇒ (x ≠ 3k veya x ≠ 5m)
(Cevap E)
Açık Önermeler BÖLÜM 02 Test 06
1. x ∈ Z için, 5. x, y ∈ Z için
p(x): x2 + 1 = 17 p(x): |x + y – 3| + (y + 2)2 = 0

açık önermesinin çözüm kümesi aşağıdakilerden açık önermesi veriliyor.


hangisidir?
Buna göre x · y çarpımı kaçtır?
A) {2, 4} B) {–2, 4} C) {–4, 4}
A) –10 B) –6 C) 6 D) 8 E) 10
D) {0, 4} E) {–4, 0, 4} x+y–3=0
x2 + 1 = 17 ⇒ x2 – 16 = 0 ⇒ y = –2 , x = 5 bulunur.
y+2=0
⇒ (x + 4) (x – 4) = 0
x.y = – 10 dur.
⇒ x + 4 = 0 veya x – 4 = 0
(Cevap A)
⇒ x = –4 veya x = 4
(Cevap C)

1 + 1 + 1 + ... + 1
6.
1⋅ 2 2 ⋅ 3 3 ⋅ 4 15 ⋅ 16
2. x ∈ Z için,
işleminin sonucu kaçtır?
p(x): x2 < 4 12 13 14 15 16
A) B) C) D) E)
açık önermesinin doğruluk kümesi aşağıdakilerden 13 14 15 16 17
hangisidir? 1 1 1 n
+ +...+ = dir.
A) {–1, 0, 1} B) {0, 1, 2} C) {–2, –1, 0} 1.2 2.3 n(n + 1) n+1
15
Buna göre, işlemin sonucu olur.
D) {–1, 1} E) {–2, 2} 16 (Cevap D)
x = –1 , x = 0 , x = 1 olabilir.
(Cevap A)
Çözüm Yayınları

7. (∀x, x2 ≥ 0) ∧ (∃x, x3 = x)
bileşik önermesinin değili (olumsuzu) aşağıdakilerden
hangisidir?

A) (∀x, x2 < 0) ∨ (∃x, x3 ≠ x)


3. x ∈ R için,
B) (∃x, x2 ≥ 0) ∨ (∀x, x2 = x)
p(x): |x – 2| + |y + 3| = 0
C) (∃x, x2 < 0) ∧ (∀x, x2 ≠ x)
açık önermesi veriliyor.
D) (∀x, x2 < 0) ∧ (∃x, x2 = x)
Buna göre, x · y çarpımı kaçtır?
E) (∃x, x2 < 0) ∨ (∀x, x3 ≠ x)
A) –12 B) –9 C) –6 D) 6 E) 9 (∃x , x2 < 0) ∨ ∀x , x3 ≠ x
x – 2 = 0 ve y + 3 = 0 ⇒ x = 2 ve y = –3
(Cevap E)
⇒ x.y = –6
(Cevap C)

39

8. x ∈ N için,
4. ∃x ∈ R için x3 <x
p(x): |x – 2| = 2 – x
önermesinin olumsuzu aşağıdakilerden hangisidir?
açık önermesinin çözüm kümesi aşağıdakilerden
A) ∀x ∈ R için x3 < x B) ∀x ∈ R için x3 ≥ x hangisidir?

C) ∀x ∈ R için x3 > x D) ∃x ∈ R için x3 > x A) {1, 2} B) {0, 1, 2} C) {0, 1}

E) ∃x ∈ R için x3 ≥x D) {0, 2} E) {–2, –1, 0, 1, 2}


∀x ∈ R için x3 ≥ x |x – 2| = –(x – 2) ⇒ x – 2 ≤ 0
(Cevap B) ⇒x≤2
x = 0 , x = 1 ve x = 2 bulunur.
(Cevap B)
Test 06 1. C 2. A 3. C 4. B 5. A 6. D 7. E 8. B 9. E 10. C 11. D 12. E 13. B 14. C 15. E 16. E

9. x = 3 ⇒ x2 = 9 13. (x = –4) ⇒ (x2 = 16)


önermesinin karşıt tersi aşağıdakilerden hangisidir? koşullu önermesinin karşıt tersi aşağıdakilerden
hangisidir?
A) x ≠ 3 ⇒ x2 ≠ 9 B) x2 = 9 ⇒ x = 3
A) (x2 = 16) ⇒ (x ≠ –4)
C) x ≠ 3 ⇒ x2 = 9 D) x2 ≠ 9 ⇒ x = 3
B) (x2 ≠ 16) ⇒ (x ≠ –4)
E) x2 ≠ 9 ⇒ x ≠ 3
(x2 = 9)I ⇒ (x = 3)I ≡ x2 ≠ 9 ⇒ x ≠ 3 C) (x ≠ 4) ⇒ (x2 = 16)
(Cevap E)
D) (x ≠ –4) ⇒ (x2 = 16)

E) (x2 ≠ 16) ⇒ (x2 ≠ 4)


(x2 = 16)ı ⇒ (x = –4)ı ≡ x2 ≠ 16 ⇒ x ≠ –4
(Cevap B)

10. x ∈ Z için,
p(x): 7 < x2 – 1 ≤ 24

açık önermesinin doğruluk kümesinin eleman sayısı


kaçtır?

A) 2 B) 4 C) 6 D) 8 E) 10 14. A = 12 + 22 + 32 + 42 + . . . + 102
7 < x2 – 1 ≤ 24 ⇒ 8 < x2 ≤ 25
B = 1 + 2 + 3 + 4 + . . . + 10
x tam sayısı –3 , –4 , –5 ve 3, 4, 5 olabilir.
(Cevap C) olduğuna göre, A – B farkı kaçtır?

A) 210 B) 270 C) 330 D) 340 E) 370


10 . 11 . 21 10 . 11
A–B= – = 330
6 2 (Cevap C)
Çözüm Yayınları

11. (∃x ∈ N için x ≥ x2) ∨ (∀x ∈ R için x2 ≥ 0)


önermesinin değili aşağıdakilerden hangisidir?

A) (∀x ∈ N için x < x2) ∧ (∀x ∈ R için x2 < 0)

B) (∀x ∈ N için x ≥ x2) ∧ (∃x ∈ R için x2 < 0)


15. x doğal sayı ve x > 10 olmak üzere, 1 den x e kadar doğal
C) (∀x ∈ N için x ≤ x2) ∧ (∃x ∈ R için x2 < 0) sayıların toplamı a, 10 dan x e kadar doğal sayıların toplamı b
D) (∀x ∈ N için x < x2) ∧ (∃x ∈ R için x2 < 0) dir.

E) (∃x ∈ N için x < x2) ∧ (∀x ∈ R için x2 < 0) Buna göre, a – b farkı kaçtır?
??????????????????????????????????????????????????? A) 35 B) 37 C) 40 D) 42 E) 45
(Cevap D) x . (x + 1) x(x + 1) 9.10
a–b= – –
2 2 2

= 45
(Cevap E)

12. m ve n pozitif tam sayıları için,


40
‟mn çift ⇒ m çifttir.ˮ

gerektirmesinin karşıtı aşağıdakilerden hangisidir? 16. ∀n ∈ N+ için


2
A) m çift ⇒ mn tektir.ˮ  n(n + 1) 
13 + 23 + 33 + . . . + n3 = 
 2 
B) ‟mn çift değil ⇒ m çift değildir.ˮ olduğuna göre,
C) ‟mn tek değil ⇒ m çifttir.ˮ 53 + 63 + 73 + . . . + 103
D) ‟mn tek değil ⇒ m çift değildir.ˮ işleminin sonucu kaçtır?
n
E) ‟m çift ⇒ m çifttir.ˮ A) 2725 B) 2800 C) 2825 D) 2900 E) 2925
m çift ⇒ mn çifttir. 10.11 2
4.5 2

(Cevap E) (13 + 23 + 33 +...+ 103) – (13 + 23 + 33 + 43) = –


2 2

= 2925
(Cevap E)
BİRE BİR BÖLÜM 02 Test 07
1. Aşağıdakilerden hangisi 5. ∃x, x2 + 2x – 3 < 0
(p ∨ q) ⇒ (q ∧ r) önermesinin olumsuzu aşağıdakilerden hangisidir?

önermesine denktir? A) ∀x, x2 + 2x – 3 ≤ 0 B) ∀x, x2 + 2x – 3 ≥ 0

A) (p′ ∧ q′) ⇒ (q′ ∨ r′) B) (p′ ∧ q′) ⇒ (p′ ∧ r′) C) ∃x, x2 + 2x – 3 ≥ 0 D) ∃x, x2 + 2x – 3 > 0

C) (p′ ∨ q′) ⇒ (q′ ∧ r′) D) (q′ ∧ r′) ⇒ (p′ ∨ q′) E) ∀x, x2 + 2x – 3 > 0
∀x , x2 + 2x – 3 ≥ 0
E) (q′ ∨ r′) ⇒ (p′ ∧ q′)
(Cevap B)
(p ∨ q) ⇒ (p ∧ r) önermesi karşıt tersine denktir.
(p ∧ r)I ⇒ (p ∨ q)I ≡ (qı ∨ rı) ⇒ (pı ∧ qı)
(Cevap E)

2. p: x = 0
q: x + y = 0
6. x bir gerçek sayı olduğuna göre, aşağıdaki önermelerden
r: x · y = 0
hangisi doğrudur?
önermeleri veriliyor.
A) ∀x, (x + 1)2 > 0 B) ∃x, x2 + x + 1 < 0
Buna göre, aşağıdaki koşullu önermelerden hangisi 1 =0
doğrudur? C) ∃x, D) ∀x, x2 + 3x + 2 ≥ 0
x −1
A) r ⇒ p B) p ⇒ r C) q ⇒ p E) ∃x, x2 – 1 ≤ 0
–1 ≤ x ≤ 1 için x2 ≤ 1 ve x2 – 1 ≤ 0 dır.
D) p ⇒ q E) q ⇒ r
(Cevap E)
x = 0 ⇒ x.y = 0 dır.
(Cevap B)
Çözüm Yayınları

3. p: 2+ 3 = 5

q: 3 − 2 =1

r: 2⋅ 3 = 6

önermeleri veriliyor. 7. x ve y gerçek sayılar olmak üzere,


Buna göre, aşağıdaki bileşik önermelerden hangisi p: ∀x, (x + 1)2 > 0
doğrudur?
q: ∃x, x2 < x
A) p ∧ (r ∨ q) B) (p ∨ q) ∧ r C) r ⇒ (p ∧ q)
önermeleri veriliyor.
D) p ∨ (r ⇒ q) E) p ⇒ (q ∧ r)
p ≡ 0 , q ≡ 0 ve r ≡ 1 dir. Buna göre, aşağıdaki önermelerden hangisi doğrudur?
p ⇒ q ∧ r ≡ 0 ⇒ (0 ∧ 1) doğrudur. A) p ∧ q B) p ∨ q′ C) q ⇒ p
(Cevap E)
D) p ⇒ q E) p′ ∧ q′
4. p: a = 0 p ≡ 0 , q ≡ 1 olduğundan
p ⇒ ⊄ ≡ 1 olur.
q: b = 0 (Cevap D)

önermeleri veriliyor.

Buna göre, a ve b gerçek sayıları için,


41
I. a + b = 0

II. a · b = 0

III. a2 + b2 = 0
8. (q′ ∧ r) ⇒ p
önermelerinden hangileri p ∧ q önermesine denktir?
bileşik önermesi yanlış iken, aşağıdakilerden hangisi
A) Yalnız II B) Yalnız III C) I ve II doğrudur?
D) I ve III E) II ve III A) r ⇒ p B) r ⇒ q C) p ∨ q
I. a + b = 0 ⇒ a = –b olduğundan
D) p ∧ r E) p ⇔ q
a = 0 ve b = 0 olmayabilir.
qı ∧ r ≡ 1 ve p ≡ 0 olmalıdır.
II. a.b = 0 ⇒ a = 0 veya b = 0
qı ∧ r ≡ 1 için q ≡ 0 ve r ≡ 1 olur.
a ve b den herhangi biri sıfır olmayabilir.
p ⇔ q doğrudur.
III. a2 + b2 = 0 ⇒ (a = 0) ∧ (b = 0) dır.
(Cevap E)
(Cevap B)
Test 07 1. E 2. B 3. E 4. B 5. B 6. E 7. D 8. E 9. B 10. C 11. E 12. A 13. C 14. E 15. A 16. D

9. Aşağıdakilerden hangisi 13. ∀x, x2 + 1 > 0


(p ∧ q) ⇒ r′ önermesinin olumsuzu aşağıdakilerden hangisidir?

önermesine denktir? A) ∃x, x2 + 1 > 0 B) ∃x, x2 – 1 > 0

A) r′ ⇒ (p ∧ q) B) r ⇒ (p′ ∨ q′) C) r ⇒ (p ∧ q) C) ∃x, x2 + 1 ≤ 0 D) ∀x, x2 + 1 ≤ 0

D) r′ ⇒ p′ ∧ q′ E) r ⇒ (p′ ∧ q′) E) ∀x, x2 – 1 ≤ 0


(rı)ı ⇒ (p ∧ q)ı ≡ r ⇒ pı ∨ qı ∀x , x2 + 1 > 0)ı ≡ ∃x , x2 + 1 ≤ 0
(Cevap B) (Cevap E)

10. p: a = 0
q: a – b = 0

r: a2 · b = 0

önermeleri veriliyor.

Buna göre, aşağıdaki koşullu önermelerden hangisi 14. x bir gerçek sayı olduğuna göre, aşağıdaki önermelerden
doğrudur? hangisi doğrudur?

A) q ⇒ p B) p ⇒ q C) p ⇒ r A) ∃x, x2 – x + 1 < 0 B) ∀x, (x – 1)2 > 0

D) r ⇒ p E) q ⇒ r C) ∀x, x2 + 4x + 3 ≥ 0 D) ∃x, (x – 1)2 < 0


a = 0 ⇒ a2b = 0 yani p ⇒ r doğrudur. E) ∃x, x2 ≤ x
(Cevap C) 0 ≤ x ≤ 1 için x2 ≤ x doğrudur.
(Cevap E)
Çözüm Yayınları

11. x, y ∈ R ve x < 0 < y olmak üzere,


p: |x| + |y| = |x + y|

q: |x| · |y| = |x · y|

r: IxI = x
IyI y
önermeleri veriliyor.
15. (p ∧ q′) v q
Buna göre, aşağıdaki bileşik önermelerden hangisi
doğrudur? bileşik önermesinin sadeleştirilmiş biçimi aşağıdakilerden
hangisidir?
A) q ⇒ p B) r ⇒ p C) q ⇒ (p ∧ r)
A) p ∨ q B) p ∨ q′ C) p′ ∨ q
D) r ⇒ (p ∧ q) E) p ⇒ q
q doğru, p ve r yanlış önermedir.
D) p ∧ q E) p ∧ q′
p ⇒ q doğru olur. (p ∧ qı) ∨ q ≡ (p ∨ q) ∧ (qı ∨ q)

(Cevap E) ≡ (p ∨ q) ∧ 1
≡p∨q
12. p: x = 0 (Cevap A)
q:y=0

önermeleri veriliyor.
42
Buna göre x ve y gerçek sayıları için,
16. (p ∨ q) ⇒ p
I. x2 + y2 = 0
bileşik önermesi aşağıdaki önermelerden hangisine
II. x3 + y3 = 0 denktir?
III. x · y = 0 A) p ∨ q B) p ∧ q C) p ∧ q′
önermelerinden hangileri (p ⇒ q′)′ önermesine denktir? D) q ⇒ p E) p ⇒ q
A) Yalnız I B) I ve II C) I ve III
(p ∨ q) ⇒ p ≡ (p ∨ q)ı ∨ p
D) II ve III E) I, II ve III ≡ (pı ∧ qı) ∨ p

(p ⇒ qı)I ≡ (pı ∨ qı) ≡ p ∨ q ≡ (pı ∨ p)ı ∧ (qı ∨ p)

x2 + y2 = 0 ise x = 0 ve y = 0 dır. ≡ 1 ∧ (qı ∨ p)

(Cevap A) ≡ qı ∨ p
≡q⇒p
(Cevap D)
Bölme ve Bölünebilme BÖLÜM 03 Test 01
1. Üç basamaklı 5AB sayısı iki basamaklı AB sayısına 5. Üç basamaklı ABC doğal sayısı 2 ile, 5 ile ve 9 ile tam
bölündüğünde, bölüm 13 ve kalan 8 olmaktadır. bölünebilmektedir.

Buna göre, A + B toplamı kaçtır? Buna göre, A > B > C koşulunu sağlayan kaç tane ABC
sayısı yazılabilir?
A) 5 B) 6 C) 7 D) 8 E) 9
A) 1 B) 2 C) 3 D) 4 E) 5
5AB = 13 . AB + 8 ⇒ 500 + AB = 13AB + 8
ABC sayısı 2 ve 5 ile tam bölündüğünden C = 0 dır.

⇒ 492 = 12 . AB
A = 9 olamaz. ( Bu durumda B = 9 veya B = 0 olması gerekir.)
⇒ AB = 41 A = 8 için B = 1

A + B = 4 + 1 = 5 bulunur. A = 7 için B = 2

(Cevap A) A = 6 için B = 3

A = 5 için B = 4 bulunur.

(Cevap D)

6. Dört basamaklı AB24 sayısının 36 ile bölümünden kalan iki


basamaklı CD sayısıdır.

AB24 36

...
2. Üç basamaklı ABC sayısı, iki basamaklı AB sayısına
bölündüğünde bölüm ile kalanın toplamı 17 olduğuna CD
göre, C rakamı kaçtır? Buna göre, bulunabilecek CD sayılarının toplamı kaçtır?
A) 3 B) 4 C) 5 D) 6 E) 7 A) 132 B) 136 C) 140 D) 144 E) 148
ABC AB
10 + C = 17 ⇒ C = 7 AB24
ve 36 sayıları 4 ile tam bölüündüğnden CD sayısı 4 ile tam bölünür.
AB 10
C (Cevap E) CD → 12 + 16 + 20 + 24 + 28 + 32 = 132
Çözüm Yayınları

(Cevap A)

7.
A 29
...

34
B2
3. 73 + 143 + 213 Yukarıdaki bölme işleminde A üç basamaklı bir doğal sayıdır.
toplamının kaç tane pozitif tam sayı böleni vardır? Buna göre, B doğal sayısının alabileceği en büyük değer
A) 18 B) 24 C) 36 D) 45 E) 48 kaçtır?

73 3 3 3 2
(1 + 2 + 3 ) = 7 . 2 . 2
2 A) 2 B) 3 C) 4 D) 5 E) 6
A = 29 . 34 + B2 ⇒ A = 986 + B2
(3 + 1) . (2 + 1) . (2 + 1) = 36 tane pozitif tam sayı bölüne vardır?
A üç basamaklı olduğundan
(Cevap C)
B2 < 1000 - 986 ⇒ B2 < 14 olur.

B nin alabilceği en büyük değer 3 tür. 43


(Cevap B)

8. Dört basamaklı 58A7 sayısının 11 ile bölümünden kalan 3


4. Aşağıdaki sayılardan hangisi asal sayıdır? olduğuna göre, 9 ile bölümünden kalan kaçtır?
A) 311 + 513 B) 715 + 1 C) 313 A) 0 B) 1 C) 2 D) 3 E) 4
D) 257 E) 53781 7 – A + 8 – 5 = 11k + 3 (k∈Z+)

11k = 7 – A, A = 7 dir.
257 sayısı asal sayıdır.
5877 sayısının 9 ile bölümünden kalan, rakamlarının toplamının 9 ile bölümünden
(Cevap D)
kalandır. Sıfır bulunur.

(Cevap A)
Test 01 1. A 2. E 3. C 4. D 5. D 6. A 7. B 8. A 9. A 10. E 11. C 12. D 13. B 14. C 15. B 16. B

9. 2700 sayısının pozitif tam sayı bölenlerinden kaç tanesi


13. a nın kaç farklı doğal sayı değeri için a + 25 ifadesi
tek sayıdır? a+1
doğal sayı olur?
A) 12 B) 15 C) 18 D) 20 E) 24
27 . 100 = 33 . 22 . 52
A) 6 B) 8 C) 9 D) 10 E) 12
a + 25 a+1 24
a + 25 = 1 +
= 22 . 33 . 52 a+1 1 a+1

Tek sayı olan 33 ve 52 asal çarpanlarının üsleri bir artırılarak çarpılır. 24

a + 1 sayısı 24 ün bölenleri olmalıdır.


4 . 3 = 12 elde edilir.
24 = 23 . 3 sayısının 4 . 2 = 8 tane doğal sayı böleni vardır
(Cevap A)
a+1=1⇒a=0

a+1=2⇒a=1
.
.
.

10. Dört basamaklı (2a7b) sayısı 9 ile tam bölünebildiğine a + 1 = 24 ⇒ a = 23

göre, beş basamaklı (5ab48) sayısının 9 ile bölümünden için verilen ifade doğal sayı olur.
kalan kaçtır? (Cevap B)
14. a ve b birer pozitif tam sayı olmak üzere,
A) 0 B) 2 C) 5 D) 7 E) 8
2 + a + 7 + b = 9k (k∈Z ) +
450 · a = (b + 6)3

a + b toplamı 9 un katıdır. olduğuna göre, a + b toplamının alabileceği en küçük


değer kaçtır?
5 + a + b + 4 + 8 = a + b + 17 ve 17 nin 9 ile bölümünden kalan 8 dir.
A) 72 B) 80 C) 84 D) 90 E) 96
(Cevap E)
450 . a = (b + 6)3 ⇒ 2 . 32 . 52 . a = (b + 6)3

⇒ a = 22 . 3 . 5
Çözüm Yayınları

⇒ a = 60 ve

(b + 6)3 = 23 . 33 . 53 ⇒ b + 6 = 2 . 3 . 5

11. 30! · (5 + 12!) ⇒ b = 24 bulunur.

sayısının sondan kaç basamağı sıfırdır? a + b = 60 + 24 = 84 tür.


(Cevap C)
A) 6 B) 7 C) 8 D) 9 E) 10
15. a pozitif tam sayı olmak üzere, 5a3 + 4 sayısının 7 ile
5 + 12! sayısının birler basamağı 5 tir. bölümünden kalan 6 dır.
Bu sayı 30! içinde bulunan çok sayıdaki 2 asal çarpanlardından biriyle çarpıldığında Buna göre, 125a9 + 25a6 + 3 sayısının 7 ile bölümünden
sonuna bir sıfır gelir. Ayrıca 30! sayısının sonunda
kalan kaçtır?
30 5 30 25
6 + 1 = 7 sıfır A) 0 B) 1 C) 2 D) 3 E) 4
6 1 5a3 = 7k + 6 ⇒ 5a3 = 7k + 2
vardır. 30! . (5 + 12!) sayısının sondan 7 + 1 = 8 basamağı sıfırdır.
5a3 sayısının 7 ile bölümünden kalan 2 dır.
(Cevap C)
125a9 + 25a6 + 3 = 25a6 . (5a3 + 1) + 3

25a6 = (5a3)2 sayısının 7 ile bölümünden kalan 22 = 4 ve 5a3 + 1 sayısının 7 ile


bölümünden kalan 3 tür.

25a6 . (5a3 + 1) = 3 sayısının 7 ile bölümünden kalan 4 . 3 + 3 ≡ 1(mod7)


44
12. a ve b pozitif tam sayılar olmak üzere, (Cevap B)

a! 16. a, b ve c asal sayılar olmak üzere,


= 240
b !
a2b + a2c = c2 – b2
olduğuna göre, a nın alabileceği en küçük değer kaçtır?
dir. b ve c iki basamaklı olduklarına göre, a + b + c
A) 6 B) 14 C) 15 D) 16 E) 24 toplamının alabileceği en küçük değer kaçtır?

A) 30 B) 32 C) 35 D) 37 E) 40
a! = 240 . b! ⇒ a! = 15 . 16 . b!
a2(b + c) = (c – b) (c + b) ⇒ a2 = c – b
b = 14 ve a = 16 bulunur.
c – b çift sayı olduğundan a = 2 dir.
(Cevap D) c–b=4⇒c=b+4

b = 13 için c = 17 olur.

a + b + c = 2 + 13 + 17 = 32 bulunur.
(Cevap B)
Bölme ve Bölünebilme BÖLÜM 03 Test 02
1. A = 182 + 272 + 362 5. A, B, C pozitif tam sayılar olmak üzere,
olduğuna göre, A sayısının en büyük asal çarpanı kaçtır?
A+2 B B C
A) 23 B) 29 C) 31 D) 37 E) 41 3 6
2 2 2 2 4
A = 9 (2 + 3 + 4 ) ⇒ A = 3 . 29
4 3
(Cevap B)

bölme işlemleri veriliyor.

Buna göre, A nın alabileceği en küçük değer kaçtır?

A) 81 B) 83 C) 84 D) 85 E) 86
Bölen kalandan büyük olacağı için C nin alabileceği en küçük değer 4 tür.

B = 6C + 3 ⇒ B = 27 bulunur

A + 2 = 3B + 4 ⇒ A + 2 = 81 + 4

⇒ A = 83 tür.
2. 3ab4 dört basamaklı bir doğal sayıdır. (Cevap B)
3ab4
12

ifadesinin alabileceği en büyük tam sayı değerleri ile en


küçük tam sayı değerleri arasındaki fark kaçtır?

A) 75 B) 80 C) 120 D) 800 E) 960 6. Dört basamaklı (3a7b) sayısı 45 ile tam bölünebilen tek
3ab4 sayısı 4 ve 3 ile tam bölünebilmelidir. sayı olduğuna göre, a rakamı kaçtır?
b = 0 → a = 2, 5, 8 A) 3 B) 4 C) 5 D) 6 E) 8
b = 2 → = 0, 3, 6, 9 b = 5 tir. 3 + a + 7 + 5 = 9k (k∈Z)
Çözüm Yayınları

b = 4 → = 1, 4, 7 a = 3 bulunur.

b = 6 → a = 2, 5, 8 (Cevap A)

b = 8 → a = 0, 3, 6, 9
3ab4
ifadesinin alabileceği en büyük tam sayı değeri ile en küçük tam sayı değeri
12
3984 3024
arasındaki fark – = 80
12 12 (Cevap B)

3. 72 sayısını tam bölebilen farklı doğal sayıların toplamı


kaçtır? 7. Beş basamaklı 72M5N sayısı 6 ile tam bölünebildiğine
göre, M + N toplamının alabileceği en büyük değer kaçtır?
A) 190 B) 195 C) 200 D) 205 E) 210
1 + 2+ 3 + 4 + 6 + 8 + 9 + 12 + 18 + 24 + 36 + 72 = 195 A) 13 B) 14 C) 15 D) 16 E) 17
(Cevap B) 72M5N sayısı 3 ile tam bölünebilen çift sayıdır. N = 8 için M = 8 olur.

M + N = 16 dır.

(Cevap D)

45
4. ABC3 dört basamaklı, DE iki basamaklı doğal sayılardır.
8. A = (29!)2 – 1
ABC3 35
olduğuna göre, A nın sondan kaç basamağı 9 dur?
...

DE A) 6 B) 8 C) 9 D) 10 E) 12
29 5 29 25
5+1=6
Yukarıdaki bölme işlemine göre, DE nin alabileceği farklı 5 1
değerlerin toplamı kaçtır? 29! sayısının sondan 6 basamağı sıfırdır.

A) 97 B) 102 C) 115 D) 128 E) 133 (29!)2 sayısının sondan 12 basamağı sıfır olur.
Kalanın birler basamğı 3 veya 8 olur.
29! – 1 sayısının sondan 12 basamağı 9 dur.
13 + 18 + 23 + 28 + 33 = 115
(Cevap C) (Cevap E)
Test 02 1. B 2. E 3. B 4. C 5. B 6. A 7. D 8. E 9. C 10. C 11. D 12. E 13. E 14. E 15. B 16. B

9. x ve y doğal sayılar olmak üzere, 13. a = 20 + 20 + 20 + . . . + 20


9x2 – y2 = 23 62 tane
olduğuna göre, x + y toplamı kaçtır? b = 14 + 14 + 14 + . . . + 14
A) 13 B) 14 C) 15 D) 16 E) 17
73 tane
9x2 – y2 = 23 ⇒ (3x – y) (3x + y) = 1 . 23
3x – y = 1 olduğuna göre, a · b nin 9 ile bölümünden kalan kaçtır?
x = 4 ve y = 11 bulunur.
3x + y = 23 A) 4 B) 5 C) 6 D) 7 E) 8
a nın 9 ile bölümünden kalan 20 . 62 → 2 . 8 → 7 ve b nin 9 ile bölümünden kalan
x + y = 4 + 11 = 15 tir.
14 . 73 → 5 . 1 → 5 tir.
(Cevap C)
A . B nin 9 ile bölümünden kalan 7 . 5 → 8 olur.

(Cevap E)

10. a nın kaç farklı doğal sayı değeri için


3a − 21
a + 1

işleminin sonucu doğal sayı olur?


14. n ∈ Z+ ve 74AB dört basamaklı bir sayıdır.
A) 1 B) 2 C) 3 D) 6 E) 8
Bir satıcı, tanesi 45 liradan 2n tane gömlek satın alıyor.
3a + 21 a+1 3a – 21 24
=3–
3a + 3 3 a+1 a+1 Bu gömlekler için 74AB lira ödediğine göre, satıcı kaç
–24 gömlek satın almıştır?

a + 1 sayısı 24 ün pozitif bölenleri olmalıdır. A) 84 B) 108 C) 120 D) 144 E) 166


2n . 45 = n . 90 → B = 0 dır.
Çözüm Yayınları

a+1=8⇒a=7
74A0 sayısı 9 ile tam bölünür.
a + 1 = 12 ⇒ a = 11
A = 7 olur.
a + 1 = 24 ⇒ a = 23
7470 : 45 = 166
için ifade doğal sayı olur. (Cevap E)

(Cevap C)
11. a ve b pozitif tam sayılar olmak üzere,
a3 = 12 · b4
15. Ahmet, Bora ve Cemil yaşlarını, verilen bir doğal sayıya
olduğuna göre, a + b toplamının alabileceği en küçük bölerek sırasıyla 6, 7 ve 9 sayılarını bulmuşlardır.
değer kaçtır?
Ahmet 20 yaşından küçük, Bora 20 yaşından büyük
A) 72 B) 96 C) 108 D) 126 E) 136 olduğuna göre, Cemil kaç yaşındadır?
a 3 = 22 . 3 . b 4 ⇒ a 3 = 22 . 3 . b . b 3
A) 24 B) 27 C) 30 D) 33 E) 36
b = 2 . 32 = 18 ve Verilen doğal sayı k olsun.

a3 = 23 . 33 . 183 ⇒ a = 2 . 3 . 18 Ahmet = 6k, Bora = 7k ve Cemil = 9k olur.


6k < 20
⇒ a = 108 bulunur. ⇒ k = 3 tür.
7k > 20
a + b = 108 + 18 = 126
(Cevap D) Cemil = 9k = 27 yaşındadır.
46 (Cevap B)

12. 4200 sayısının pozitif bölenlerinden kaç tanesi 14 ile tam


bölünür?
16. 11 e tam bölünemeyen pozitif tam sayılar küçükten büyüğe
A) 14 B) 15 C) 16 D) 17 E) 18 doğru sıralanıyor.
4200 = 42 . 100
Bu sıralamada 50. sayı kaçtır?
= 2 . 3 . 7 . 2 2 . 52
A) 50 B) 54 C) 56 D) 58 E) 60
= 23 . 3 . 52 . 7 n
n– = 50 ⇒ 10n = 50 . 11
11
= 14 . (22 . 3 . 52)
⇒ n = 55
22 . 3 . 52 sayısının pozitif bölenlerinin sayısı 3 . 2 . 3 = 18 dir.
(Cevap E) Atılan son sayı 55 olduğundan 50. sayı, son sayı olan 54 tür.
(Cevap B)
Bölme ve Bölünebilme BÖLÜM 03 Test 03
1. 1112 + 2222 + 3332 5. Altı basamaklı 1AB284 sayısı 66 ile tam bölünebilmektedir.
toplamı aşağıdaki sayılardan hangisine tam bölünemez? Buna göre, A + B toplamının alabileceği en büyük değer
kaçtır?
A) 11 B) 111 C) 222 D) 333 E) 666
1112 . (12 + 22 + 32) = 111 . 111 . 2 . 7 A) 7 B) 10 C) 12 D) 15 E) 17

= 3 . 37 . 111 . 2 . 7 66 = 2 . 3 . 11 olduğundan sayı 2 ile 3 ve 11 ile tam bölünebilmelidir.

Sayı 11 ile tam bölünemez. 1 + A + B + 2 + 8 + 4 = A + B + 15 olduğundan A + B, 3 ün katı olmalıdır.


(Cevap A)
4 – 8 + 2 – B + A – 1 = A – B – 3, 11 in katı ve A – B = 3 olmalıdır.

A + B = 18 olamaz. (A = 9, B = 9, A – B = 0)

A + B = 15 olur.
(Cevap D)

2. Beş basamaklı 4A54B sayısı 15 ile kalansız bölünebilmektedir.


Buna göre, aşağıdakilerden hangisi kesinlikle bir tam sayı
olamaz?

22 25 33 48 49 6. Üç basamaklı ABC sayısının 9 ile bölümünden kalan


A) B) C) D) E)
A A A A A 4 olduğuna göre, beş basamaklı 8ABC7 sayısının 9 ile
B = 0 veya B = 5 tir. bölümünden kalan kaçtır?

4A540 sayısı 3 katı olmalıdır. A) 0 B) 1 C) 5 D) 7 E) 8


A + B + C = 9k + 4 (k∈Z+)
A rakamı 2, 5, 8 olabilir.
8 + A + B + C + 7 = (9k + 4) + 15
4A545 sayısı 3 ün katı olmalıdır.

A rakamı 0, 3, 6, 9 olabilir. A yerine yazılabilecek rakamların hiçbirine 49 tam bölünmez. = 9k + 19


(Cevap E)
19 un 9 ile bölünenden kalan 1 dir.
Çözüm Yayınları

(Cevap B)

3.
A 14 A+B C
...

...

B B

1 1
7. 360 sayısının pozitif tam sayı bölenlerinden kaç tanesi çift
Yukarıdaki bölme işlemlerinde A, B, C pozitif tam sayılar
sayıdır?
olduğuna göre, C kaçtır?
A) 12 B) 15 C) 18 D) 20 E) 24
A) 12 B) 13 C) 14 D) 15 E) 16
Bölenlerin her birinde 2 çarpanı bulunmalıdır.
A = 14 . B + 1
36 . 10 = 23 . 32 . 5
A + B = BC + 1 ⇒ (14 . B + 1) + B = BC + 1
= 2(22 . 32 . 51)
⇒ 15B = BC
22 . 32 . 51 sayısının pozitif bölünlerinin sayısı (2 + 1) . (2 + 1) . (1 + 1) = 18 dir.
⇒ C = 15 bulunur. (Cevap C)
(Cevap D)

8. Beş basamaklı AB248 sayısının 5 ve 9 ile bölümünden


kalanlar aynıdır.
4. A = 215 – 75 47
Buna göre, beş basamaklı 2A4B8 sayısının 11 ile
olduğuna göre, A sayısının kaç tane asal olmayan doğal bölümünden elde edilebilecek kalanların toplamı kaç
sayı böleni vardır? olur?

A) 31 B) 33 C) 35 D) 37 E) 39 A) 6 B) 9 C) 10 D) 12 E) 16
A = 215 – 75 ⇒ A = 75(35 – 1) 5 ile bölümünden kalan 3 olduğundan 9 ile bölümünden kalan da 3 olmalıdır.

⇒ A = 75 . 242 A + B + 2 + 4 + 8 = 9k + 3 (k∈Z+)

⇒ A = 21 . 112 . 75 A + B = 9k – 11 olur.

A sayısının (1 + 1) (2 + 1) (5 + 1) = 36 tane doğal sayı böleni vardır. Bunlardan 3 tanesi 2A4B8 sayısının 11 ile bölümünden kalan 8 – b + 4 – A + 2 = 14 – (A + B) = 14 – (9k – 11)
asal 33 tanesi asal değildir.
(Cevap B) = 25 – 9k

k = 0 için kalan 3, k = 1 için kalan 5 ve k = 2 için kalan 7 olur toplamları 16 dır.


(Cevap E)
Test 03 1. A 2. E 3. D 4. B 5. D 6. B 7. C 8. E 9. C 10. B 11. C 12. B 13. B 14. D 15. D 16. E

9. 13. a, b ∈ Z, a ≠ 0 ve b ≠ 0 olmak üzere,


A 38
108 · a = (1 – b)4

...
x
x2 olduğuna göre, b nin alabileceği en küçük değer kaçtır?

Pozitif tam sayılarla yapılan yukarıdaki bölme işlemine A) –7 B) –5 C) 4 D) 5 E) 7


göre, A nın alabileceği en büyük değer kaçtır? 108 . a = (1 – b)4 ⇒ 22 . 33 . a = (b – 1)4

A) 236 B) 241 C) 264 D) 272 E) 290 a = 22 . 3 = 12 ve


x2 < 38 ⇒ x = 6 (En büyük)
(b – 1)4 = 24 . 34 ⇒ b – 1 = 2 . 3 veya b – 1 = – 2 . 3
A = 38 . x + x2 ⇒ A = 264 bulunur.
(Cevap C) ⇒ b = 7 veya b = –5

bulunur. b nin alabileceği en küçük değer –5 tir.


(Cevap B)

14. 3 x − 78
x−2
ifadesi bir tam sayı olduğuna göre, x in alabileceği farklı
10. 36n sayısının birbirinden farklı tüm doğal sayı bölenlerinin tam sayı değerlerinin toplamı kaçtır?
sayısı 49 olduğuna göre, n doğal sayısı kaçtır?
A) 24 B) 32 C) 36 D) 48 E) 60
A) 2 B) 3 C) 4 D) 5 E) 6
3x – 78 x–2 3x – 78 72
36n = (22 . 32)n = 22n . 32n =3–
3x – 6 3 x+2 x–2

(2n + 1) (2n + 1) = 49 ⇒ 2n + 1 = 7 –72


x–2=–1⇒x=1 x–2=–1⇒x=3
⇒ n = 3 bulunur.
(Cevap B) x–2=–2⇒x=0 x–2=2⇒x=4

x – 2 = – 3 ⇒ x = –1 x–2=3⇒x=5
. .
. .
Çözüm Yayınları

. .
x – 2 = – 72 ⇒ –70 x – 2 =72 ⇒ x = 74
toplamları = 4 . 12 = 48 olur. (Cevap D)

15. 28 !− 27 !
11. a ve b doğal sayılardır. 3n

64 ! = b ifadesi bir tam sayı olduğuna göre, n nin alabileceği kaç


a farklı doğal sayı değeri vardır?
5
A) 14 B) 15 C) 16 D) 17 E) 18
olduğuna göre, a nın alabileceği en büyük değer kaçtır?
27!(28 – 1) 27! . 27 27! . 33
= =
A) 12 B) 13 C) 14 D) 15 E) 16 3n 3n 3n

64 5 64 25 27 3 9 3 3 3
12 2 9 3 1

a nın alabileceği en büyük değer 12 + 2 = 14 tür. n nin alabileceği en büyük değer (n + 3 + 1) + 3 = 16 dır. n nin alabileceği değerler 0, 1, 2,
(Cevap C) ..., 16 olmak üzere 17 tanedir.
(Cevap D)

16. a ve b birer pozitif tam sayı ve p asal sayı olmak üzere,


a2 – 4b2 = p
12. a ve b pozitif tam sayılar olmak üzere, olduğuna göre, a + b toplamının p türünden eşiti
48
aşağıdakilerden hangisidir?
a2 – b2 = 116
2p − 1 2p + 1 2p + 1
olduğuna göre, a kaçtır? A) B) C)
2 2 4
A) 28 B) 30 C) 36 D) 40 E) 58
a–b=2
3p − 1 3p + 1
D) E)
⇒ a = 30 ve b = 28 bulunur. 4 4
a + b = 58
a – 2b = 1 P+1 P–1
a= ve b =
a–b=1 a–b=4 2 4
a + 2b = p
ve olamaz
a + b = 116 a + b = 29 P+1 P–1 3P + 1
Bu durumda a ve b tam sayı olmaz. a+b= + = bulunur.
2 4 4
(Cevap B)
(Cevap E)
Bölme ve Bölünebilme BÖLÜM 03 Test 04
1. Dört basamaklı A35B sayısı 12 ile tam bölünebildiğine 5. 81 · 6n
göre, bölümün alabileceği en büyük değer kaçtır?
sayısının birbirinden farklı tüm doğal sayı bölenlerinin
A) 613 B) 661 C) 696 D) 697 E) 698 sayısı 32 olduğuna göre, n kaçtır?
Son iki basamağı 4 ün katı ve rakamları toplamı 3 ün katı olmalıdır.
A) 2 B) 3 C) 4 D) 5 E) 6
B = 2 veya B = 6 olabilir. 81 . 6n = 34 . 2n . 3n

B = 2 için A nın alabileceği en büyük değer 8 ve B = 6 için A = 4 olur. = 2n . 3n+4 


8352
Buna göre, A35B sayısı en büyük 8352 olur. = 696 dır. (n + 1) . (n + 4 + 1) = 32 ⇒ (n + 1) (n + 5) = 32
12 (Cevap C)
⇒ n = 3 bulunur.
(Cevap B)

2. 220 sayısının a tam sayısına bölümünden kalan 4, 365


sayısının a tam sayısına bölümünden kalan 5 ve 410 6. 239 · 241 + 1
sayısının a tam sayısına bölümünden kalan 14 tür.
sayısının pozitif bölenlerinin sayısı kaçtır?
Buna göre, a nın alabileceği en büyük değer kaçtır?
A) 79 B) 80 C) 81 D) 83 E) 85
A) 24 B) 32 C) 36 D) 48 E) 72 239 . 241 + 1 = (240 – 1) (240 + 1) + 1
EBOB(216, 360, 396) = 36
(Cevap C) = 2402 – 1 + 1

= 2402

= 28 . 32 . 52
Çözüm Yayınları

Pozitif bölenlerinin sayısı, üsler 1 artırılarak çarpılırsa 9 . 3 . 3 = 81 bulunur.


(Cevap C)

3. Kenar uzunlukları tam sayı ve alanı 84 birimkare olan kaç


farklı dikdörtgen çizilebilir?

A) 6 B) 8 C) 9 D) 10 E) 12
84 ün doğal sayı olan bölenlerinin sayısı, 84 = 22 . 31 . 71 olduğundan 7. a ve b aralarında asal iki doğal sayı olmak üzere,
(2 + 1) . (1 + 1) . (1 + 1) = 12 dir.  12 − 12  : a − 1 = 15
 
12  a  b
gibi iki dikdörtgenin alanı aynı olduğundan =6
2
2 42
farklı dikdörtgen çizilebilir. olduğuna göre, a + b toplamı kaçtır?

42 (Cevap A) A) 7 B) 9 C) 11 D) 13 E) 15
2 12a – 12 . b 12(a – 1) . b
= 15 ⇒ = 15
a a–1 a a–1
12b
⇒ = 15
a
4b
⇒ =5
a
⇒ 5a = 4b
4. Üç basamaklı (7ab) sayısı iki basamaklı (ab) sayısının 21
katıdır. a = 4, b = 5 ve a + b = 9 dur. 49
(Cevap B)
Buna göre, (ab) sayısı aşağıdaki sayılardan hangisin tam
bölünür?

A) 2 B) 3 C) 4 D) 5 E) 11
8. Dört basamaklı 7a6b sayısı 30 ile kalansız bölünebildiğine
7ab = 21 . ab ⇒ 700 + ab = 21 . ab göre, bölümün en büyük değeri kaçtır?
⇒ 700 = 20 . ab A) 242 B) 248 C) 252 D) 262 E) 268
⇒ ab = 35 bulunur. Sayı 10 un ve 3 ün katı olmalıdır.

Bu sayı 5 ile tam bölünebilir. b = 0 ve sayı en büyük 7860 olur.


7860
(Cevap D) = 262 dir. (Cevap D)
30
Test 04 1. C 2. C 3. A 4. D 5. B 6. C 7. B 8. D 9. C 10. D 11. C 12. D 13. B 14. C 15. D 16. C

9. a, b ve c pozitif tam sayılardır. 13. a nın kaç farklı doğal sayı değeri için
a + 26
a 6 c b
ve a + 2
c b–2
ifadesi doğal sayı olur?
b 3
A) 6 B) 7 C) 8 D) 9 E) 10
a + 26 a+2 a + 26 24
olduğuna göre, a nın alabileceği değerler toplamı kaçtır? =1+
a+2 1 a+2 a+2
A) 70 B) 113 C) 183 D) 264 E) 296 –72
3 < b < 6 olmalıdır.
a + 2 sayısı 24 ün pozitif bölenleri olmalıdır.
a = 6c + b ve c = b(b – 2) + 3 tür.
a + 2 = 1 ⇒ a = –1 (olamaz)
b = 4 için c = 11 ve a = 70 ve a+2=2⇒a=0

b = 5 için c = 18 ve a = 113 bulunur. a+2=3⇒a=1


.
.
a nın alabileceği değerlerin toplamı .
a + 2 = 24 ⇒ a = 22
70 + 113 = 183 tür.
a nın alabileceği 7 farklı değer vardır.
(Cevap C) (Cevap B)

10. Dört basamaklı 6a7b sayısı 45 ile tam bölünebildiğine


14. a ve b birer doğal sayı olmak üzere,
göre, a rakamı kaç farklı değer alır?
488 · (a – 4)2 = (b – 1)3
A) 0 B) 1 C) 2 D) 3 E) 4
Sayı 5 ve 9 ile bölünebilmelidir. olduğuna göre, a + b toplamının alabileceği en küçük
değer kaçtır?
b = 5 için a = 0 veya a = 9 olabilir.
A) 65 B) 123 C) 188 D) 215 E) 260
b = 0 için a = 5 olur. a rakamı 3 farklı değer alabilir.
23 . 61 . (a – 4)2 = (b – 1)3
Çözüm Yayınları

(Cevap D)
a – 4 = 61 ⇒ a = 65 ve

(b – 1)3 = 23 . 613 ⇒ b – 1 = 2 . 61

⇒ b = 123 bulunur.

a + b = 188 dir.

11. a ve b birer pozitif tam sayı olmak üzere, (Cevap C)

35! = 3a ·b 15. x, y, z, t birer doğal sayı olmak üzere,

olduğuna göre, a nın alabileceği en büyük değer kaçtır? 13 !+ 14 !


x y z t
2 ⋅ 3 ⋅ 5 ⋅ 7
A) 12 B) 14 C) 15 D) 16 E) 18
ifadesinin alabileceği en küçük tam sayı değeri kaçtır?
35 3 11 3 3 3
11 3 1 A) 91 B) 117 C) 132 D) 143 E) 182
13!(1 + 14) 13!15
bölümler toplanır ve a = 11 + 3 + 1 = 15 bulunur. =
2x . 3y . 5z . 7t 2x . 3y . 5z . 7t
(Cevap C) Elde edilen rasyonel ifadede 2 nin, 3 ün, 5 in ve 7 nin tüm kuvvetleri sadeleştirilirse,
sonuç 11.13 = 143 olur.
(Cevap D)

16. AB8 üç basamaklı doğal sayıdır.


50 12. a ve b doğal sayıları için
AB8 25
a2 – b2 = 41
...

dir. x

Buna göre, a sayısı aşağıdakilerden hangisine tam
Yukarıdaki bölme işlemine göre, farklı x kalanlarının
bölünür?
toplamı kaç olur?
A) 4 B) 5 C) 6 D) 7 E) 9
A) 24 B) 41 C) 65 D) 150 E) 300
(a – b) (a + b) = 1 . 41
Bir sayının 25 ile bölümünden kalan, son iki basamağının 25 ile bölümünden kalandır.
a–b=1
⇒ a = 21 ve b = 20 dir. B = 0 için x = 8
a + b = 41 B = 1 için x = 18
a sayısı 7 ile tam bölünür. B = 2 için x = 3

(Cevap D) B = 3 için x = 13
B = 4 için x = 23 olur. Toplamları 65 tir.
(Cevap C)
Bölme ve Bölünebilme BÖLÜM 03 Test 05
1. a, b, c ∈ Z+ olmak üzere, 5. Boyutları 8 cm, 12 cm ve 16 cm olan dikdörtgenler
prizması şeklindeki kutulardan en az kaç tanesi ile içi
a · b = 45
dolu bir küp yapılabilir?
b · c = 75
A) 40 B) 56 C) 60 D) 72 E) 96
olduğuna göre, a · b · c çarpımının alabileceği en küçük EKOK(8, 12, 16) = 48
değer kaçtır? 48 . 48 . 48
= 6 . 4 . 3 = 72
8 . 12 . 16
A) 180 B) 196 C) 216 D) 225 E) 240 (Cevap D)
EBOB(45, 75) = 15 tir.

b = 15, a = 3 ve c = 5 bulunur.

15 . 3 . 5 = 225 elde edilir.

(Cevap D)

6. Bir torbada 150 den fazla ve 250 den az sayıda bilye vardır.
2. Çarpımları 720 olan iki doğal sayının ortak bölenlerinin en Torbadaki bilyeler beşer, altışar ve sekizer sayıldığında daima
büyüğü 6 dır. 3 bilye artmaktadır.

Buna göre, bu iki sayının ortak katlarının en küçüğü Buna göre, torbadaki bilyelerin sayısı kaçtır?
kaçtır?
A) 234 B) 237 C) 240 D) 243 E) 245
A) 96 B) 120 C) 180 D) 240 E) 360 x = 5a + 3 = 6b + 3 = 8c + 3

EBOB(a, b) . EKOK(a, b) = a . b x – 3 = OKEK(5, 6, 8) = 120

6. EKOK(a, b) = 720 x = 120k + 3, k∈Z+

EKOK(a, b) = 120 k = 2 için bilye sayısı 243 olur.


Çözüm Yayınları

(Cevap B) (Cevap D)

3. Dikdörtgenler prizması biçimindeki bir kutunun boyutları


48 cm, 72 cm ve 84 cm dir. Bu kutu, küp biçimindeki özdeş 7. 6 < a ≤ 36 olmak üzere,
kutularla doldurulacaktır.
EBOB(a, 18) = 6
Buna göre, en az kaç tane kutu ile dolar?
dır.
A) 160 B) 168 C) 174 D) 180 E) 196
Buna göre, a nın alabileceği tam sayı değerlerinin toplamı
EBOB(48, 72, 84) = 12
kaçtır?
48 . 72 . 84
= 4 . 6 . 7 = 168
12 . 12 . 12 A) 102 B) 72 C) 66 D) 54 E) 42
(Cevap B)
a = 6k ve a ≠ 18t (k, t∈Z+)

a = 12, a = 24, a = 30 olabilir.

toplamları 12 + 24 + 30 = 66 dır.

(Cevap C)
4. 20 m, 24 m ve 28 m uzunluğundaki üç demir çubuk kesilerek
birbirine eşit ve en büyük boyda parçalara ayrılacaktır. 51

Buna göre, kaç kesim yapmak gerekir?

A) 9 B) 12 C) 15 D) 16 E) 18

EBOB(20, 24, 28) = 4


8. Ali ile Berk bir miktar cevizi dörder dörder paylaştıklarında 1
Bir çubuğu n tane kesim n + 1 tane parçaya ayarır. ceviz, altışar altışar paylaştıklarında ise 5 ceviz artıyor.
20 Paylaşılan ceviz sayısı 100 den çok olduğuna göre,
= 5 parça 4 kesim,
4
24 en az kaçtır?
= 6 parça 5 kesim ve
4
28 A) 113 B) 114 C) 115 D) 166 E) 117
= 7 parça 6 kesim olmak x = 8a + 1 = 12b + 5 ⇒ x + 7 = 8(a + 1) = 12(b + 1)
4
üzere, 4 + 5 + 6 = 15 kesim yapmak gerekir. x + 7 = k . EKOK(8, 12) ⇒ x + 7 = k . 24

(Cevap C) k = 5 için x + 7 = 120 ⇒ x = 113 bulunur.

(Cevap A)
Test 05 1. D 2. B 3. B 4. C 5. D 6. D 7. C 8. A 9. B 10. B 11. B 12. A 13. B 14. E 15. E 16. C

9. Üç zilden birincisi 40 dakika bir, ikincisi 60 dakikada bir, 13. Komşu iki kenarının uzunlukları 168 metre ve 180 metre olan
üçüncüsü de 90 dakikada bir çalmaktadır. dikdörtgen şeklindeki bir tarla, alanları eşit kare şeklinde
parsellere ayrılarak karelerin köşelerine birer fidan dikilecektir.
Saat 14 te birlikte çalan bu üç zil, saat kaçta tekrar ilk kez
birlikte çalar? Buna göre, en az kaç fidana ihtiyaç vardır?

A) 19 B) 20 C) 21 D) 22 E) 23 A) 210 B) 240 C) 250 D) 260 E) 280


EKOK(40, 60, 90) = 360 168 = 23 . 3 . 7
360 EBOB(168, 180) = 12
= 6 saat sonra, yani saat 20 de tekrar birlikte çalar. 180 = 22 . 32 . 5
60
(Cevap B)
168
= 14 ara, 15 sıra ve
12
180
= 15 ara, 16 sıra fidan dikilecektir. 15 . 16 = 240 fidana ihtiyaç vardır.
12
(Cevap B)

10. a ve b birer pozitif tam sayı olmak üzere, 14. Eni 1240 cm, boyu 1800 cm ve derinliği 400 cm olan
a + a + a =b dikdörtgenler prizması biçiminde üstü açık bir yüzme
12 15 18 havuzunun iç yüzeyleri, kare biçimindeki eş fayanslarla
kaplanacaktır.
olduğuna göre, b nin alabileceği en küçük değer kaçtır?
Buna göre, bu iş için en az kaç fayansa ihtiyaç vardır?
A) 36 B) 37 C) 38 D) 39 E) 40
a a a 37a
A) 2600 B) 2740 C) 2760 D) 2800 E) 2915
+ + =b⇒ =b EBOB = 40
12 12 12 180
Taban çevresi x 400 + 1800 . 1240
(15) (12) (10)
40 . 40
a = 180 için b = 37 bulunur. 2 . 3040 . 400 1800 . 1260
Çözüm Yayınları

= + = 1420 + 1395
(Cevap B) 40 . 40 40 . 40
= 2815

(Cevap E)

15. 9x + 6y = EBOB(9, 6)
11. Üç ayrı televizyon kanalından birincisinde 40 dakikada bir,
eşitliğini sağlayan x ve y tam sayıları için, (x, y) ikilisi
ikincisinde 48 dakikada bir, üçüncüsünde ise 60 dakikada bir
aşağıdakilerden hangisi olamaz?
reklam yayımlanmaktadır.
A) (1, –1) B) (3, –4) C) (5, –7)
Saat 15:00 te aynı anda reklam yayımlayan bu üç kanal,
saat kaçta tekrar birlikte reklam yayımlar? D) (7, –10) E) (10, –14)
3x + 2y = 1 denklemi x = 1 ve y = –1 için sağlanır.
A) 18:30 B) 19:00 C) 19:30
3x + 2y = 3 . 1 + 2 . (–1) ⇒ 3x – 3 = –2y – 2
D) 20:00 E) 20:30
⇒ 3(x – 1) = 2(–y – 1)
EKOK(40, 68, 60) = 24 . 3 . 5 = 240
x–1 –y – 1
240 = = t ⇒ x = 1 + 2t, y = –1 – 3t
= 4 saat sonra saat 19:00 da 2 3
60
(Cevap B) (x, y) ⇒ (1, –1), (3, –4), (5, –7), (7, –10), ...

(10, – 14) ikilisi denklemi sağlamaz.

52 (Cevap E)

16. a ve b birer doğal sayı olmak üzere,


12. a ve b ardışık iki doğal sayı olmak üzere,
EBOB(a, b) = 3
EBOB(a, b) = x – 11
12 · EKOK(a, b) = 35 · (a + b)
EKOK(a, b) = 9x + 2
olduğuna göre, a + b toplamı kaçtır?
olduğuna göre, bu sayılardan küçük olan kaçtır?
A) 32 B) 35 C) 36 D) 45 E) 48
A) 10 B) 12 C) 15 D) 18 E) 20 a = 3k ve b = 3t, k, t∈Z+ olsun.
Ardışık iki doğal sayı aralarında asal sayılardır. En büyük ortak bölenleri 1 ve en küçük 12. EKOK(a,b) = 35 . (a + b) ⇒ 12. (k . t . 3) = 35 . 3(k + t)
ortak katları bu iki sayının çarpımı olur.
⇒ 12 . k . t = 35(k + t)
EBOB(a, b) = 1 ⇒ x – 11 = 1 k . t = 35
⇒ x = 12 ⇒ k = 5 ve t = 7
k + t = 12
EKOK(a, b) = x – 11) . (9x + 2) = 1 . 110 = 10 . 11 küçük sayı 10 dur. a = 15, b = 21 ve a + b = 36 bulunur.
(Cevap A) (Cevap C)
Bölme ve Bölünebilme BÖLÜM 03 Test 06
1. Hem 48 hem de 60 ile tam bölünebilen üç basamaklı kaç 5. 185, 423 ve 482 sayıları, aynı a doğal sayısına bölündüğünde
doğal sayı vardır? sırasıyla 5, 3 ve 2 kalanlarını vermektedir.

A) 2 B) 3 C) 4 D) 5 E) 6 Bu koşula uyan en büyük a sayısı kaçtır?


EKOK(48, 60) = 240
A) 36 B) 48 C) 60 D) 72 E) 90
240 sayısının katları olan üç basamaklı sayılar 240, 480, 720, 960 olmak üzere 4 tanedir. EBOB(180, 420, 480) = 60
(Cevap C) (Cevap C)

3 , 4 ve 8
6. sayılarına tam bölünebilen en küçük
19 7 38
pozitif tam sayının pozitif bölenlerinin toplamı kaçtır?
2. İki doğal sayının ortak bölenlerinin en büyüğü 12 ve ortak A) 21 B) 24 C) 25 D) 28 E) 30
katlarının en küçüğü 420 dir. Verilen sayılara tam bölünebilen en küçük doğal sayı x olsun.
3 19x 4 7x 8 19x
Bu sayılardan biri 60 olduğuna göre, diğeri kaçtır? x: = , x: = ,x: =
19 3 7 4 38 4
A) 72 B) 84 C) 90 D) 96 E) 108 olduğundan x sayısı 3 ve 4 ün en küçük ortak katı, yani 12 olmalıdır.
60 . x = 12 . 420 ⇒ x = 84
12 sayısının pozitif bölenlerinin toplamı 1 + 2 + 3 + 4 + 6 + 12 = 28 bulunur.
(Cevap B)
(Cevap D)
Çözüm Yayınları

7. Bir terzi 3 günde 4 gömlek, kalfası ise 4 günde 3 gömlek


dikebilmektedir.

İkisi birlikte 100 gömleği bitirdiğinde, bunlardan kaç


tanesini kalfa dikmiş olur?

A) 24 B) 30 C) 36 D) 42 E) 45
3. Tanesi 75 liradan bir miktar pantolon ve tanesi 35 liradan bir Terzi: 3 günde 4 gömlek
miktar kazak alan bir kimse toplam 405 lira ödemiştir. Kalfa: 4 günde 3 gömlek
––––––––––––––––––––––
Buna göre, alınan pantolon sayısı kaçtır? Terzi: 12 günde 16 gömlek

A) 2 B) 3 C) 4 D) 5 E) 6 Kalfa: 12 günde 9 gömlek diker


x tane pantolon ve y tane kazak almış olsun. İkisi birlikte 12 günde 25 gömlek ve 4 . 12 = 48 günde 100 gömlek dikmiş olurlar.

75x + 35y = 405 Bunun 4 . 9 = 36 sını kalfa dikmiş olur.

4 pantolon ve 3 kazak almıştır. (Cevap C)

(Cevap C)

8. Bir dikdörtgenler prizmasının boyutları 42 cm, 54 cm ve


72 cm dir. 53
Bu prizmadan en büyük boyutlu ve eşit hacimli kaç küp
kesilebilir?
4. 24 litrelik, 27 litrelik ve 30 litrelik kaplarda bulunan üç farklı A) 720 B) 750 C) 756 D) 760 E) 780
kalitedeki yağ, birbirine karıştırılmadan eşit hacimli şişelere
EBOB(42, 54, 72) = 6, küpün bir ayrıtı 6 cm olmalıdır.
doldurulacaktır.
Dikdörtgenler prizmasının hacmi 42 . 54 . 72
=
Buna göre, bu iş için en az kaç şişeye ihtiyaç vardır? küpün hacmi 6.6.6
= 7 . 9 . 12
A) 24 B) 27 C) 30 D) 36 E) 45
EBOB(24, 27, 30) = 3 = 756
24 27 30
+ + = 8 + 9 + 10 = 27 (Cevap C)
3 3 3
tane şişeye ihtiyaç vardır.

(Cevap B)
Test 06 1. C 2. B 3. C 4. B 5. C 6. D 7. C 8. C 9. D 10. B 11. C 12. C 13. C 14. D 15. A 16. A

9. 36 kg fasulye, 48 kg nohut, 60 kg bulgur hiç artmayacak ve 13. Dairesel bir yarış pistinin etrafını üç atletten birincisi 60
birbirine karışmayacak şekilde torbalara konacaktır. saniyede, ikincisi 75 saniyede ve üçüncüsü de 90 saniyede
koşmaktadır.
Torbalara konulan ağırlıklar eşit olacağına göre en az kaç
torba gerekir? Başlangıç noktasından aynı anda koşmaya başlayan bu
üç atlet, tekrar başlangıç noktasında ilk kez bir araya
A) 8 B) 9 C) 10 D) 12 E) 15
geldiğinde, en yavaş koşan atlet kaç tur atmış olur?
EBOB(36, 48, 60) = 12
36 48 60 A) 6 B) 9 C) 10 D) 12 E) 15
= 3 torba, = 4 torba, = 5 torba olmak üzere, toplam 3 + 4 + 5 = 12 torba
12 12 12 EKOK(60, 75, 90) = 900 saniye
gerekir.
900
(Cevap D) = 10 tur.
90
(Cevap C)

10. 6 ve a doğal sayısı aralarında asal sayılar olmak üzere, 14. a, b ve c doğal sayılar olmak üzere,
EKOK(6 · a, a + 6) = 330 3a = 4b = 5c

olduğuna göre, a kaçtır? EBOB(a, b, c) = 4

A) 3 B) 5 C) 7 D) 11 E) 13 olduğuna göre, a + b + c toplamı kaçtır?


Bu sayıların çarpımları ile toplamlarıda aralarında asaldır.
A) 160 B) 172 C) 180 D) 188 E) 192
EKOK(6a, a + 6) = 330 ⇒ 6a . (a + b) = 330 EKOK(a, b, c) = 60 tır.

⇒ a(a + b) = 55 3a = 4b = 5c = 60k olsun.

⇒ a = 5 bulunur. a = 20k, b = 15k, c = 12k olur.

(Cevap B) EBOB(a, b, c) = 4 olduğundan


Çözüm Yayınları

k = 4 alınırsa a = 80, b = 60 ve

c = 48 olur. a + b + c = 80 + 60 + 48 = 188 dir.


(Cevap D)

11. a sayısı 105, 135 ve 165 sayılarını kalansız bölen bir doğal 15. 87x + 45y = EBOB(87, 45)
sayıdır.
eşitliğini sağlayan x ve y tam sayıları için x + y toplamı
a + 90 aşağıdakilerden hangisi olamaz?
Buna göre, ifadesinin en küçük değeri kaçtır?
a
A) –26 B) –13 C) 1 D) 15 E) 29
A) 5 B) 6 C) 7 D) 8 E) 9 87x + 45y = 3 ⇒ 29x + 15y = 1
a = EBOB(105, 135, 165) ⇒ a = 15 x = –1 ve y = 2 denklemi sağlar.
a + 90 a 90
= + 29x + 15y = 29(–1) + 15 . 2
a a a
x + 1 –y + 2
=1+6 29(x + 1) = 15(–y + 2) ⇒ = =k
15 29
⇒ x = –1 + 15k, y = 2 – 29k
=7
x + y = 1 – 14k, k∈Z+ bulunur.
(Cevap C)
k = –2 için x + y = 29
k = –1 için x + y = 25
k = 0 için x + y = 1
12. a, b ∈ Z+ için k = 1 için x + y = –13 olur. x + y = –26 olamaz.
54 (Cevap A)
EBOB(a, b) = 6

a2 – b2 = 612 16. a ve b ardışık tek doğal sayılar olmak üzere,


olduğuna göre, a kaçtır? EKOK(a, b) + 1 = 7(a + b)
A) 36 B) 48 C) 54 D) 60 E) 72 olduğuna göre, a + b toplamı kaçtır?
EBOB(a, b) = 6 ⇒ a = 6k ve b = 6t, k, t∈Z+ olsun.
A) 28 B) 30 C) 32 D) 34 E) 36
a2 – b2 = 612 ⇒ 36k2 – 36t2 = 612 a ve b aralarında asal sayılar ve b = a + 2 dir.
a . (a + 2) + 1 = 7(a + a + 2)
⇒ k2 – t2 = 17
a2 + 2a + 1 = 14a + 14
⇒ (k – t) (k + t) = 1 . 17
k–t=1 a2 – 1a – 13 = 0 ⇒ (a – 13) (a + 1) = 0
⇒ k = 9 ve t = 8 bulunur.
k + t = 17 ⇒ a = 13
a = 6k = 54 tür. b = a + 2 = 15 bulunur. a + b = 28 dir.
(Cevap A)
(Cevap C)
Modüler Aritmetikte İşlemler BÖLÜM 03 Test 07
1. (4862)27 5. Aşağıdakilerden hangisi
sayısının 9 ile bölümünden kalan kaçtır? 7x ≡ 2x (mod 13)

A) 4 B) 5 C) 6 D) 7 E) 8 denkliğinin bir kökü değildir?


21 ≡ 2 (mod 9) 24 . 6 + 3 ≡ 1 . 23 ≡ 8 (mod 9)
A) 12 B) 24 C) 30 D) 36 E) 48
22 ≡ 4 (mod 9)
12 ≡ 1 (mod 13)
23 ≡ 8 (mod 9)
x = 12, x = 24, x = 36, x = 48, ...
24 ≡ 7 (mod 9)
için 7x ≡ 2x ≡ 1 olur.
5
2 ≡ 5 (mod 9)
x = 30 olamaz.
26 ≡ 1 (mod 9) (Cevap C)
(Cevap E)

2. 443 + 743 + 1043 6. Aynı hastanede çalışan Hemşire Fatma 4 günde bir, Doktor
toplamının 11 ile bölünden kalan kaçtır? Ahmet 6 günde bir nöbet tutmaktadır.

A) 6 B) 7 C) 8 D) 9 E) 10 Bu iki görevli birinci nöbetlerini pazartesi günü birlikte


tutuğuna göre, altıncı nöbetlerini hangi gün birlikte tutar?
7 ≡ –4 (mod 11)
743 ≡ (–4)43 (mod 11) A) Salı B) Çarşamba C) Perşembe
443 + (–4)43 + 1043 ≡ 443 – 443 + 1043 (mod 11) D) Cuma E) Cumartesi
10 = 10 (mod 11) EKOK(4, 6) = 12 olduğundan 12 nin katlarında birlikte nöbet tutarlar.
1. 2. 3. 4. 5. 6.
102 = 1 (mod 11) . . . . . .
Çözüm Yayınları

12 12 12 12 12
(10)2 . 21 + 1 = 10 (mod 11)
6 nöbetlerini 5 . 12 = 60 gün sonra tutarlar
(Cevap E)
60 ≡ 4 (mod 7)
Pazartesi gününden 4 gün sonra cuma günü olur.
(Cevap D)

3. 4240 + 7300
sayısının 6 ile bölümünden kalan kaçtır? 7. 200x ≡ 1 (mod 19)
A) 1 B) 2 C) 3 D) 4 E) 5 denkliğini sağlayan en küçük pozitif tam sayı kaçtır?
41 ≡ 4 (mod 6) 71 ≡ 1 (mod 6)
.
42 ≡ 4 (mod 6) . A) 10 B) 12 C) 15 D) 18 E) 21
. . m asal sayı, a ve m aralarında asal ise
.
. 7300 ≡ 1 (mod 6) am – 1 ≡ 1 (mod m) dir.
4240 ≡ 4 (mod 6) Buna göre, x = 18 olur.
4240 + 4300 ≡ 4 + 1 ≡ 5 (mod 6) (Cevap D)

(Cevap E)
55

4. 3x ≡ 4 (mod 10)
denkliğini sağlayan üç basamaklı en küçük x doğal sayısı
kaçtır? 8. 5x ≡ 3 (mod 9)
A) 105 B) 106 C) 107 D) 108 E) 109 denkliğini sağlayan en küçük x doğal sayısı kaçtır?
7/ 3x ≡ 4 (mod 10)
A) 4 B) 5 C) 6 D) 7 E) 8
21 . x ≡ 7 . 4 (mod 10)
5x ≡ 3 (mod 9) ⇒ 2 . 5x ≡ 2 . 3 (mod9)
x ≡ 8 (mod 10)
⇒ x ≡ 6 (mod 9)
x = 8 + 10k
(Cevap C)
k = 10 için x = 108 bulunur.
(Cevap D)
Test 07 1. E 2. E 3. E 4. D 5. C 6. D 7. D 8. C 9. B 10. B 11. E 12. C 13. C 14. C 15. A 16. B

9. 723 13. a tam sayı, p asal sayı ve p, a nın tam böleni değilse
sayısının birler basamağındaki rakamı aşağıdakilerden ap–1 ≡ 1 (mod p)
hangisidir?
dir. (Fermat Teoremi)
A) 1 B) 3 C) 5 D) 7 E) 9
Buna göre, 12162 sayısının 17 ile bölümünden kalan
71 ≡ 7 (mod 10)
kaçtır?
72 ≡ 9 (mod 10)
A) 4 B) 7 C) 8 D) 12 E) 15
73 ≡ 3 (mod 10)
1216 ≡ 1 (mod 17)
74 ≡ 1 (mod 10)
(1216)10 ≡ 1 (mod 17)
723 ≡ 74 . 5 + 3 ≡ 73 ⇒ 3 (mod 10)
12160 ≡ 1 (mod 17)
(Cevap B)
12162 ≡ 12160 . 122 ≡ 1 . 8 ≡ 8 (mod 17)

(Cevap C)

10. m pozitif tam sayı olmak üzere,


63 ≡ 15 (mod m) 14. x = 521769
olduğuna göre, m nin alabileceği değerlerin toplamı olduğuna göre, x23 sayısının 11 ile bölümünden kalan
kaçtır? kaçtır?

A) 120 B) 124 C) 127 D) 132 E) 144 A) 3 B) 5 C) 7 D) 9 E) 10


63 ≡ 15 (mod m) ⇒ 63 – 15 ≡ 0 (modm) x sayısının 11 ile bölümünden kalan
⇒ 48 ≡ 0 (mod m) 9 – 6 + 7 – 1 + 2 – 5 = 6 dır.
m sayısı 48 in poziti bölenleri olmalıdır. x ≡ 6 (mod 11) x10 ≡ 1 (mod 11)
Çözüm Yayınları

Toplamları 1 + 2 + 3 + 4 + 6 + 8 + 12 + 16 + 24 + 48 = 124 olur. x2 ≡ 3 (mod 11) x23 ≡ (x10) . x3 ≡ 1 . 7 (mod 11)


(Cevap B) x3 ≡ 7 (mod 11) x23 ≡ 7 (mod 11)

(Cevap C)

15. 723!
11. 6x ≡ 9 (mod 15)
sayısının 13 ile bölümünden kalan kaçtır?
denkliğini sağlayan üç basamaklı en küçük x doğal sayısı
A) 1 B) 4 C) 6 D) 9 E) 12
kaçtır?
712 ≡ 1 (mod 13) tür.
A) 100 B) 101 C) 102 D) 103 E) 104
23! içinde 12 çarpanı bulunduğundan
3/ 2x ≡ 3 (mod 5) ⇒ 3 . 2x ≡ 3 . 3 (mod5)
723! ≡ 1 olur.
⇒ x ≡ 4 (mod 5)
x doğal sayısı en küçük 104 olur. (Cevap A)
(Cevap E) 16. AB iki basamaklı bir doğal sayıdır.
Buna göre,

AB ≡ 4 (mod 7)
56
AB ≡ 2 (mod 5)

sistemini sağlayan kaç farklı AB sayısı vardır?

A) 1 B) 2 C) 3 D) 4 E) 5
5 . AB ≡ 20 (mod 35)
12. x3 ≡ 9 (mod 11)
7 . AB ≡ 14 (mod 35)
x4 ≡ 3 (mod 11) İkinci denklikten birinci denkliği taraf tarafa çıkaralım.

olduğuna göre, x21 in 11 ile bölümünden kalan kaçtır? 2 . AB ≡ –6 (mod 35)


AB ≡ –3 (mod 35)
A) 2 B) 3 C) 4 D) 5 E) 6
AB ≡ –3 + 35 ≡ 32 (mod 35)
x3. x4 ≡ 9 . 3 (mod 11) ⇒ x7 ≡ 5 (mod 11)
AB ≡ 32 + 35 ≡ 67 (mod 35)
⇒ (x7)3 ≡ 53 (mod 11)
Sistemi sağlayan iki farklı AB sayısı vardır.
⇒ x21 ≡ 4 (mod 11)
(Cevap B)
(Cevap C)
Modüler Aritmetikte İşlemler BÖLÜM 03 Test 08
1. A = 25 + 35 + 45 + 55 + 65 5. Aşağıdakilerden hangisi
olduğuna göre, A sayısının 7 ile bölümünden kalan 63x ≡ 4 (mod 5)
kaçtır?
denkliğini sağlayan x doğal sayılarından biri değildir?
A) 1 B) 3 C) 4 D) 5 E) 6
A) 6 B) 8 C) 10 D) 14 E) 18
5 ≡ –2 (mod 7)
63 ≡ 3 (mod 5) n = 1 için x = 6
4 ≡ –3 (mod 7) dir.
31 ≡ 3 (mod 5) n = 2 için x = 10
25 + 35 + (–3)5 + (–2)5 + 65 ≡ 65 (mod 7)
32 ≡ 4 (mod 5) n = 3 için x = 14
61 ≡ 6 (mod 7)
3 ≡ 2 (mod 5)
3 n = 4 için x = 18
62 = 1 (mod 7)
34 ≡ 1 (mod 5) bulunur.
(62)2 . 61 ≡ 1 . 6 ≡ 6 (mod 6)
34n+2 ≡ 4 (mod 5) x = 8 olamaz.

(Cevap E) x = 4n + 2 dir.

(Cevap B)

2. x ≡ 4 (mod 5)
6. Pazartesi günü, 17 günlük izne çıkan bir memur haftanın
x ≡ 6 (mod 7)
hangi günü işbaşı yapar?
sistemini sağlayan en küçük x doğal sayısı kaçtır?
A) Pazartesi B) Salı C) Çarşamba
A) 10 B) 12 C) 24 D) 34 E) 35
D) Perşembe E) Cuma
7x ≡ 28 (mod 35)
17 ≡ 3 (mod 7)
5x ≡ 30 (mod 35)
Pazartesi gününden 3 gün sonra Perşembe olur.
2x ≡ –2 (mod 35)
x ≡ –1 (mod 35) (Cevap D)
Çözüm Yayınları

x ≡ –1 + 35 ≡ 34 (mod 35)

(Cevap E)

7. n ∈ Z+ olmak üzere,
3. 3x ≡ 1 (mod 11)
5n – 1
denkliğini sağlayan iki basamaklı x doğal sayılarının
toplamı kaçtır? sayısı 23 ile tam bölünebilmektedir.

A) 395 B) 406 C) 417 D) 428 E) 432 Buna göre, n nin alabileceği en küçük değer kaçtır?
3x ≡ 1 (mod 11) ⇒ 4 . 3x ≡ 4 . 1 (mod 11)
A) 19 B) 20 C) 21 D) 22 E) 23
⇒ x ≡ 4 (mod 11) n
5 – 1 ≡ 0 (mod 23)
x = 4 + 11k (k∈Z+)
5n ≡ 1 (mod 23)
k = 1 için x = 15
n ≡ 22 (en az)
k = 2 için x = 26
(Cevap D)
k = 3 için x = 37
k = 4 için x = 48
k = 5 için x = 59
k = 6 için x = 70 8. 8x ≡ 4 (mod 5)
k = 7 için x = 81
27x ≡ 4 (mod 11)
k = 8 için x = 92 57
+
––––––
428 sistemini sağlayan en küçük x pozitif tam sayısı kaçtır?
(Cevap D) A) 15 B) 16 C) 17 D) 18 E) 19
4. (–18)26 ≡ x (mod 13) x x
8 ≡ 3 ≡ 4 (mod 5), x x
27 ≡ 5 ≡ 4 (mod 11)

denkliğini sağlayan en büyük negatif x tam sayısı kaçtır? 34 ≡ 1 (mod 5) 55 ≡ 1 (mod 11)

A) –12 B) –9 C) –6 D) –3 E) –1 34k+2 ≡ 1 . 32 ≡ 4 (mod 5) 55t + 3 ≡ 4 (mod 11)


(–18)26 = 1826 dir.
x ≡ 4k + 2 ≡ 5t + 3, x + 2 sayısı 4 ün ve 5 in katıdır. x + 2 = 20 ⇒ x = 18 bulunur.
18 ≡ 5 (mod 13)
182 ≡ 12 (mod 13) (Cevap D)

1812 ≡ 1 (mod 13)


182 . 12 . 182 ≡ 1 . 12 = 12 (mod 13)
x = 12 + 13k (k∈Z+)
k = –1 için x = –1 bulunur.
(Cevap E)
Test 08 1. E 2. D 3. D 4. E 5. B 6. D 7. D 8. D 9. E 10. B 11. C 12. B 13. E 14. A 15. C 16. E

9. (–27)50 ≡ x (mod 5) 13. 10!


denkliğini sağlayan en küçük x doğal sayısı kaçtır? sayısının 11 ile bölümünden kalan kaçtır?

A) 0 B) 1 C) 2 D) 3 E) 4 A) 6 B) 7 C) 8 D) 9 E) 10
50 50 10! ≡ –1 ≡ 10 (mod 11)
(–27) = 27 ve 27 ≡ 2 (mod 5)
(Cevap E)
21 ≡ 2 (mod 5)
22 ≡ 4 (mod 5)
23 ≡ 3 (mod 5)
24 ≡ 1 (mod 5)
250 ≡ 24 . 12 + 2 ≡ 1 . 22 ≡ 4 (mod 5)
x = 4 bulunur.

(Cevap E)

10. 4x ≡ 5 (mod 7)
denkliğini sağlayan iki basamaklı kaç doğal sayı vardır? 14. 16! + 535

A) 12 B) 13 C) 14 D) 15 E) 16 sayısının 17 ile bölümünden kalan kaçtır?


4x ≡ 5 (mod 7) ⇒ 2 . 4x ≡ 2 . 5 (mod 7)
A) 5 B) 6 C) 8 D) 11 E) 15
⇒ x ≡ 3 (mod 7) 16! ≡ –1 (mod 17) dir.
x = 3 + 7k (k∈Z+) 516 ≡ 1 (mod 17)
k = 1, 2, 3, ..., 13 için x sayısı iki basamaklı olur. 13 tane doğal sayı vardır. 535 ≡ 52 . 16 + 3 ≡ 1 . 53 = 6 (mod 17)

(Cevap B) 16! ≡ 535 ≡ –1 + 6 ≡ 5 (mod 17)

(Cevap A)
Çözüm Yayınları

11. 30! ≡ 0 (mod 3n)


olduğuna göre, n nin alabileceği en büyük tam sayı değeri
15. a iki basamaklı bir doğal sayı olmak üzere,
kaçtır?
a2 ≡ 5 (mod 11)
A) 12 B) 13 C) 14 D) 15 E) 16
30 3 10 3 3 3 denkliğini sağlayan kaç farklı a sayısı vardır?
10 3 1
A) 14 B) 15 C) 16 D) 17 E) 18
n nin alabileceği en büyük tam sayı değeri 10 + 3 + 1 = 14 tür. a2 ≡ 5 (mod 11) ⇒ a = 4 (mod 11) veya a = 7 (mod 11)

(Cevap C) a sayısı 15, 26, 37, 48, 59, 70, 81, 92 veya 18, 29, 40, 51, 62, 73, 84, 95 olabilir.
Denkliği sağlayan 16 tane iki basamaklı sayı vardır.
(Cevap C)

58

12. p bir asal sayı ise,


(p – 1)! ≡ –1 (mod p)

dir. (Wilson Teoremi)


16. x3 + 2x2 + 5x – 3 ≡ (x2 + ax + 1) [mod (x – 2)]
Buna göre, 12! + 326 sayısının 13 ile bölümünden kalan
kaçtır? olduğuna göre, a kaçtır?

A) 6 B) 8 C) 9 D) 10 E) 12 A) 2 B) 3 C) 5 D) 7 E) 9
312 ≡ 1 (mod 13) Polinomların x – 2 ile bölümünden kalanlar eşittir.
26 = 2 . 13 + 2 olduğundan x = 2 için 8 + 8 + 10 – 3 = 4 + 2a + 1 ⇒ a = 9 bulunur.
(Cevap E)
36 ≡ 32 ≡ 9 (mod 13)
12! ≡ 1 (mod 13)
12! + 326 ≡ –1 + 9 ≡ 8 (mod 13) (Cevap B)
Modüler Aritmetikte İşlemler BÖLÜM 03 Test 09
1. (81473)53 5. m pozitif tam sayı olmak üzere,
sayısının 11 ile bölümünden kalan kaçtır? 51 ≡ 33 (mod m)

A) 1 B) 2 C) 5 D) 8 E) 10 29 ≡ 5 (mod m)
81473 sayısının 11 ile bölümünden kalan 3 – 7 + 4 – 1 + 8 = 7 dir. 81473 ≡ 7 (mod 11) denklemini sağlayan m sayılarının toplamı kaçtır?
71 ≡ 7 (mod 11)
A) 12 B) 18 C) 24 D) 30 E) 36
72 ≡ 5 (mod 11)
51 – 33 ≡ 0 (mod m) ⇒ 18 ≡ 0 (mod m)
73 ≡ 2 (mod 11)
29 – 5 ≡ 0 (mod m) ⇒ 24 ≡ 0 (mod m)
710 ≡ 1 (mod 11)
EBOB(18, 24) = 6 dır.
753 ≡ 25 . 10 + 3 ≡ 73 ≡ 2 (mod 11)
m sayısı 6 nın bölenleri olmalıdır.
(Cevap B) m → 1 + 2 + 3 + 6 = 12
(Cevap A)

2. 915 + 1015 + 1115 + 1215 + 1315


6. (–25)20 ≡ x (mod 11)
toplamının 7 ile bölümünden kalan kaçtır?
denkliğini sağlayan en büyük negatif x tam sayısı kaçtır?
A) 2 B) 3 C) 4 D) 5 E) 6
915 + 1015 + 115 + 1215 + 1315 + ≡ 215 + 315 + 415 + 515 + 615 (mod 7) A) –10 B) –9 C) –8 D) –7 E) –6
215 + 515 ≡ 0 (mod 7) ve (–25)20 = 2520 ve 25 ≡ 3 (mod 11)
315 + 415 ≡ 0 (mod 7) dir. 310 ≡ 1 (mod 11)
61 ≡ 6 (mod 7) 320 ≡ 1 (mod 11)
62 ≡ 1 (mod 7) x = 1 + 11k (k∈Z)
615 ≡ (62)7 . 6 ≡ 6 (mod 7) k = –1 için x = –10 olur.
(Cevap A)
Çözüm Yayınları

(Cevap E)

3. x ≡ 3 (mod 4)
7. 72 ≡ 12 (mod p)
x ≡ 2 (mod 5)
denkliğini sağlayan kaç tane p asal sayısı vardır?
sistemini sağlayan en küçük x doğal sayısı kaçtır?
A) 2 B) 3 C) 4 D) 5 E) 6
A) 4 B) 5 C) 6 D) 7 E) 8 p |72 – 12 ≡ p| 60
x ≡ 3 (mod 4) ⇒ 5x ≡ 15 (mod 20)
60 = 22 . 3 . 5 olduğundan
x ≡ 2 (mod 4) ⇒ 4x ≡ 8 (mod 20)
p asal sayıları 2, 3 ve 5 tir.
Sonda bulunan iki denklik taraf tarafa çıkarılılarak (Cevap B)

x ≡ 7 (mod 20)
bulunur. Sistem sağlayan en küçük x doğal sayısı 7 dir.
(Cevap D)

59
8. (314 – 1) · (314 + 1)
çarpımının 7 ile bölümünden kalan kaçtır?

A) 0 B) 1 C) 2 D) 3 E) 4
4. x, üç basamaklı bir doğal sayı olmak üzere,
(314 – 1) (314 + 1) = 328 – 1
x = 7 (mod 8)
31 ≡ 3 (mod 7)
x = 5 (mod 6) 32 ≡ 2 (mod 7)

olduğuna göre, x in alabileceği en küçük değer kaçtır? 33 ≡ 6 (mod 7)


34 ≡ 4 (mod 7)
A) 103 B) 116 C) 119 D) 121 E) 124
3x ≡ 21 (mod 24) 35 ≡ 5 (mod 7)
⇒ x ≡ –1 (mod 24) 36 ≡ 1 (mod 7)
4x ≡ 20 (mod 24)
328 ≡ 34 . 6 + 4 ≡ 34 ≡ 4 (mod 7)
x = –1 + 24k (k∈Z+)
328 – 1 ≡ 3 (mod 7)
k = 5 için x = 119 bulunur. (Cevap D)
(Cevap C)
Test 09 1. B 2. E 3. D 4. C 5. A 6. A 7. B 8. D 9. B 10. A 11. B 12. A 13. C 14. A 15. B 16. B

9. x2 ≡ 1 (mod 7) 13. 3x ≡ 467 (mod 11)


denkliğini sağlayan iki basamaklı en küçük x doğal sayısı denkliğini sağlayan iki basamaklı en büyük x tam sayısı
kaçtır? kaçtır?

A) 11 B) 13 C) 15 D) 20 E) 22 A) 95 B) 96 C) 97 D) 98 E) 99
x2 ≡ 1 (mod 7) ⇒ x ≡ 1 (mod 7) veya x ≡ –1 (mod 7) 467 nin 11 ile bölümünden kalan
⇒ x ≡ 1 + 7k veya x = –1 + 7t 7 – 6 + 4 = 5 tir.
x → 8, 15, 22, ... ve ya x → 6, 13, 20 ... 3x ≡ 5 (mod 11) ⇒ 4 . 3 ≡ 4 . 5 (mod 11)
Denkliği sağlayan iki basamaklı en küçük sayı 13 tür. ⇒ x ≡ 9 (mod 11)
(Cevap B)
x = 9 + 11k, k = 8 için x = 97 bulunur.
(Cevap C)

10. Bir ilaç kutusunda 20 tablet vardır. Hastaya ilk tablet saat
21:00 de veriliyor ve her 8 saatte bir tablet alması söyleniyor.
14. 540!
Buna göre, hasta son tableti saat kaçta alır?
sayısının 17 ile bölümünden kalan kaçtır?
A) 05:00 B) 11:00 C) 15:00
A) 1 B) 3 C) 5 D) 7 E) 9
D) 19:00 E) 23:00 516 ≡ 1 (mod 17) dir.
Hastaya 1 tablet verildiğine göre geri kalan 19 tabletten sonuncusu 19 . 8 = 152 saat 40! içinde 16 çarpanı bulunduğundan 40! sayısı 16 nın katıdır.
sonra verilecektir. 540! ≡ 516k ≡ 1 dir.
(Cevap A)
152 ≡ 8 (mod 24)

21:00 + 8 ≡ 05:00 (mod 24)


Çözüm Yayınları

(Cevap A)

11. Aşağıdakilerden hangisi


3x ≡ 2x (mod 5) 15. (12!)15!
denkliğinin bir kökü değildir? sayısının 13 ile bölümünden kalan kaçtır?
A) 4 B) 6 C) 8 D) 12 E) 16 A) 0 B) 1 C) 3 D) 5 E) 12
34 = 1 (mod 5) ve 24 ≡ 1 (mod 5) 12! ≡ –1 (mod 13) ve 15! çift sayıdır.
(12!)15! ≡ (–1)15! ≡ 1 (mod 13)
olduğundan x = 4k (k∈Z+) (Cevap B)

k = 1 için x = 4

k = 2 için x = 8

k = 3 için x = 12

k = 4 için x = 16 olur.

x = 6 olamaz.
60
(Cevap B)
16. ab iki basamaklı bir doğal sayı olmak üzere,
ab ≡ 4 (mod 5)
12. 15 + 25 + 35 +...+ 165 ≡ x (mod 17) ab ≡ 6 (mod 7)
denkliğini sağlayan en küçük x doğal sayısı kaçtır? olduğuna göre, a + b toplamının alabileceği en büyük
değer kaçtır?
A) 0 B) 1 C) 2 D) 3 E) 4
15 + 165 ≡ 0 (mod 17) A) 16 B) 15 C) 12 D) 9 E) 7
ab ≡ 4 (mod 5) ⇒ 7 . ab ≡ 28 (mod 35)
25 + 155 ≡ 0 (mod 17)
.
. ab ≡ 6 (mod 7) ⇒ 5 . ab ≡ 30 (mod 35)
.
Son iki denklik taraf tarafa çıkarılarak
75 + 85 ≡ 0 (mod 17) 2 . ab ≡ –2(mod 35) ⇒ ab ≡ –1 (mod 35) bulunur.

olduğundan x ≡ 0 (mod 17) olur. ab = –1 + 35k, k = 2 için ab = 69 ve a + b = 15 olur.


(Cevap B)
(Cevap A)
Modüler Aritmetikte İşlemler BÖLÜM 03 Test 10
1. 520 5. 36715
sayısının 7 ile bölümünden kalan kaçtır? sayısının birler basamağındaki rakam kaçtır?

A) 2 B) 3 C) 4 D) 5 E) 6 A) 1 B) 3 C) 5 D) 7 E) 9
56 ≡ 1 (mod 7)
71 ≡ 7 (Mod 10)
56 . 3 + 2 ≡ 1 . 52 = 4 (mod 7)
(Cevap C) 72 ≡ 9 (Mod 10)

73 ≡ 3 (Mod 10)

74 ≡ 1 (Mod 10)

73 . 4 + 3 ≡ 73 ≡ 3 (Mod 10)

(Cevap B)

2. 5 günde bir nöbet tutan bir doktor birinci nöbetini salı 6. 4163
günü tuttuğuna göre, altıncı nöbetini hangi gün tutar?
sayısının 17 ile bölümünden kalan kaçtır?
A) Çarşamba B) Perşembe C) Cuma
A) 1 B) 10 C) 13 D) 5 E) 16
D) Cumartesi E) Pazar 416 ≡ 1 (Mod 17)

Birinci nöbetinden sonra 5 nöbet daha tutacaktır. Altıncı nöbetini 5 . 5 = 25 gün sonra 416 . 10 + 3 ≡ 43 ≡ 13 (Mod 17)
tutar
(Cevap C)
25 ≡ 4 (Mod 7)

Salı gününden 4 gün sonra cumartesi günü tutar.


Çözüm Yayınları

(Cevap D)

7. 7x ≡ 1 (mod 18)
3. (42973)42 denkliğini sağlayan en küçük pozitif x tam sayısı kaçtır?

sayısının 11 ile bölümünden kalan kaçtır? A) 2 B) 3 C) 4 D) 5 E) 6


71 ≡ 7 (Mod 18)
A) 5 B) 6 C) 7 D) 8 E) 9
42973 sayısının 11 ile bölümünden elde edilen kalan 3 – 7 + 9 – 2 + 4 = 7'dir. 72 ≡ 13 (Mod 18)

42973 ≡ 7 (Mod 11) 73 ≡ 1 (Mod 18)

710 ≡ 1 (Mod 11) x = 3 bulunur.

710 . 4 + 2 ≡ 72 ≡ 5 (Mod 11) (Cevap B)

(Cevap A)
61

8. Aşağıdakilerden hangisi

4. 7x ≡ 15 (mod 23) 5x ≡ 3x (mod 7)

denkliğini sağlayan en küçük x doğal sayısı kaçtır? denkliğinin bir köküdür?

A) 10 B) 11 C) 12 D) 13 E) 14 A) 4 B) 7 C) 10 D) 12 E) 15
7x ≡ 15 (mod 23) ⇒ 10 . 7x ≡ 10 . 15 (mod 23) 56 ≡ 1 (Mod 7)
36 ≡ 1 (Mod 7)
⇒ x ≡ 12 (mod 23)
x = 6k, k∈Z dir. x ∈ {6, 12, 18, ...}
(Cevap C)
x = 12 denkliğin bir köküdür.
(Cevap D)
Test 10 1. C 2. D 3. A 4. C 5. B 6. C 7. B 8. D 9. D 10. D 11. A 12. E 13. C 14. C 15. B 16. B

9. x ≡ 3 (mod 6) 13. 414 + 144 + 34! ≡ x (mod 6)


x ≡ 7 (mod 8) olduğuna göre, x in alabileceği en küçük tam sayı değeri
kaçtır?
sistemini sağlayan en küçük x doğal sayısı kaçtır?
A) 0 B) 1 C) 2 D) 3 E) 4
A) 3 B) 5 C) 9 D) 15 E) 39
x ≡ 3 (mod 6) ⇒ 4 . x ≡ 12 (mod 24) 41 ≡ 4 (mod 6) 141 ≡ 2 (mod 6)

x ≡ 7 (mod 8) ⇒ 3 . x ≡ 21 (mod 24) 42 ≡ 4 (mod 6) 142 ≡ 4 (mod 6)


.
.
Denklikleri taraf tarafa çıkarılır. .
414 ≡ 4 (mod 6) 143 ≡ 2 (mod 6)
x ≡ –9 (mod 24) ⇒ x ≡ –9 + 24 (mod 24)
34! ≡ 0 (mod 6) olduğundan
⇒ x ≡ 15 (mod 24)
414 + 1414 + 34! ≡ 4 + 4 + 0 ≡ 2 (mod 6) olur.
(Cevap D)
(Cevap C)

14. n pozitif tam sayı olmak üzere,


10. 5x ≡ 313 (mod 13) 91 · 92 · 93 · . . . · 9n ≡ 1 (mod 17)
denkliğini sağlayan en küçük x doğal sayısı kaçtır? olduğuna göre, n nin alabileceği en küçük değer kaçtır?
A) 8 B) 9 C) 10 D) 11 E) 12 A) 2 B) 3 C) 16 D) 20 E) 23
312 ≡ 1 (mod 3) ⇒ 4 . x ≡ 12 (mod 24) n(n + 1)
(32) 2 = 3 n(n + 1) = (3n)n + 1
313 ≡ 3 (mod 13) olduğundan
316 ≡ 1 (mod 17) ⇒ (316)17 ≡ 1 (mod 17)
5x ≡ 3 (mod 13) olmalıdır.
n nin alabileceği en küçük değer 16 dır.
Çözüm Yayınları

x = 11 için 5x = 55 ve 55 = 3 (mod 13) olur.


(Cevap C)
(Cevap D)

15. x ve y pozitif tam sayılar olmak üzere,


5x – 3y = 2 (mod 7)
11. 25! – 24!
3x + 11y = 3 (mod 7)
sayısının 23 ile bölümünden kalan kaçtır?
olduğuna göre, x in alabileceği en küçük değer kaçtır?
A) 0 B) 1 C) 2 D) 3 E) 4
24!(25 – 1) = 24! . 24 ≡ 0 (mod 23) A) 2 B) 3 C) 4 D) 5 E) 6
4./5x – 3y ≡ 2 (mod 7) ⇒ 6x –5y = 1 (mod 7)
(Cevap A)
3./3x – 4y ≡ 3 (mod 7) ⇒ 2x –5y = 2 (mod 7)

Denklikler taraf tarafa toplanarak

8x ≡ 3 (mod 7) ⇒ x ≡ 3 (mod 7) bulunur.

(Cevap B)
62

16. 152+30!
sayısının 13 ile bölümünden kalan kaçtır?
12. m pozitif tam sayı olmak üzere,
A) 1 B) 4 C) 5 D) 10 E) 12
35 ≡ 0 (mod m)
52 + 30! = 152 . 1530! dir.
olduğuna göre, m nin alabileceği değerlerin toplamı
15 ≡ 2 (mod 13)
kaçtır?
152 ≡ 4 (mod 13)
A) 243 B) 314 C) 320 D) 343 E) 364
36 – 1 1512 ≡ 1 (mod 13)
1 + 3 + 32 + 33 + 34 + 35 = = 364
3–1 1530! ≡ 1 (mod 13) (30! içinde 12 çarpanı bulunduğundan)
(Cevap E)
152 + 30! ≡ 152 . 1530! ≡ 4 . 1 ≡ 4 (mod 13)

(Cevap B)
BİRE BİR BÖLÜM 03 Test 11
1. A4BC ve A5BC dört basamaklı birer doğal sayıdır. 5. n bir pozitif tam sayı olmak üzere, n yi kalansız bölen pozitif
tam sayıların kümesi S(n) ile gösteriliyor.
A4BC sayısı 15 e bölündüğünde kalan 12 olduğuna göre,
A5BC sayısı 15 e bölündüğünde kalan kaç olur? Buna göre, S(36) ∩ S(48) kümesi kaç elemanlıdır?

A) 7 B) 8 C) 9 D) 10 E) 11 A) 4 B) 5 C) 6 D) 7 E) 8
A4BC = 15k + 12
S(36) = {1, 2, 3, 4, 6, 9, 12, 18, 36}
A5BC = A4BC + 100
S(48) = {1, 2, 3, 4, 6, 8, 12, 16, 24, 48}
= (15k + 12) + 100
S(36) ∩ S(48) = {1, 2, 3, 4, 6, 12}
= 15k + 112 = 15k + 15 . 7 + 7
(Cevap C)
Kalan 7 olur.

(Cevap A)

6. İki basamaklı a ve b pozitif tam sayıları için


2. a, b ve c farklı asal sayılar olmak üzere, a ! = 132
!
b
ab + ac = 2a2 + 12
olduğuna göre, a + b toplamı kaçtır?
olduğuna göre, a · b · c çarpımı kaçtır?
a(b + c) = 2(a2 + 6) A) 20 B) 22 C) 24 D) 26 E) 28
a! = b! . 11 . 12 dir.
a = 2 dir. b + c = a2 + 6 ⇒ b + c = 10
b = 10 ⇒ a = 12 bulunur.
b ve c asal sayılarının biri 3 ve diğeri 7 dir.
a + b = 22 dir.
a . b . c = 2 . 3 . 7 = 42 bulunur.
(Cevap B)
(Cevap 42)
Çözüm Yayınları

7. n bir pozitif tam sayı olmak üzere, n yi tam bölen her bir p asal
sayısı için p2 de n yi tam bölüyorsa n ye bir kuvvetli sayı denir.
3. 5 e tam bölünemeyen pozitif tam sayılar küçükten büyüğe Buna göre, aşağıdakilerden hangisi bir kuvvetli sayı
doğru sıralanıyor. değildir?
Bu sıralamadaki 60. sayı kaçtır? A) 8 B) 27 C) 32 D) 63 E) 125
63 = 32 . 7 dir.
A) 70 B) 74 C) 75 D) 82 E) 86
63 sayısı 72 ile tam bölünmediği için kuvvetli sayı değildir.

n (Cevap D)
n– = 60 ⇒ 4n = 300
5
n ⇒ = 75

Atılan sayıların sonuncusu 75 tir.

60. sayı 74 olur.

(Cevap B) 8. a ve b birer pozitif tam sayı ve p asal sayı olmak üzere,


63
a2 – b2 = p

olduğuna göre, b nin p türünden eşiti aşağıdakilerden


hangisidir?

4. Dört basamaklı 7A8B sayısı 45 sayısının tam katıdır. p −1 p −1 p +1


A) B) C)
2 3 2
Buna göre, A yerine yazılabilecek rakamların toplamı
kaçtır? p +1 p+2
D) E)
3 3
A) 6 B) 8 C) 9 D) 10 E) 12
(a – b) (a + b) = 1 . p
a–b=1 –a + b = –1
B = 0 için A = 3 ve ⇒
a+b=p a+b=p
B = 5 için A = 7 bulunur. ⇒ 2b = p – 1
p–1
A yerine yazılabilecek rakamların toplamı 10 dur. ⇒ b = dir.
2
(Cevap D) (Cevap A)
Test 11 1. A 2. (42) 3. B 4. D 5. C 6. B 7. D 8. A 9. E 10. (6) 11. C 12. B 13. (9) 14. B 15. E 16. D

9. a bir pozitif tam sayı ve p = a2 + 3 tür. 13. a, b ve c pozitif tam sayıları için
p bir asal sayı olduğuna göre, 7! – 6! = 2a · 3b · 5c

I. a çift sayıdır. olduğuna göre, a + b + c toplamı kaçtır?

II. p nin 4 ile bölümünden kalan 3 tür. 6!(7 – 1) = 6! . 6

= 24 . 32 . 5 . (2 . 3)
III. p + 4 asaldır.
= 25 . 3 3 . 5 1
ifadelerinden hangileri doğrudur?
a + b + c = 5 + 3 + 1 = 9 bulunur.
A) Yalnız I B) I ve II C) I ve III
(Cevap 9)
D) II ve III E) I, II ve III
p sayısı 3 ten büyük bir asal sayı olduğundan tek sayıdır. a2 çift sayı olur.

a2 çift sayı ⇒ a çift sayıdır.

⇒ a2, 4 ile tam bölünür.

p = a2 + 3 sayısının 4 ile bölümünden kalan 3, p + 4 = a2 + 7 asal sayıdır.

(Cevap E) 14. A = 11 + 22 + 33 + . . . + 121


olduğuna göre, A yı tam bölen asal sayıların toplamı
kaçtır?

A) 15 B) 16 C) 17 D) 18 E) 19
11 . 12
10. (n + 1)! – n! – (n – 1)! = 23 ·3· 52 ·7 11(1 + 2 + 3 + ... + 11) = 11 .
2
olduğuna göre, n kaçtır? = 11 . 6 . 11
(n – 1)! [n(n + 1) – n – 1] = (n – 1)! [n2 + n – n – 1] = 2 . 3 . 112
= (n 1)! (n2 – 1) Toplamları 2 + 3 + 11 = 16 bulunur.
= 23 . 3 . 5 . 35
Çözüm Yayınları

14243 (Cevap B)
5!

n2 – 1 = 35 ⇒ n = 6 bulunur.

(Cevap 6) 15. a = 3! · 7!
b = 4! · 6!

c = 5! · 5!

olduğuna göre, aşağıdaki sıralamalardan hangisi


11. a bir pozitif tam sayı olmak üzere, doğrudur?
8a A) a < b < c B) a < c < b C) b < a < c
a + 3
D) b < c < a E) c < b < a
ifadesi bir tam sayının karesine eşit olduğuna göre, a = 42 . 3! . 5!
a kaçtır? b = 24 . 3! . 5!

A) 1 B) 2 C) 3 D) 4 E) 5 c = 20 . 3! . 5!

8a a+3 8a 24 olduğundan c < b < a dır.


=8–
8a + 24 8 a+3 a+3
(Cevap E)
–24
a + 3 sayısı 24 ün bir bölenidir.
64
8a
a + 3 = 6 ⇒ a = 3 için = 4 olur.
a+3
60
(Cevap C) 16. n bir tam sayı olmak üzere, ifadesi bir asal sayıya eşittir.
n
Buna göre, n nin alabileceği değerlerin toplamı kaçtır?

12. 6! · a A) 54 B) 56 C) 58 D) 62 E) 64
60 22 . 3 . 5
çarpımı bir pozitif tam sayının karesi olduğuna göre, =
n n
a nın alabileceği en küçük pozitif tam sayı değeri kaçtır? 2
2 .3.5
= 2 ⇒ n = 30
n
A) 4 B) 5 C) 10 D) 15 E) 20
22 . 3 . 5
6! . a = b2 ⇒ 24 . 32 . 5 . a = b2 = 3 ⇒ n = 20
n
a = 5 bulunur. 22 . 3 . 5
= 5 ⇒ n = 12
n
(Cevap B) Toplamları 62 bulunur.

(Cevap D)
BİRE BİR BÖLÜM 03 Test 12
1. a ve b pozitif tam sayılar olmak üzere, 5. a ve b pozitif tam sayıları arasında
EBOB(a, b) = 1 a = EBOB(120, b)

dir. bağıntısı vardır.


Buna göre,
a · b = 120 olduğuna göre, kaç farklı (a, b) sıralı ikilisi
bulunabilir? I. a çift sayı ise b de çift sayıdır.
II. a tek sayı ise b çift sayıdır.
A) 4 B) 6 C) 8 D) 12 E) 16
(1, 120), (3, 40), (5, 24), (8, 15) ya da III. b çift sayı ise a da çift sayıdır.
(120, 1), (40, 3), (24, 5), (15, 8) olmak üzere 8 tane (a, b) ikilisi bulunur. ifadelerinden hangileri doğrudur?
(Cevap C) A) Yalnız I B) I ve II C) I ve III

D) II ve III E) I, II ve III
120 = 23 . 3 . 5 dir.

b çift sayı ise EBOB(120, 2k) = a çitf sayıdır.

(Cevap C)

2. a ve b pozitif tam sayılarının en büyük ortak böleni d olmak


üzere, 6. a ve b pozitif tam sayılar olmak üzere,
I. d2 sayısı, a2 sayısını böler. EBOB(a, b) = 6

II. d2 sayısı, a + b3 sayısını böler. EKOK(a, b) = 72


a + b = 42
III. d2 sayısı, a3 + b2 sayısını böler.
olduğuna göre, |a – b| kaçtır?
ifadelerinden hangileri her zaman doğrudur?
A) 6 B) 8 C) 9 D) 12 E) 15
A) Yalnız I B) I ve II C) I ve III a.b

D) II ve III E) I, II ve III a + b = 42 ⇒ b = 42 – a
Çözüm Yayınları

d2 sayısı a sayısını her zaman bölmediği için a + b3 sayısını bölmez.


a . (42 – a) = 6 . 72 ⇒ 42a – a2 = 432
(Cevap C)
⇒ a2 – 42a + 432 = 0

⇒ (a – 18) (a – 24)
a = 18 için b = 24
⇒ |a - b| = 6 bulunur.
a = 24 için b = 18
(Cevap A)
7. 16x ≡ 1 (mod 7)
17y ≡ 4 (mod 7)
3. Eni 108 metre, boyu 180 metre olan dikdörtgen biçimindeki
bir tarla, hiç alan artmayacak biçimde eş karelere bölünerek denkliklerini sağlayan x ve y pozit tam sayıları için
küçük bahçeler yapılıyor. x + y toplamının alabileceği en küçük değer kaçtır?

Bu şekilde en az kaç tane eş bahçe elde edilir? A) 4 B) 5 C) 6 D) 7 E) 8


16x ≡ 2x ≡ 1 (mod 7) olmalıdır.
108 = 22 . 33, 180 = 22 . 32 . 5, EBOB = 36
108 . 180 21 ≡ 2 (mod 7)
= 3 . 5 = 15
36, 36 22 ≡ 4 (mod 7)
(Cevap 15) 23 ≡ 4 (mod 7) ⇒ x = 3 (En küçük)
17y ≡ 3y ≡ 4 (mod 7) olmalıdır.
31 ≡ 3 (mod 7)
65
4. a, b, c pozitif tam sayılar ve a < b < c dir.
32 = 2 (mod 7)
EBOB(a, b) = 6 33 = 6 (mod 7)

EBOB(b, c) = 7 34 = 4 (mod 7)
x + y = 3 + 4 = 7 dir.
olduğuna göre, a + b + c toplamının alabileceği (Cevap D)
en küçük değer kaçtır? 8. 1 < a < 10 olmak üzere,

A) 55 B) 75 C) 84 D) 97 E) 105 18 – a ≡ 0 (mod a)
a = 6k, b = 6t denkliğini sağlayan kaç tane a tam sayısı vardır?
b = 7m, c = 7n A) 2 B) 3 C) 4 D) 5 E) 6
18 – a
b sayısı 6 ve 7 nin katıdır. tam sayı olmalıdır.
a
a = 6, b = 42, c = 49 olur. 18 – a 18
= –1 ve 1 < a < 10
a a
a + b + c = 6 + 42 + 49 = 97 a = 2, a = 3, a = 6, a = 6 olabilir.
(Cevap C)
(Cevap D)
Test 12 1. C 2. C 3. (15) 4. D 5. C 6. A 7. D 8. C 9. D 10. B 11. E 12. (90) 13. (4) 14. D 15. D 16. C

9. 1934 + 2050 13. 165 + 245 + 325


toplamının 17 ile bölümünden kalan kaçtır? toplamının 7 ile bölümünden kalan kaçtır?
16 + 245 + 325 ≡ 25 + 35 + 45 (mod 7)
15
A) 4 B) 6 C) 9 D) 13 E) 16 123
1934 ≡ 234 ≡ 22 . 16 + 2 ≡ 1 . 2 ≡ 4 (mod 17) ve 0

2050 ≡ 33 . 16 + 2 ≡ 1 . 32 ≡ 9 (mod 17) dir. ≡ 4 (mod 7)

Toplamları 4 + 9 = 13 olur. (Cevap 4)

(Cevap D)

14. 230 · 315


çarpımının 5 ile bölümünden kalan kaçtır?

A) 0 B) 1 C) 2 D) 3 E) 4
10. 2a · 3b ≡ 0 (mod 108) 24 ≡ 1 (mod 5)

2b · 3a ≡ 0 (mod 9) 230 ≡ 24 . 7 + 2 ≡ 1 . 22 ≡ 4 (mod 5)


34 ≡ 1 (mod 5)
sistemini sağlayan a ve b pozitif tam sayıları için a + b
toplamı en az kaçtır? 315 ≡ 34 . 3 + 3 ≡ 1 . 32 ≡ 2 (mod 5)

230 ≡ 215 ≡ 4 . 2 ≡ 3 (mod 5)


A) 4 B) 5 C) 6 D) 7 E) 8
2b . 3a ≡ 0 (mod 9) ⇒ a = 2 (En az) (Cevap D)

2a . 3b ≡ 0 (mod 108) ⇒ 22 . 3b ≡ 0 (mod 108)

⇒ b = 3 (En az)

a + b = 2 + 3 = 5 bulunur. 15. Aşağıda beş lambadan oluşan bir reklam panosu


Çözüm Yayınları

(Cevap B) gösterilmiştir.

A B C D E

11. 3, 6 ve 9 ile kalansız bölünebilen üç basamaklı sayıların Panodaki lambalar A lambasından başlayarak soldan sağa
en küçüğü ile en büyüğünün toplamı kaçtır? doğru, E lambasından sonra ise sağdan sola doğru devamlı
olarak yanıp sönmektedir.
A) 918 B) 998 C) 1008 D) 1018 E) 1098
EKOK(3, 6, 9) = 18 Örneğin, lambalar A - B - C - D - E - D - C - B - A - B - ...
sırasında yanıp söndüğünden 7. sırada yanıp sönen lamba C
999 18 En büyük olan 999 – 9 = 990 dır.
lambasıdır.
55
9 Buna göre, 100. sırada yanıp sönen lamba hangisidir?
En küçük olan 18 . 6 = 108 dir. A) A B) B C) C D) D E) E
Toplamları 108 + 990 = 1098 olur. 100 ≡ 4 (mod 8) olduğundan 100 . sırada yanıp sönen lamba 4. sırada bulunan D
lambasıdır.
(Cevap E)
(Cevap D)

66
12. m, 1 den büyük bir tam sayı ve
75 ≡ 3 (mod m)
16. 7k + 4
110 ≡ 2 (mod m)
biçimindeki bir sayı 5 ile kalansız bölünebildiğine göre,
olduğuna göre, m nin alabileceği değerler toplamı kaçtır?
30 dan küçük k pozitif tam sayıları kaç tanedir?
72 ≡ 0 (mod m)
A) 4 B) 5 C) 6 D) 7 E) 8
108 ≡ 0 (mod m)
7k + 4 ≡ 0 (mod 5) ⇒ (7k + 4) + 1 ≡ 1 (mod 5)
EBOB(72, 108) = 36
⇒ 7k ≡ 1 (mod 5)
m sayısı 36 nın bölenleri olmalıdır.
k = 3 için 21 ≡ 1 (mod 5)
Toplamları: 2 + 3 + 4 + 6 + 9 + 12 + 18 + 36 = 90
olduğundan en küçük k sayısı 3 tür.
(Cevap 90)
k∈ {3, 8, 13, 18, 23, 28} olur.

(Cevap C)
TÜMEVARIM - I Test 01
1. P(x) = (x – 2)3 · (x2 + 3x – 1)4 5. q ∨ (q′ ∧ p)

polinomunun sabit terimi kaçtır? önermesinin sadeleştirilmiş biçimi aşağıdakilerden


hangisidir?
A) –10 B) –8 C) –5 D) 3 E) 8
P(0) = (–2)3 . (–1)4 = –8 A) p ∧ q B) p C) q

(Cevap B) D) q′ ∨ p E) p ∨ q
q ∨ (p ∧ q) = (qı ∨ qı) ∧ (q ∨ p)
123
1
= 1 ∧ (p ∨ q)
=p∨q
(Cevap E)

2. P(x) = x3 – x2 + 2x + a

polinomunun bir çarpanı x – 1 olduğuna göre diğer


çarpanı aşağıdakilerden hangisidir?
6. q ∧ (p′ ∨ q)′
bileşik önermesinin olumsuzu (değili) aşağıdakilerden
A) x2 – 2 B) x2 – 1 C) x2 + 1
hangisine denktir?
D) x2 + 2 E) x2 + 2x
A) p′ B) p ∧ q′ C) 1
P(1) = ⇒ 1 – 1 + 2 + a = 0
D) q E) 0
⇒ a = –2 bulunur.
[q ∧ (pı ∧ q)ı] ı = qı ∨ (pı ∨ q)
P(x) = x3 – x2 + 2x – 2 = p ∨ (q ∨ qı)
=p∨1
= x2(x – 1) + 2(x – 1)
=1
Çözüm Yayınları

= (x – 1) (x2 + 2) (Cevap C)

(Cevap D)

3. Katsayılar toplamı 2 olan bir P(x) polinomunun x – 2 ile


bölümünden kalan 5 tir.
7. Boyutları 240 ve 360 metre olan dikdörtgen şeklindeki
Buna göre, P(x) polinomunun x2 – 3x + 2 ile bölümünden
bir arsanın etrafına, köşelere de dikilmek üzere x metre
kalan aşağıdakilerden hangisidir?
aralıklarla ağaç dikilecektir.
A) 3x – 1 B) 5x – 3 C) x + 3
x bir tam sayı olduğuna göre, bu işlem kaç farklı şekilde
D) 2x + 1 E) 4x – 3 yapılabilir?
P(1) = 2 ve p(2) = 5
A) 16 B) 20 C) 24 D) 32 E) 36
P(x) = (x2 – 3x + 2) . Q(x) + ax + b EBOB(240, 360) = 120
P(1) = 2 ⇒ a + b = 2 120 nin pozitif sölenlerinin sayısı = 120 = 23 . 3 . 5 → 4 . 2 . 2 = 16
⇒ a = 3 ve b = –1
P(2) = 5 ⇒ 2a + b = 5 (Cevap A)

Kalan ax + b = 3x – 1

(Cevap A)

67

x − 3( x − 2) 2
4. ⋅ 29 − x
x−3 2x + 4 x − 6 8. A = 0! + 1! + 2! + 3! + . . . + 50!
ifadesinin sadeleştirilmiş biçimi aşağıdakilerden
olmak üzere, A + x doğal sayısı 10 ile tam bölünebilmektedir.
hangisidir?
Buna göre, x in alabileceği en küçük değer kaçtır?
3−x x−3
A) B) C) 3 - x
x −1 x −1 A) 2 B) 4 C) 5 D) 6 E) 9
D) x – 3 E) x – 1 A = 0! + 1! + 2! + 3! + 4! +14243
144424443 5! + ... + 50!
34 10 ile tam bölünür.
x – 3x + 6 (3 – x) (3 + x) –2(x – 3) (3 – x) (3 + x)
. = . A sayısının 10 ile bölümünden kalan 4 tür.
x–3 2(x2 + 2x –3) x –3 2(x + 3) (x – 1)
A + 6 sayısı 10 ile tam bölünür.
x–3
= x = 6 bulunur.
x –1
(Cevap D)
(Cevap B)
Test 01 1. B 2. D 3. A 4. B 5. E 6. C 7. A 8. D 9. D 10. B 11. D 12. C 13. D 14. B 15. B 16. C

9. xy x+y xy x–y 13. x = 2873


8 13 olduğuna göre, x27 sayısının birler basamağında hangi
4 1 rakam bulunur?

Yukarıdaki bölme işleminde (xy) iki basamaklı bir doğal A) 4 B) 5 C) 6 D) 7 E) 8


sayı olduğuna göre, x + y toplamı kaçtır? x ≡ 3 (mod 10)

A) 6 B) 9 C) 10 D) 11 E) 13 x2 ≡ 9 (mod 10)
xy = 8(x + y) + 4 = 13(x – y) + 1 ⇒ 5x = 21y + 3
x3 ≡ 7 (mod 10)
x+y>4 3
⇒ y >
x–y>1 2 x4 ≡ 1 (mod 10)
⇒y≥2
x27 ≡ x4 . 6 + 3 ≡ 1 . x3 ≡ 7 (mod 10)
y = 2 için x = 9 bulunur. x + y = 11 olur. (Cevap D)
(Cevap D)

14. Dört basamaklı a53b sayısı 12 ile tam bölünebildiğine göre,


a53b
10. 69x0 + 21y0 = 3
12
eşitliğini sağlayan (x0, y0) tam sayı ikilisi aşağıdakilerden
hangisidir? bölümünün alabileceği en büyük değer kaçtır?

A) (–10, 23) B) (–3, 10) C) (4, –3) A) 628 B) 711 C) 715 D) 718 E) 723
Son iki rakam 4 ün katı ve rakamlarının toplamı 3 ün katı olmalıdır.
D) (11, –23) E) (18, –36)
b = 2 için a = 2, 5, 8
3 ile sadeleştirilerek 23x0 + 7y0 = 1
b = 6 için a = 1, 4, 7 olur.
x = –3 ve y = 10 denklemi sağlar.
a = 8 ve b = 2 alınırsa
Çözüm Yayınları

(x0, y0) = (–3, 10) bulunur.


8532
(Cevap B) = 711
12
(Cevap B)

15. a, b, c birer pozitif tam sayı ve

11. EBOB(36, 45, x) = 9 x = 9a + 6 = 12b + 9 = 15c + 12

EKOK(36, 45, x) = 360 dir.

olduğuna göre, x doğal sayısı en az kaçtır? Buna göre, x in alabileceği en küçük değer için a kaç
olur?
A) 48 B) 60 C) 64 D) 72 E) 90
A) 18 B) 19 C) 20 D) 21 E) 24
36 = 22 . 32 , 45 = 32 . 5, 360 = 23 . 32 . 5
x + 3 = 9(a + 1) = 12(b + 1) = 15(c + 1)
x = 32 . 23 = 72 için EBOB = 9 ve
EKOK(9, 12, 15) = 180
EKOK = 360 olur.
9(a + 1) = 180 ⇒ a + 1 = 20
(Cevap D)
68 ⇒ a = 19 bulunur.

(Cevap B)

16. x ∈ Z olmak üzere,


12. 4 günde bir nöbet tutan bir doktor, 3. nöbetini salı günü
tuttuğuna göre, 16. nöbetini hangi gün tutar? 2x – 1 ≡ 2 (mod 5)

A) Çarşamba B) Perşembe C) Cuma olduğuna göre, x sayısı aşağıdakilerden hangisi olamaz?

D) Cumartesi E) Pazar A) 9 B) 14 C) 20 D) 24 E) 29
13 nöbet daha tutacaktır. 16. nöbetini 13. 4 = 52 gün sonra tutar 2x ≡ 3 (Mod 5) ⇒ 3 . 2x ≡ 3 . 3 (mod 5)
⇒x ≡ 4 (mod 5)
52 ≡ 3 (mod 7)
x = 4 + 5k (k∈Z)
16. nöbetini salı gününden 3 gün sonra cuma günü tutar.
k yerine yazılan tam sayılardan hiçbirinden x = 20 bulunmaz.
(Cevap C) (Cevap C)
TÜMEVARIM - I Test 02
2 x − x2
1. P(x) = 2x2 + 3x – 4 5. x − 2x + 1 :
x 2 − 4 x + 3 4 x 2 − 12x
polinomunun x – 2 ile bölümünden elde edilen bölüm ve
ifadesinin sadeleştirilmiş biçimi aşağıdakilerden
kalanın toplamı aşağıdakilerden hangisidir?
hangisidir?
A) 2x – 3 B) 2x + 7 C) 2x + 10 A) –4 B) –2 C) x – 2 D) x – 1 E) x
2
D) 2x + 15 E) 2x + 17 (x – 1) 4x(x – 3)
= = –4
(x – 1) (x – 3) –x(x – 1)
2x2 + 3x – 4 x–2 (Cevap A)
2x2 – 4x 2x + 7
7x – 4
7x – 14
10

(2x + 7) + 10 = 2x + 17 bulunur.

(Cevap E)

6. Aşağıda verilen bileşik önermelerden hangisi bir


2. P(x) ve Q(x) polinomları için,
totolojidir?
P(x) = 2·x·Q(x + 1) + x2 + 3 A) p ∧ p′ B) p′ ⇒ p C) p ⇒ p′
bağıntısı veriliyor. D) p ⇔ p′ E) p ⇒ (q ∨ q′)
P(x) polinomunun çarpanlarından biri x – 3 olduğuna p ⇒ (q ∨ qı) ≡ p ⇒ 1
göre, Q(4) kaçtır? ≡1
(Cevap E)
A) –2 B) –1 C) 0 D) 1 E) 2
x = 3 için p(3) = 2 . 3 . Q(4) + 12
Çözüm Yayınları

0 = 6Q(4) + 12

Q(4) = –2 bulunur.

(Cevap A)

7. x sayısı 24, 60, 84 sayılarını tam bölen bir doğal sayıdır.

7x − 48
Buna göre, ifadesinin en büyük değeri kaçtır?
3. P(x) = x4 + 3x2 + ax + 2 x

polinomu x – 1 ile tam bölünebildiğine göre, x – 2 ile A) 2 B) 3 C) 4 D) 6 E) 7


bölümünden kalan kaçtır? 7x – 48 48
EBOB(24, 60, 84) = =7–
x x
A) 5 B) 12 C) 15 D) 16 E) 18
p(1) = 0 ⇒ 1 + 3 + a + 2 = 0 x = 12 için 3 bulunur.
(Cevap B)
⇒ a = –6

p(2) = 16 + 12 – 12 + 2 = 18 bulunur.

(Cevap E)
69
8. Ayrıtları 20 cm, 25 cm ve 30 cm olan tahtadan yapılmış
dikdörtgenler prizmasının dış yüzeyi boyanmıştır. Bu prizma
hiç artmayacak şekilde en büyük boyutlu ve eşit hacimli
küplere bölünüyor.

x +1 = A + B Buna göre, kaç tanesinin yüzeyinde boya vardır?


4.
x 2 − 5x + 6 x − 3 x − 2
A) 120 B) 112 C) 100 D) 96 E) 80
olduğuna göre, A · B çarpımı kaçtır? EBOB(20, 25, 30) = 5
20 . 25 . 30
A) –8 B) –9 C) –10 D) –12 E) –14 Tüm küplerin sayısı = = 120
5.5.5
x + 1 = (x – 2)A + (x – 3)B Dış yüzeyde kalan küplerde boya vardır. İç yüzeyde bulunan küp sayısı =
(20 – 10) . (25 – 10) . (30 – 10) 10 . 15 . 20
=
x = 3 için A = 4 5.5.5 5.5.5
= 24
x = 2 için B = –3 bulunur.
Dış yüzeyde küp sayısı 120 – 24 = 96
a . B = 4 . (–3) = –12 dir.
(Cevap D) (Cevap D)
Test 02 1. E 2. A 3. E 4. D 5. A 6. E 7. B 8. D 9. C 10. C 11. A 12. A 13. C 14. B 15. B 16. B

9. Dört basamaklı A35B sayısı 12 ile tam bölünebildiğine 13. 215 – 75


göre, bölümün alabileceği en büyük değer kaçtır?
sayısının asal olmayan kaç tane pozitif tam sayı böleni
A) 613 B) 661 C) 696 D) 697 E) 698 vardır?
Son iki basamak 4 ün katı ve rakamları toplamı 3 ün katı olmalıdır.
A) 24 B) 30 C) 33 D) 36 E) 40
B = 2 için a = 2, 5, 8 ve
B = 6 ve a = 1, 4, 7 bulunur. 215 – 75 = 75(35 – 1)
8352 = 75 . 242
a = 8 ve b = 2 için = 696
12 = 75 . 21 . 112
(Cevap C)
Bölen sayısı (5 + 1) (1 + 1) . (2 + 1) = 36 olur.
Asal olmayan bölen sayısı 36 – 3 = 33 tür.
(Cevap C)

10. [(n − 1)!+ (n − 2)!] ⋅ n


n!
14. m iki basamaklı bir doğal sayı ve
ifadesinin sadeleştirilmiş biçimi aşağıdakilerden
hangisidir? 290 ≡ 2 (mod m)
n
A) n B) n2 C) olduğuna göre, m kaç farklı değer alır?
n −1
n2 n A) 8 B) 9 C) 10 D) 11 E) 12
D) E) 290 ≡ 2 (mod m) ⇒ 288 ≡ 0 (mod m)
n +1 n +1
288 = 25 . 32 sayısının 6 . 3 = 18 tane pozitif böleni vardır. Bunlardan 1, 2, 3, 4, 6, 8, 9 bir
(n – 2)F [n – 1 + 1] . n n.n n
= = basamaklı 144, 288 üç basamaklı diğerleri iki basamaklı 18 – 9 = 9
n! (n – 1) . n n–1
(Cevap C) (Cevap B)
Çözüm Yayınları

15. A = 0! + 1! + 2! + 3! + . . . + 89! + 90!


11. (AB) ve (CD) iki basamaklı (ABCD) dört basamaklı doğal olduğuna göre, aşağıdakilerden hangisi 6 ile tam
sayıdır. bölünür?

AB 13 CD 13 A) A + 1 B) A + 2 C) A + 3
ve
D) A + 4 E) A + 5
A = 0! + 1! + 2! + 14243
3! + ... + 90!
4 5 14243
4 6 nın katı
A + 2 sayısı 6 ile tam bölünür.
olduğuna göre, (ABCD) sayısının 13 ile bölümünden kalan
(Cevap B)
kaçtır?

A) 2 B) 3 C) 7 D) 8 E) 12
10A + B = 13k + 4, 10C + D = 13t + 5, k, t∈Z+
16. 2342 + 421
1000A + 100B + 10C + D = 100(10A + B) + 10C + D
= 100(13k + 4) + 13t + 5 toplamının birler basamağındaki rakam aşağıdakilerden
= 13(100k + t) + 405 hangisidir?
70
405 sayısı 13 ile bölündüğünde kalan 2 olur.
A) 2 B) 3 C) 4 D) 6 E) 9
(Cevap A)
23 ≡ 3 (mod 10)
232 ≡ 9 (mod 10)
233 ≡ 7 (mod 10)
234 ≡ 1 (mod 10)
234 . 10 + 2 ≡ 1 . 232 ≡ 9 (mod 10)
12. 120, 144 ve 360 sayılarının pozitif ortak bölenlerinin 41 ≡ 4 (mod 10)
kümesi kaç elemanlıdır? 42 ≡ 6 (mod 10)
A) 8 B) 12 C) 15 D) 16 E) 18 43 ≡ 4 (mod 10)
.
EBOB(120, 144, 360) = 24 = 23 . 3 .
.
Pozitif bölenlerinin sayısı (3 + 1) (1 + 1) = 8 bulunur.
421 ≡ 4 (mod 10)
(Cevap A)
2342 + 421 ≡ 9 + 4 ≡ 3 (mod 10)
(Cevap B)
TÜMEVARIM - I Test 03
1. Bir P(x) polinomu için, 5. (p ⇒ q) ⇒ q'

P(x – 1) = x2 + 4x + m bileşik önermesinin tersi aşağıdakilerden hangisidir?

dir. A) q ⇒ (p ∧ q) B) p′ ⇒ q

P(x) polinomunun sabit terimi 12 olduğuna göre, C) q′ ⇒ (p ⇒ q) D) q ⇒ (p ∧ q′)


katsayıları toplamı kaçtır? E) p ⇒ q
A) 16 B) 17 C) 18 D) 19 E) 20 p ⇒ q önermesinin tersi pı ⇒ qı önermesidir.
(p ⇒ q) ⇒ qı önermesinin tersi
p(0) = 12 olmalıdır.
(p ⇒ q) ⇒ (pı ∨ q)ı ⇒ q
x = 1 için p(0) = 1 + 4 + m ⇒ 12 = 5 + m
⇒ (pı ∨ q) ∨ q
⇒ m = 7 bulunur.
⇒ pı ∨ q
x = 2 için p(1) = 4 + 8 + m
⇒ p ⇒ q olur.
= 12 + 7
(Cevap E)
= 19 elde edilir.
(Cevap D)

6. (p ∧ q) ⇒ p
bileşik önermesinin en sade şekli aşağıdakilerden
2. P(x) = 2x2 + 3x + 5 hangisidir?

polinomunun x – 1 ile bölümünden kalan kaçtır? A) p B) 1 C) q

A) 10 B) 11 C) 12 D) 13 E) 14 D) p v q E) q′
P81) = 2 + 3 + 5 = 10 p ∧ q) ⇒ p ≡ (p ∧ q)ı ∨ p
(Cevap A) ⇒ (pı ∨ qı) ∨ p
⇒ (pı ∨ q) ∨ qı
123
1
Çözüm Yayınları

⇒ 1 ∨ qı
⇒1
(Cevap B)

a(a + b)3 − (a 2 + ab)2


3.
a 2b + ab 2 7. Dört basamaklı 3A4B sayısı 45 ile tam bölünebilen çift sayıdır.
ifadesinin sadeleştirilmiş biçimi aşağıdakilerden 3A4B
hangisidir? Buna göre, işleminin sonucu kaçtır?
15
A) – b B) a + b C) a(a + b) A) 180 B) 196 C) 208 D) 216 E) 240
D) b(a + b) E) a2 – b2 B = 0 ve rakamların toplamı 9 un katı olmalıdır.

a(a + b)3 – [a2(a + b)2] a(a + b)2 (a + b – a) B = 0 için A = 2 bulunur.


= 3240
ab(a + b) ab(a + b) = 216 dır.
15
(Cevap D)
=a+b
(Cevap B)

71

8. a ve b iki doğal sayı olmak üzere,


2
4. x + ax − 3 EKOK(a, b) = 160
x 2 + 5x + b
x −1 EBOB(a, b) = 8
ifadesinin sadeleştirilmiş biçimi olduğuna göre,
x+2
a + b toplamı kaçtır? olduğuna göre, (a + b) toplamı en az kaçtır?

A) 4 B) 5 C) 6 D) 7 E) 8 A) 72 B) 68 C) 64 D) 56 E) 48
x2 + ax – 3 (x – 1) (x + 3) x2 + 2x – 3 a = 8k, b = 8t, k, t∈Z+
= =
x25x + b (x + 2) (x + 3) x2 + 5x + 6 a . b = EKOK(a, b) . EBOB(a . b)
8k . 8t = 160 . 8 ⇒ k . t = 20
a = 2, b = 6 ve a + b = 8 dir.
k = 5 ve t = 4 için a = 40 ve b = 32 bulunur. a + b = 40 + 32 = 72 dir.
(Cevap E)
(Cevap A)
Test 03 1. D 2. A 3. B 4. E 5. E 6. B 7. D 8. A 9. D 10. E 11. D 12. C 13. B 14. C 15. A 16. C

9. 43! + 44! – 1 13. 25203


toplamının sondan kaç basamağı 9 dur? sayısının 11 ile bölümünden kalan kaçtır?

A) 7 B) 8 C) 9 D) 10 E) 11 A) 1 B) 5 C) 7 D) 8 E) 9
43! + 44! – 1 = 43! . (1 + 44) – 1 25 ≡ 3 (mod 11)

43! . 45 – 1 252 ≡ 9 (mod 11)

5 asal çarpanlarının sayısı kadar 43! . 45 sayısının sonunda sıfır bulunur. 253 ≡ 5 (mod 11)
.
43 5 8 5 .
= 45 = 5 . 9 .
8 1
2510 ≡ 1 (mod 11)
sayıda 8 + 1 + 1 = 10 tane 5 asal çarpanı bulunduğunan, sayının sondan 10 basamağı
25203 ≡ 2510 . 20 + 3 ≡ 1 . 253 ≡ 5 (mod 11)
sıfırdır. 43! . 45 – 1 sayısının sondan 10 basamağı 9 olur.
(Cevap B)
(Cevap D)

10. x ve y doğal sayılar olmak üzere,


1! · 2! · 3! · ... · x! = y · 1317 14. n bir doğal sayı olmak üzere,

olduğuna göre, x kaçtır? 126n+1

A) 17 B) 19 C) 21 D) 24 E) 27 sayısının 7 ile bölümünden kalan kaçtır?


1! . 2! . 3! .... 13!
14.414!
24 ....
43 25!
. 26!
12 .3
27!
13 tane 4 tane A) 1 B) 3 C) 5 D) 7 E) 9
12 ≡ 5 (mod 7)
13 asal çarpanı 13 asal çarpanı
x = 27 olmalıdır. 126 ≡ 1 (mod 7)

(Cevap E) 126n ≡ 1 (mod 7)


126n . 12 = 1 . 5 ⇒ 126n + 1 ≡ 5 (mod 7)
(Cevap C)
Çözüm Yayınları

15. Her biri üç basamaklı olan (7A4) ve (BCD) doğal sayıları


11. a ve b birer doğal sayı olmak üzere,
veriliyor.
EBOB(a, b) = 6
(7A4) = 6 · (BCD)
EKOK(a, b) = 150
olduğuna göre, A rakamının alabileceği farklı değerlerin
olduğuna göre, a · b nin kaç tane pozitif tam sayı böleni toplamı kaç olur?
vardır?
A) 12 B) 14 C) 15 D) 16 E) 18
A) 15 B) 21 C) 24 D) 27 E) 30 7A4 sayısı 6 ile tam bölünür. Bu sayı 3 ün katı olan çift sayıdır.
a . b = EBOB(a, b) . EKOK(a, b) ⇒ a . b = 6 . 150
A = 1, A = 4, A = 7 olur. Toplamları 12 dir.
⇒ a . b = 2 2 . 32 . 52
(Cevap A)
Asal sayıların üsleri 1 artırılarak çarpılır. 3 . 3 . 3 = 27 tane pozitif tam sayı bölen vardır.
(Cevap D)

72
16. 65 + 75 + 85
toplamının 13 ile bölümünden kalan kaçtır?
12. Dört basamaklı 3a5b sayısının 4 ile bölümünden kalan
A) 1 B) 5 C) 8 D) 9 E) 12
3 tür.
7 ≡ –6 (mod 13) olduğundan
Bu sayı 9 ile tam bölünebildiğine göre, a yerine kaç
65 + 75 ≡ 65 + (–6)5 ≡ 0 (mod 13) olur.
değişik rakam yazılabilir?
81 ≡ 8 (mod 13)
A) 2 B) 3 C) 4 D) 5 E) 6
82 ≡ 12 (mod 13)
Son iki basamağın 4 ile bölümünden kalan 3 olmalıdır.
83 ≡ 5 (mod 13)
b = 1, b = 5 ve b = 9 olabilir.
b = 1 için a = 0 veya a = 9 olur. 84 ≡ 1 (mod 13)

b = 5 için a = 5 olur. 85 ≡ 8 (mod 13)


b = 9 için a = 1 olur. a yerine 4 farklı rakam yazılabilir. (Cevap B)
(Cevap C)
Doğrusal Denklem Sistemlerinin Çözümleri BÖLÜM 04 Test 01
1. 3x + 2y = 1
5. 3 x + 2y − 10 = 0
2x + 3y = –6 3x − 2
olduğuna göre, y nin değeri aşağıdakilerden hangisi
denklem sisteminin çözüm kümesi aşağıdakilerden
olamaz?
hangisidir?
A) 3 B) 4 C) 5 D) 6 E) 8
A) {(–3, 4)} B) {(3, 2)} C) {(3, –4)}
2
3x – 2 = 0 ⇒ x = olamaz.
D) {(–4, 3)} E) {(2, 3)} 2 3 2
x= için 3x + 2y – 10 = 0 ⇒ 3 . ( ) + 2y – 10 = 0
3./3x + 2y = 1 3 3
⇒ y = 4 olamaz
–2/2x + 3y = –6
––––––––––––––– (Cevap B)

5x = 15 ⇒ x = 3
x = 3 için y = –4 bulunur. Ç = {(1, –4)}
(Cevap C)

6. x + y + 2z = 6

2. 2x – 3y = 5 x–y=3

–6x + 9y = 3 x2 – y2 = –12

denklem sisteminin çözüm kümesi aşağıdakilerden olduğuna göre, z kaçtır?


hangisidir? A) –6 B) –3 C) –2 D) 5 E) 6
A) {(2, –1)} B) {(1, –1)} C) {(–2, –3)} (x – y) (x + y) = –12 ⇒ 3(x + y) = –12
⇒ x + y = –4
D) ∅ E) R x + y + 2Z = 6 ⇒ –4 + 2Z = 6
2x – 3y = 5 2x – 3y = 5 ⇒ Z = 5 bulunur.

Çözüm Yayınları

–2x + 3y = 1 2x –3y = –1 (Cevap D)


(Cevap D)

7. (a – 1)x + 4y = 2
2x + (a + 1)y = 5

denklem sisteminin çözüm kümesi boş küme olduğuna


göre, a nın alabileceği değerlerin kümesi aşağıdakilerden
3. x − y − x + y = − 7 hangisidir?
3 2 3 A) {–1, 1} B) {–2, 2} C) {–3, 3}
x−y x+y
+ =3 D) {–3, 2} E) {–2, 3}
2 3
a–1 4
= ⇒ a2 – 1 = 8
olduğuna göre, x2 – y2 ifadesinin değeri kaçtır? 2 a+1
⇒ a2 = 9
A) 4 B) 6 C) 9 D) 12 E) 15
x – y = a, x + y = b olsun. ⇒ a = –3 veya a = 3
a b 7 (Cevap C)
2./ – =–
3 2 3
a b ⇒ a = 2 ve b = 6 bulunur.
3./ – =3
2 3 8. 3x + (a + 1)y = 1
73
x2 – y2 = (x – y) (x + y) = a . b = 12 dir. (b – 1)x + 2y = 6
(Cevap D)
denklem sisteminin sonsuz çözümü olduğuna göre,
a + b toplamı kaçtır?

53 55 59
A) B) 18 C) D) 19 E)
3 3 3
4. 2x − 3 y + 6 + | 3 x − y − 5 | = 0 3 1
= ⇒ b – 1 = 18
b–1 6
olduğuna göre, x + y toplamı kaçtır?
⇒ b = 19
a+1 1
A) 4 B) 5 C) 6 D) 7 E) 8 = ⇒ 6a + 6 = 2
2 6
2x – 3y = –6 ⇒ 6a = –4
2
–3./ 3x – y = 5 ⇒ a = –
–––––––––––––––– 3
–7x = –21 ⇒ x = 3 2 55
a + b = – + 19 =
x = 3 için y = 4 bulunur. x + y = 7 dir. 3 3
(Cevap C)
(Cevap D)
Test 01 1. C 2. D 3. D 4. D 5. B 6. D 7. C 8. C 9. B 10. B 11. C 12. A 13. C 14. E 15. B 16. B

9. 1 1 13. 2x – y + z = –3
+ =9
a b
x + 2y – z = 8
1 + 1 = −3
b c 3x + y + z = 2
1 + 1 = −6 olduğuna göre, x + y + z toplamı kaçtır?
a c
A) –2 B) –1 C) 0 D) 1 E) 2
olduğuna göre, b kaçtır?
2x – y + z = –3
⇒ 3x + y = 5
1 1 1 1 x + 2y – z = 8
A) B) C) D) E) 1
9 6 3 2 3x + y + z = 2 ⇒ 5 + z = 2 ⇒ z = –3 tür.
3x + y = 5
1 1 _ ⇒ x = 1 ve y = 2 bulunur.
+ =9 b 4x + 3y = 10
a b b x + y + z = 1 + 2 – 3 = 0 elde edilir.
1 1 b 2
+ = - 3 ` ⇒ = 12 (Cevap C)
b c b b
1

1 1 b ⇒b=
- - = 6b 6
a c a (Cevap B)

10. 2x + y = –4 14. Dikdörtgen biçimindeki bir tarlanın boyu 10 metre, eni 5 metre
2y + z = 8 artırılırsa alanı 1150 metrekare artıyor. Eğer boyu 5 metre ve
eni 10 metre azaltılırsa alanı 1250 metrekare azalıyor.
x + 2z = 5
Buna göre, bu tarlanın boyu kaç metredir?
denklem sisteminin çözüm kümesi aşağıdakilerden
hangisidir? A) 80 B) 85 C) 90 D) 95 E) 100
Tarlanın boyu a metre, eni b metre olsun.
A) {(–4, 2, 3)} B) {(–3, 2, 4)}
(a + 10) . (b + 5) = ab + 1150 ⇒ a + 2b = 220
C) {(–2, 3, 4)} D) {(–2, –3, 4)} (a – 5) (b – 10) = ab – 1250 ⇒ 2a + b = 260
a + 2b = 220
Çözüm Yayınları

E) {(2, 3, 4)} ⇒ a = 100 ve b = 60


2a + b = 260
Üç denklemi taraf tarafa toplanarak 3x + 3y + 3z = 9 ⇒ x + y + z = 3 bulunur. İlk iki (Cevap E)

denklem taraf tarafa çıkarılarak 2x – y – z = –12 elde edilir.


x+y+z=3
⇒ 3x – 9 ⇒ x = 3 bulunur.
2x – y – z = –12
x in bu değeri denklemlerde yerine yazılarak y = 2 ve z = 4 elde edilir.
(Cevap B)

15. Bir kayıkçı, akıntılı bir nehirde kürek çekerek 6 kilometrelik bir
yolu, akıntı yönünde 1 saatte alıyor.
11. x + 2y – z = –2 Kayıkçı, aynı yolu 2 saatte döndüğüne göre, nehrin akıntı
hızı saatte kaç kilometredir?
x – y + 2z = 7

x+y–z=0 3 5
A) 1 B) C) 2 D) E) 3
2 2
olduğuna göre, x kaçtır? Kayığın saatteki hızı V1 km, akıntının saatteki hızı V2 km olsun.

A) –3 B) –2 C) 3 D) 4 E) 5 1 . (V1 + V2) = 6 1 . (V1 + V2) = 6 3


İlk iki denklem taraf tarafa toplanarak 2x + y + z = 5 bulunur. ⇒ ⇒ V2 =
2 . (V1 – V2) = 6 2 . (V1 – V2) = 6 2
Son iki denklem taraf tarafa toplanarak 2x + z = 7 bulunur. (Cevap B)
2x + y + z = 5
⇒ y = –2 elde edilir.
2x + z = 7
74 y = –2 için x + y – z = 0 ⇒ x – 2 – z = 0
⇒ x – z = 2 bulunur. 16. a, b, c pozitif tam sayılar olmak üzere,
2x + z = 7
⇒x=3
x–z=2 a + 4b + 5c = 75
(Cevap C)
12. x – 3y + 4z = –15 a + 2b + 7c = 63

x + 2y – 3z = 9 olduğuna göre, a nın alabileceği en büyük değer kaçtır?

–2x + y – 2z = 8 A) 40 B) 42 C) 45 D) 50 E) 54
İki denklem taraf tarafa çıkarılır.
olduğuna göre, z kaçtır?
2b – 2c = 12 ⇒ b – c = 6
A) –2 B) –1 C) 1 D) 2 E) 3 ⇒b=c+6

Üç denklem taraf tarafa toplanarak c = 1 için b = 7 olur.

–z = 2 ⇒ z = –2 bulunur. a + 2b + 7c = 63 ⇒ a + 14 + 7 = 63

(Cevap A) ⇒ a = 42
(Cevap B)
Doğrusal Denklem Sistemlerinin Çözümleri BÖLÜM 04 Test 02
1. 3 + 2 = 3 5. (a + 1)x – 3y = 1
x y 2
2x + (b – 2)y = 4
2 + 3 = 17
x y 12 denklem sisteminin çözüm kümesi sonsuz elemanlı
olduğuna göre, a + b toplamı kaçtır?
olduğuna göre, x kaçtır?
21 19
A) 1 B) 2 C) 3 D) 4 E) 5 A) –11 B) − C) –10 D) − E) –9
2 2
3 2 3_
- 3./ + = b a+1 –3 1
x y 2b 5 9 17 = = olmalıdır.
` &- x =- 2 + 6 2 b–2 4
2 3 17 b a+1 1 1
2./ + = = ⇒ a = – ve
x y 12 b 2 4 2
a
5 10 –3 1
⇒ – =– = ⇒ b = –10 bulunur.
x 6 b–2 4
⇒ x = 3 1 21
a + b = – –10 = – dir.
(Cevap C) 2 2
(Cevap B)

6. ax – 2y = 5

2. 2x – y = a 3x + 4y = 3

x + 2y = b denklem sisteminin çözüm kümesi boş küme olduğuna


göre, a kaçtır?
olduğuna göre, x in a ve b cinsinden değeri
aşağıdakilerden hangisidir? 3 1 1
A) − B) − C) D) 1 E) 2
2 2 2
2a − b 2a − b
A) B) C) 2a + b a –2 3
3 5 = ⇒a=–
3 4 2
(Cevap A)
2a + b 2a + b
D) E)
3 5
Çözüm Yayınları

2./2x – y = a
⇒ 5x = 2a + b
x + 2y = b 2a + b
⇒ x =
5
(Cevap E)

7. 1+1 =5
x y
1+1=7
y z

3. 2x − y = 5 1+ 1 =6
3 z x
y
3x + =0 olduğuna göre, z kaçtır?
4
1 1 1 1 2
olduğuna göre, y kaçtır? A) B) C) D) E)
6 4 3 2 3
A) –10 B) –5 C) –1 D) 2 E) 5 Birinci denklemin işareti değiştirilerek taraf tarafa toplanırsa,
6x – y = 15 5 2 1
⇒x= ve y = –10 =8 ⇒y= bulunur.
12x + y = 0 6 y 4
(Cevap A) (Cevap B)
75

8. a + b = 8
b + c = –3

4. xy – 2x + 4y + 3 = 0 a+c=7

eşitliğinde, x in hangi değeri için y bulunamaz? olduğuna göre, a kaçtır?

A) –4 B) –2 C) 2 D) 3 E) 4 A) 7 B) 8 C) 9 D) 10 E) 11
2./2x – y = a İkinci denklem işaret değiştirilerek taraf tarafa toplanırsa,
⇒ 5x = 2a + b
x + 2y = b 2a = 18 ⇒ a = 9 bulunur.
2x + 6
⇒ x = (Cevap C)
x+4
x + 4 = 0 ⇒ x = –4 için y hesaplanamaz.
(Cevap A)
Test 02 1. C 2. E 3. A 4. A 5. B 6. A 7. B 8. C 9. C 10. B 11. D 12. B 13. A 14. C 15. C 16. E

9. x + y + z = 18 13. a, b, c pozitif gerçek sayılar olmak üzere,


xy + xz = 81 a·b=2

olduğuna göre, x kaçtır? b · c = 24

A) 3 B) 6 C) 9 D) 12 E) 15 a·c=3
y + z = 18 – x tir. olduğuna göre, a kaçtır?
x(y + z) = 81 ⇒ x(18 – x) = 81
⇒ 18x – x2 = 81 1 3
A) B) 1 C) D) 2 E) 4
2
⇒ x – 18x + 81 = 0
2 2
⇒ (x – 9)2 = 0 a2 . b2 . c2 = 2 . 24 . 3 ⇒ a2 . b2 . c2 = 4 . 6 . 6
⇒ x = 9 bulunur. ⇒ a . b . c = 12
(Cevap C) ⇒ a . 24 = 12
1
⇒a=
2
(Cevap A)

14. Üç basamaklı bir sayının rakamları toplamı 13, onlar ve birler


basamağındaki rakamların toplamı 10 dur. Sayının birler
10. (2x – y – 11)2 + (x + 2y + 2)2 = 0
ve yüzler basamağındaki rakamları yer değiştirdiğinde ilk
olduğuna göre, y kaçtır? sayıdan 99 fazla olan bir sayı elde ediliyor.

A) –4 B) –3 C) –2 D) 1 E) 2 Buna göre, bu sayı kaçtır?


2./2x – y = 11
⇒ x = 4 ve y = –3 bulunur. A) 348 B) 352 C) 364 D) 372 E) 384
x + 2y = –2
(Cevap B) Üç basamaklı sayı abc olsun.
a + b + c = 13
⇒a=3
b + c = 10
cba = abc + 99 ⇒ 100c + 10b + a = 100a + 10b + c + 99
⇒ 99c = 99a + 99
Çözüm Yayınları

⇒c=a+1
⇒c=4
a + b + c = 13 ⇒ 3 + b + 4 = 13
⇒ b = 6
Sayı abc = 364 tür.
(Cevap C)

15. x+y 1
=
11. 2x + 5y + 3z = –8 xy 6
y+z
3x + 2y – z = –1 =−1
yz 6
x – 3y + 2z = 13
x+z = 2
olduğuna göre, x kaçtır? xz 3
olduğuna göre, x kaçtır?
A) –3 B) –2 C) 1 D) 2 E) 3
İkinci denklem işaret değiştirilerek taraf tarafa toplanır. A) –5 B) –2 C) 2 D) 3 E) 5
6z = 6 ⇒ z = 1 bulunur. 1 1 1 _
+ = b
3x + 2y – 1 = –1 3x + 2y = 0 y x 6 b

x – 3y + 2 = 13 x – 3y = 11 1 1 1b 2
1./ + = - ` & = 1 & x = 2
⇒ x = 2 elde edilir. z y 6b x
(Cevap D) 1 1
2 b
+ = b
z x 3 (Cevap C)
76 a
16. İki alaşımdan birincisinde bulunan bakır miktarı gümüş
miktarının 3 katı, ikinci alaşımda ise gümüş miktarı bakır
miktarının 5 katıdır. Her iki alaşımdan da birer miktar alınarak,
bakır miktarı gümüş miktarının iki katı olacak şekilde 140
gramlık yeni bir alaşım yapılıyor.
12. x + y – z = 5
Buna göre, birinci alaşımdan kaç gram alınmıştır?
2x + y – 3z = 12
A) 60 B) 75 C) 90 D) 105 E) 120
olduğuna göre, y + z toplamı kaçtır?
Alınan Bakır Gümüş
A) –4 B) –2 C) 1 D) 2 E) 3 1. 3x x
2. y 5y
Birinci denklemin her iki yanı –2 ile çarpılarak, iki denklem taraf tarafa toplanır.
4x + 6y = 140 ⇒ 2x + 3y = 70
–2x – 2y + 2z = –10
⇒ –y – z = 2 3x + y = 2 . (x + 5y) ⇒ x = 9y
2x + y – 3z = 12 10
2(9y) + 3y = 70 ⇒ y = ve x = 30 gram
⇒ y + z = –2 bulunur. 3
1. alaşımdan 4x = 120 gram alınmıştır.
(Cevap B) (Cevap E)
Doğrusal Denklem Sistemlerinin Çözümleri BÖLÜM 04 Test 03
1. 4x – 3y = 12 5. 4x – 3y = 2x + 3y = 12
2x + 5y = –7 olduğuna göre, y kaçtır?

olduğuna göre, x kaçtır? 2 4 7


A) B) 1 C) D) 2 E)
3 3 3
1 3
A) –2 B) –1 C) D) E) 2
2 2 4x – 3y = 12
⇒ 6x = 24
4x – 3y = 12 4x + 6 = 12 2x + 3y = 12
⇒x=4
4
+ –4x – 10y = + 14 4x = 6 2 . 4 + 3y = 12 ⇒ y =
–––––––––– –––– 6 3 3
–13y = 26 x= =
4 2 (Cevap C)
y = –2 (Cevap D)

2. x − y = −1
2 3 6. 2x – y + 8z = 3

x+y =8 x + 2y – z = 4
3 2
olduğuna göre, x + y + z toplamı kaçtır?
olduğuna göre, x + y toplamı kaçtır?
A) 3 B) 4 C) 6 D) 9 E) 11
A) 6 B) 9 C) 12 D) 15 E) 18 2x – y + 8z = 3
⇒ 5x + 5y + 5z = 15
3x 2x 6 y 3./x + 2y – z = 4
+ = –3 + 16 + =8 ⇒x+y+z=3
2 3 3 2
(Cevap A)
13x y
= 13 =6
6 2
x= 6 y = 12
Çözüm Yayınları

x + y = 18 elde edilir.
(Cevap E)

3. 12 − 18 = 13
x y 7. (a + 1)x + 2y = 7
24 + 6 = 5 (2a – 1)x + 3y = 2
x y
denklem sisteminin çözüm kümesi boş küme olduğuna
olduğuna göre, x kaçtır? göre, a kaçtır?
A) –3 B) –2 C) 3 D) 4 E) 5 A) –3 B) –2 C) 1 D) 4 E) 5
12 18
– = 13
x y 84 a+1 2
⇒ = 28 + ⇒ 4a – 2 = 3a + 3
24 6 x 2a – 1 3
3./ – =5 ⇒a=5
x y
⇒ x = 3 (Cevap E)
(Cevap C)

77

4. ab + 1 = 12
c

ac + 1 = 15 8. 2(x + 3y) = x – y
b
b + c = 18 3x + 2y = 19

olduğuna göre, b kaçtır? olduğuna göre, x + y toplamı kaçtır?

A) 6 B) 8 C) 9 D) 10 E) 12 A) 4 B) 5 C) 6 D) 7 E) 8
abc + 1 = 12c 2x + 6y – x + y = 0 ⇒ x + 7y = 0
⇒ 12c = 15b
abc + 1 = 15b 5b x + 7y = 0
⇒c= ⇒ 19x + 19y = 6 . 19
4 6./3x + 2y = 19
⇒x+y=6
5b (Cevap C)
b + c = 18 ⇒ b + = 18 ⇒ b = 8
4 (Cevap B)
Test 03 1. D 2. E 3. C 4. B 5. C 6. A 7. E 8. C 9. C 10. A 11. C 12. D 13. C 14. B 15. C 16. A

9. 3−1=7 13. x + z = 1
x y x + y + 2z = –3
3 − 1 = −5
x + 2y + z = –5
y z
3−1=4 olduğuna göre, z kaçtır?
z x
A) –7 B) –5 C) –1 D) 4 E) 5
olduğuna göre, y kaçtır?
x + 2y + z = –5 ⇒ 2y + 1 = –5
1 ⇒ y = –3
A) –3 B) –2 C) –1 D) E) 1
3 1 _ 2 x – 3 + 2s = –3 ⇒ x + 2z = 0
3./ - = - 5 bb
y z 9 1 x + 2z = 0
` & y - x = - 11 ⇒ x = 2 ve z = –1
(Cevap C)
3 1 x+z=1
- = 4 bb
z x a

3 1 _
- =7 b
x y b 26
` & y - 26
9 1
3./ - = - 11bb
14. Aynı yol üzerinde hareket eden A ve B araçları arasındaki
y x uzaklık 300 kilometredir. Bu araçlar, aynı anda ve aynı yönde
a
& y = - 1 bulunur. (Cevap C) hareket ettiğinde A aracı, B ye 10 saat sonra yetişiyor. Yine
aynı anda yola çıkıp karşılıklı hareket ettiklerinde ise, 3 saat
10. (5x – 2y + 3) · a + (2x + y + 12) · b = 0
sonra karşılaşıyorlar.
eşitliği, a nın ve b nin her gerçek sayı değeri için
Buna göre, A aracının hızı saatte kaç kilometredir?
sağlandığına göre, y kaçtır?
A) 60 B) 65 C) 70 D) 72 E) 80
A) –6 B) –4 C) –1 D) 2 E) 3
5x – 2y + 3 = 0 A aracının hızı V1, B aracının hızı V2 olsun.
⇒ 9x = –27
2./2x + y + 12 = 0 10V1 – 10V2 = 300 ⇒ V1 – V2 = 30
⇒ x = –3 tür. 3V1 + 3V2 = 300 ⇒ V1 + V2 = 100
Çözüm Yayınları

2x + y + 12 = 0 ⇒ –6 + y + 12 = 0 Denklem sisteminin çözümünden V1 = 65 km/saat bulunur.


⇒ y = –6 bulunur. (Cevap B)
(Cevap A)

11. x – y + z = 2 15. Bir adam, parasının bir kısmını yıllık olarak %6 dan, bir
kısmını %8 den ve kalanını da %9 dan faize veriyor.
x+y+z=6
Hepsinden bir yılda aldığı toplam faiz 692 lira ve %6 dan
x + 2y = 5 bankaya yatırılan paranın bir yılda getirdiği faiz, diğer
ikisinin bir yılda getirdiği toplam faizden 88 lira fazladır. Eğer
olduğuna göre, z kaçtır?
parasının tamamını %7 den faize vermiş olsaydı bir yılda 8
A) 1 B) 2 C) 3 D) 4 E) 5 lira fazla faiz almış olacaktı.
x–y+z=2
⇒ 2x + 2z = 8 Buna göre, bu adamın yıllık %6 den faize verdiği para kaç
x+y+z=6 liradır?
⇒x+z=4
x+y+z=6⇒4+y=6 A) 6000 B) 6200 C) 6500 D) 6800 E) 7000
⇒ y = 2 6a 8b 9c 6a 8b 9c
= + + 88 ⇒ – 88 = +
x + 2y = 5 ⇒ x + 4 = 5 100 100 100 100 100 100

⇒ x = 1 6a 8b 9c 6a 6a
= + = 692 ⇒ + – 88 = 692
100 100 100 100 100
x+y+z=6⇒1+2+z=6
12a
⇒ z = 3 ⇒ = 780
100
(Cevap C) ⇒ a = 6500
78
(Cevap C)

12. x + y = 7
16. a, b, c gerçek sayılar ve a ≠ b olmak üzere,
2x – y + z = 6
a = bc + 4
x + y + 2z = 9
b = ac + 4
olduğuna göre, 3x + y + z toplamı kaçtır?
olduğuna göre, a + b + c toplamı kaçtır?
A) 13 B) 14 C) 15 D) 16 E) 17
A) 3 B) 5 C) 7 D) 9 E) 11
x + y + 2z = 9 ⇒ 7 + 2z = 9 a–4 b–4 2 2
= = c ⇒ a – 4a = b – 4b
⇒z=1 a a
⇒ a2 – b2 – 4(a – b) = 0
2x – y + z = 6 ⇒ 2x – y = 5 ⇒ (a – b) (a + b – 4) = 0
x+y=7 ⇒a+b=4
⇒ x = 4 ve y = 3 a + b = c(a + b) + 8 ⇒ 4 = 4c + 8
2x – y = 5
3x + y + z = 12 + 3 + 1 = 16 ⇒ c = –1
a + b + c = 4 + (–1) = 3 tür. (Cevap A)
(Cevap D)
İkinci Dereceden Denklemler BÖLÜM 04 Test 04
5. x2 – 8x + m + 4 = 0
1. 2x = 5
x +1 x−2
denkleminin kökleri x1 ve x2 dir.
denkleminin çözüm kümesi aşağıdakilerden hangisidir?
x12 – x22 = –16
A) −5,
1
2
{ } {
B) −3,
3
2
} { }
C) −1,
5
2
olduğuna göre, m kaçtır?

{ } {21 , 5}
A) 7 B) 8 C) 9 D) 10 E) 11
1
D) − , 5 E) (x1 – x2) (x1 + x2) = –16 ⇒ (x1 – x2) . 8 = –16
2
⇒ x1 – x2 = –2
2x2 – 4x = 5x + 5 ⇒ 2x2 – 9x – 5 = 0 x1 – x2 = –2
⇒ x1 = 3 ve x2 = 5 bulunur.
⇒ (2x + 1) (x – 5) = 0 x1 + x2 = 8
1
⇒ x = – veya x = 5 m + 4 = x1 . x2 ⇒ m + 4 = 15
2 ⇒ m = 11 elde edilir.
(Cevap D)
(Cevap E)

2. (m + 2)x2 – (3m + 5)x + 2m + 3 = 0


6. Köklerinden biri 3 − 2 5 olan rasyonel katsayılı ikinci
denkleminin köklerinin geometrik ortalaması 2 olduğuna
dereceden denklem aşağıdakilerden hangisidir?
göre, aritmetik ortalaması kaçtır?
A) x2 – 6x – 9 = 0 B) x2 – 6x – 11 = 0
3 5 7
A) B) 2 C) D) 3 E)
2 2 2 C) x2 – 6x + 9 = 0 D) x2 + 6x – 11 = 0

2m + 3 2m + 3 E) x2 + 6x – 13 = 0
=2& =4
m+2 m+2
x1 = 3 – 2ñ5
5 ⇒ x1 + x2 = 6 ve x1 . x2 = –11
& m =- x2 = 3 + 2ñ5
2
x2 – (x1 + x2)x + x1 . x2 = 0 ⇒ x2 – 6x – 11 = 0
Çözüm Yayınları

x1 + x2 1 3m + 5 5
= . = (Cevap B)
2 2 m+5 2
(Cevap C)

3. m ≠ 0 olmak üzere,
(m + 1)x2 – 4mx + 3m = 0
7. x2 – 4x + a = 0
denkleminin kökleri x1 ve x2 dir.
denkleminin kökleri x1 ve x2 dir.
x12 + x22 = 2x1x2
3x1 – x2 = 8
olduğuna göre, m kaçtır?
olduğuna göre, a kaçtır?
A) –3 B) –2 C) –1 D) 2 E) 3
x12 – 2x1x2 + x22 = 0 ⇒ (x1 – x2)2 = 0
A) 1 B) 2 C) 3 D) 4 E) 5
x1 + x 2 = 4
⇒ x1 = x 2
⇒ x1 = 3 ve x2 = 1
∆ = 0 ⇒ 16m2 – 4(m + 1) . 3m = 0 3x1 – x2 = 8

⇒ 16m2 – 12m2 – 12m = 0 a = x1 . x2 = 3 bulunur.

⇒ 4m(m – 3) = 0 (Cevap C)

m = 3 bulunur. (m ≠ 0)
(Cevap E) 79

4. x2 – 3x + m + 1 = 0
denkleminin kökleri x1 ve x2 dir.

x1 + x 2 = 3
8. 2x2 + 8x – 3 = 0
olduğuna göre, m kaçtır? denkleminin kökleri x1 ve x2 dir.

A) –5 B) –3 C) 4 D) 6 E) 8 Buna göre, x 12 + x 2 2 ifadesinin değeri kaçtır?

A) 18 B) 19 C) 20 D) 21 E) 22
(ñx1 + ñx2)2= 32 ⇒ x1 + 2óx1.x2 + x2 = 9 x12 + x22 = (x1 + x2)2 – 2x1 . x2
3
⇒ 3 + 2óm+1 = 9 = (–4)2 – 2 . (– )
2
⇒ m = 8 bulunur. = 19

(Cevap E) (Cevap B)
Test 04 1. D 2. C 3. E 4. E 5. E 6. B 7. C 8. B 9. A 10. A 11. A 12. C 13. D 14. C 15. A 16. D

9. x2 – (a + b)x + a · b = 0 13. x2 + (4m – 1)x + 3m + 1 = 0


denkleminin köklerinden biri aşağıdakilerden hangisidir? denkleminin kökleri x1 ve x2 dir.
a
A) a B) –b C) 0 D) a · b E) (x1 – 1) (x2 – 1) = 15
b
x = a için a2 – (a + b) . a + a . b = a2 – a2 – a . b + a . b = 0 olduğuna göre, m kaçtır?
x = a denklemi sağladığından denklemin köküdür.
1
(Cevap A) A) –2 B) –1 C) − D) 2 E) 3
2
(x1 – 1) (x2 – 1) = 15 ⇒ x1 . x2 – (x1 + x2) + 1 = 15
⇒ 3m + 1 – (–4m + 1) = 14
⇒m=2
(Cevap D)

10. x2 – 2x + m + 3 = 0 14. x2 + (3 – p)x + 4q = 0


denkleminin kökleri x1 ve x2 dir. denkleminin kökleri sıfırdan farklı p ve q sayılarıdır.
x22 – x12 = 12 Buna göre, p – q farkı kaçtır?
olduğuna göre, m kaçtır? A) –7 B) –4 C) 7 D) 9 E) 10
A) –11 B) –7 C) –4 D) 3 E) 8 p . q = 4q ⇒ p = 4 ve

(x2 – x1) (x2 + x1) = 12 ⇒ (x2 – x1) . 2 = 12 p + q = –3 + q ⇒ q = –3 bulunur.

⇒ x2 – x 1 = 6 p – q = 4 – (–3) = 7 dir.
x2 + x 1 = 2
⇒ x2 = 4 ve x1 = –2 dir. (Cevap C)
x2 – x1 = 6
Çözüm Yayınları

m + 3 = x1 . x2 ⇒ m + 3 = –8
m = –11 bulunur.
(Cevap A)

15. x2 – 4x + 3 = 0
x2 – ax – 4 = 0

II. dereceden denklemlerinin birer kökleri eşit olduğuna


göre, a aşağıdakilerden hangisi olabilir?
11. x2 – 9x – 52 = 0
A) –3 B) –2 C) –1 D) 0 E) 1
denkleminin kökleri x1 ve x2 olduğuna göre, x2 – 4x + 3 = 0 ⇒ x = 1 veya x = 3 tür.
Bu köklerden biri ikinci denklemin kökü olmalıdır.
x12 · x2 + x1 · x22
x = 1 için 1 – a – 4 = 0 ⇒ a = –3 bulunur.
ifadesinin değeri kaçtır?
(Cevap A)
A) –468 B) –234 C) –52 D) 52 E) 468
x1 . x2 (x1 + x2) = –52 . 9
= –468
(Cevap A)

80 16. x1 ve x2 kökleri arasında,


2(x1 + x2) + x1x2 = –11

3(x1 + x2) – 2x1x2 = 36

bağıntısı bulunan ikinci dereceden denklem


aşağıdakilerden hangisidir?

A) x2 + 3x – 10 = 0 B) x2 – 3x – 10 = 0
12. Karesinin iki katının dört eksiği, kendisinin 7 katına eşit
olan sayıların çarpımı kaçtır? C) x2 – 2x – 3 = 0 D) x2 – 2x – 15 = 0

A) –4 B) –3 C) –2 D) –1 E) 0 E) x2 + 2x – 15 = 0
2x2 – 4 = 7x ⇒ 2x2 – 7x – 4 x1 + x2 = m, x1 . x2 = n olsun
–4 2m + n = –11
x1 . x 2 = = –2 ⇒ m = 2 ve n = –15 bulunur.
2 3m – 2n = 36
(Cevap C)
x2 – mx + n = 0 ⇒ x2 – 2x – 15 = 0
(Cevap D)
İkinci Dereceden Denklemler BÖLÜM 04 Test 05
2 5. x2 – 10x + a = 0
1. ( x − 5 x − 15) ( x − 4) = 0
x+3 denkleminin kökleri x1 ve x2 dir.
eşitliğini sağlayan x reel (gerçek) sayılarının toplamı (x1 + x2) · (2x1 + x2) = 110
kaçtır?
olduğuna göre, a kaçtır?
A) 9 B) 8 C) 6 D) 5 E) 2
((x2 – 5x – 15) (x – 4) = 0 ⇒ x2 – 5x – 15 = 0 veya x = 4
A) 5 B) 6 C) 7 D) 8 E) 9
(x1 + x2) (x1 + x1 + x2) = 110 ⇒ 10 . (x1 + 10) = 110
Birinci denklemin iki kökünün toplamı 5 tir.
⇒ x1 + 10 = 11
Üç bölüm toplamı 5 + 4 = 9 olur.

⇒ x1 = 1
(Cevap A)
x1 = 1 denklemi sağlar.
1 – 10 + a = 0 ⇒ a = 9 bulunur.
(Cevap E)

2
2. ( x + 1)( x − 6 x + 1) = 0
2
x +x
6. x2 – 2ax + a2 – 4 = 0
olduğuna göre, x 2 + 1 ifadesinin değeri kaçtır? denkleminin köklerinden biri aşağıdakilerden hangisidir?
x2
A) 32 B) 34 C) 36 D) 38 E) 40 A) a B) 2a C) a – 1

(x + 1) (x2 – 6x + 1) 1 D) a + 1 E) a + 2
=0⇒x+ =6
x(x + 1) x x2 – 2a + a2 – 4 = 0
1
⇒ x2 + = 34 x1 . x2 = (a – 2) (a + 2)
x2
(Cevap B) x1 + x2 = (a – 2) + (a + 2) = 2a
olduğundan a + 2 denklemin bir köküdür.
Çözüm Yayınları

(Cevap E)

3. x2 + 6x – 3 = 0
denkleminin kökleri x1 ve x2 olduğuna göre,
7. x2 – (15 – m)x + 6n = 0
x1 x 2
+
x 2 x1 denkleminin kökleri sıfırdan farklı m ve n sayıları

olduğuna göre, n kaçtır?
ifadesinin değeri kaçtır?
3 6 12
1 1 1 A) B) C) 2 D) E) 3
A) –14 B) –2 C) − D) − E) − 5 5 5
2 5 10
x12 + x22 (x1 + x2) – 2x1 . x2 m . n = 6n ⇒ m = 6
=
x1 . x 2 x1 . x 2 m + n = 15 – m ⇒ 6 + n = 15 – 6
(–6)2 – 2(–3)
= ⇒n=3
–3
= –14 (Cevap E)

(Cevap A)

4. 3x2 – 4x – 2 = 0

denkleminin kökleri x1 ve x2 dir. 81


Kökleri (2x1 – 1) ve (2x2 – 1) olan ikinci dereceden
8. 3x2 – 6x – 4 + k = 0
denklem aşağıdakilerden hangisidir?
denkleminin kökleri x1 ve x2 dir.
A) 3x2 – 4x – 7 = 0 B) 2x2 – 3x + 13 = 0
3x12 + 7x1x2 + 4x22 = 16
C) 2x2 + 2x + 13 = 0 D) 3x2 + 2x + 13 = 0
olduğuna göre, k kaçtır?
E) 3x2 – 2x – 13 = 0
a = 2x1 – 1
A) 3 B) 4 C) 6 D) 9 E) 10
b = 2x2 – 1
4 2 3x12 + 3x22 + 6x1 . x2 + x1 . x2 + x22 = 16
a + b = 2(x1 + x2) – 2 = 2 . ( ) – 2 =
3 3 3(x12 + 2x1 . x2 + x22) + x2(x1 + x2) = 16
a . b = (2x1 – 1) (2x1 – 1) = 4x1x2 – 2(x1 + x2) + 1
3(x1 + x2)2 + x2 (x1 + x2) = 16
2 4
= 4(– ) –2( ) + 1 3 . 4 + x2 . 2 = 16 ⇒ x2 = 2 bulunur.
3 3
13 x2 = 2 denklemi sağlar.
= –
3 3 . 22 – 6 . 2 – 4 + k = 0 ⇒ k = 4 tür.
2 13
x2 – x – = 0 ⇒ 3x2 – 2x – 13 = 0 (Cevap E) (Cevap B)
3 3
Test 05 1. A 2. B 3. A 4. E 5. E 6. E 7. E 8. B 9. C 10. C 11. A 12. C 13. E 14. A 15. A 16. A

9. 3x2 – (m + 4)x + m – 8 = 0 13. x2 + 5x – 2m + 5 = 0


denkleminin simetrik iki kökü olduğuna göre, büyük kökü denkleminin kökleri x1 ve x2 dir.
kaçtır?
x12 – x22 = –15
A) –2 B) –1 C) 2 D) 3 E) 4
olduğuna göre, m kaçtır?
x1 = –x2 ⇒ x1 + x2 = 0
m+4
⇒ =0 1 1 1
3 A) –2 B) − C) − D) 0 E)
⇒ m = –4 bulunur. 2 3 2
3x2 – 4 – 8 = 0 ⇒ 3x2 = 12 (x1 – x2)(x1 + x2) = –15 ⇒ (x1 – x2) . (–5) = –15
⇒ x2 = 4 ⇒ x1 – x 2 = 3
⇒ x1 = –2, x2 = 2 x1 – x 2 = 3
⇒ x1 = –1 ve x2 = –4
(Cevap C) x1 + x2 = –5

–2m + 5 = x1 . x2 ⇒ –2m + 5 = 4
1
⇒m=
10. Köklerinden biri 1 + 3 olan rasyonel katsayılı ikinci 2
(Cevap E)
dereceden denklem aşağıdakilerden hangisidir?

A) x2 + 2x – 2 = 0 B) x2 + 2x + 2 = 0

C) x2 – 2x – 2 = 0 D) x2 – 2x + 2 = 0 14. x2 + ax + b = 0 denkleminin bir kökü 3,

E) x2 – x + 2 = 0 x2 – cx + d = 0 denkleminin bir kökü –2


x1 = 1 x ñ3 dir.
⇒ x1 + x2 = 2, x1 . x2 = –2
x2 = 1 – ñ3
x2 – (x1 + x2)x + x1 . x2 = 0 ⇒ x2 – 2x – 2 = 0
Bu iki denklemin diğer kökleri eşit olduğuna göre,
(Cevap C)
a + c toplamı kaçtır?

A) –5 B) –2 C) 1 D) 4 E) 5
Eşit kök a olsun
Çözüm Yayınları

3 + a = –a –3 – a = a

–2 + a = c + –2 + a = + c
––––––– –––– (Cevap A)
–5 = a + c
11. x2 – mx + 36 = 0
denkleminin kökleri x1 ve x2 dir.
3 + x =9
2
x1 2

olduğuna göre, m kaçtır?


15. x2 + mx + 24 = 0
A) 13 B) 12 C) 11 D) 10 E) 9
denkleminin kökleri birer tam sayıdır.
9ñx1 9ñx1
3 + óx1.x2 = ⇒ 3 + ò36 =
2 2 Buna göre, m nin alabileceği kaç farklı tam sayı değeri
⇒ ñx1 = 2 vardır?
⇒ x1 = 4 tür.
A) 8 B) 10 C) 12 D) 14 E) 16
x = 4 için, 16 – 4m + 36 = 0 ⇒ m = 13 bulunur.
x1 . x2 = 1 . 24 ⇒ x1 + x2 = 25 ⇒ m = –25
(Cevap A)
x1 . x2 = 2 . 12 ⇒ x1 + x2 = 14 ⇒ m = –14
x1 . x2 = 3 . 8 ⇒ x1 + x2 = 11 ⇒ m = –11
x1 . x2 = 4 . 6 ⇒ x1 + x2 = 10 ⇒ m = –10
Bunun gibi m nin 4 tane de pozitif değeri vardır.
82 m nin alabileceği 8 farklı tam sayı değeri vardır.
12. İkinci dereceden (Cevap A)

ax2 + bx + c = 0

denkleminin kökleri x1 ve x2 dir. 16. ax2 + bx + c = 0

a+b+c=0 denkleminin kökleri –3 ve 5 olduğuna göre,

x1 · x2 = 4 a(2x – 1)2 + b(2x – 1) + c = 0

olduğuna göre, x1 + x2 toplamı kaçtır? denkleminin köklerinin çarpımı kaçtır?

A) 2 B) 4 C) 5 D) 6 E) 8 A) –3 B) –2 C) 2 D) 3 E) 6
x = 1 için a + b + c = 0 olduğundan denklemin köklerinden biri x1 = 1 dir. 2x – 1 = –3 ⇒ x = –1

x1 . x2 = 4 ⇒ 1 . x2 = 4 2x – 1 = 5 ⇒ x = 3
Kökler çarpımı –3 olur.
⇒ x2 = 4 bulunur.
(Cevap A)
x1 + x2 = 1 + 4 = 5 tir.
(Cevap C)
İkinci Dereceden Denklemler BÖLÜM 04 Test 06
5. a ≠ 0 olmak üzere,
1. 2x = 10 + 1
x
ax2 + (a + 1)x + a = 0
denkleminin çözüm kümesi aşağıdakilerden hangisidir?
denkleminin köklerinin aritmetik ortalaması geometrik
A) −2,
1
2
{ } { }
B) −2,
5
2
{ 5
C) − , 2
2 } ortalamasına eşit olduğuna göre, denklemin köklerinin
toplamı kaçtır?


1
{
D) − , 2
2 } E) {–2, 3}
x1 + x 2
A) –2 B) –1
–(a + 1) a
C) 1 D) 2 E) 3
= óx1.x2 ⇒ =
2x2 = 10 + x ⇒ 2x2 – x – 10 = 0 2 2a a
⇒ (2x – 5) (x + 2) = 0 –(a + 1)
⇒ =1
5 2a
⇒x= veya x = –2
2 1
(Cevap B) ⇒a=–
3
–(a + 1)
x1 + x 2 = ⇒ x1 + x2 = 2 bulunur.
a
(Cevap D)

2. (m + 1)x2 – (m – 1)x – 3m + 1 = 0 6. x2 – 3x + m + 1 = 0

denkleminin kökleri x1 ve x2 dir. denkleminin kökleri p ve q dur.

x1 x 2 q
+ = −2 p=
x 2 x1 2

olduğuna göre, m kaçtır?
olduğuna göre, x1 · x2 kaçtır?
1 3
A) –2 B) –1 C) 1 D) 2 E) 3 A) B) 1 C) 2 D) E) 4
x12 + x22 2 2
= –2 ⇒ x12 + x22 = –2x1x2
x1 . x 2 q
p= ⇒ 2p = q
⇒ (x1+ x2)2 = 0 2
Çözüm Yayınları

m–1
⇒ =0 p + q = 3 ⇒ p + 2p = 3
m+1
⇒m=1 ⇒p=1
Denklem: 2x2 – 2x – 2 = 0 ⇒ x2 – x – 1 = 0 p = 1 denklemi sağlar.
x1 . x2 = –1 bulunur. 1 – 3 + m + 1 = 0 ⇒ m = 1 bulunur.
(Cevap B) (Cevap B)

1 7. ax2 – 4x – 10 = 0
3. Köklerinden biri olan rasyonel katsayılı ikinci
2− 3
(a + 1)x2 – 6x + b = 0
dereceden denklem aşağıdakilerden hangisidir?
denklemlerinin çözüm kümeleri aynı olduğuna göre,
A) x2 + 4x + 1 = 0 B) x2 – 4x + 1 = 0
a + b toplamı kaçtır?
C) x2 – 2x + 3 = 0 D) x2 + 2x + 3 = 0
A) –15 B) –13 C) –6 D) 10 E) 15
E) x2 – 4x + 3 = 0
a –4 –10
2 + ñ3 = =
x1 = = 2 + ñ3 a+1 –6 b
1 a 2
x2 = 2 – ñ3 olur. = ⇒ a = 2 ve
a+1 3
x2 – 4x + 1 = 0 denkemi elde edilir. a –4
= ⇒ b = –15 tir.
(Cevap B) a + 1 –6
a + b = 2 – 15 = –13 bulunur.
83
(Cevap B)

8. x2 + ax + b = 0 denkleminin bir kökü –18,


4. x2 – 3x + m – 10 = 0
x2 + cx + d = 0 denkleminin bir kökü 6
denkleminin kökleri x1 ve x2 dir.
dır.
|x1 – x2| = 11 b
Bu iki denklemin diğer kökleri eşit olduğuna göre,
d
olduğuna göre, m kaçtır? oranı kaçtır?

A) –20 B) –18 C) –15 D) 12 E) 15 A) –6 B) –4 C) –3 D) 2 E) 3


ñ∆ eşit kök a olsun.
= 11 ⇒ 9 – 4(m – 10) = 11
|a| –18 . a = b
⇒ 9 – 4m + 40 = 121 b
⇒ =3
6.a=d d
⇒ m = –18 (Cevap C)
(Cevap B)
Test 06 1. B 2. B 3. B 4. B 5. D 6. B 7. B 8. C 9. B 10. B 11. B 12. A 13. D 14. D 15. A 16. E

9. x2 – x – 3 = 0 13. x2 – 4x + a = 0
denkleminin kökleri x1 ve x2 dir. denkleminin kökleri x1 ve x2 dir.

Kökleri 2x1 + 1 ve 2x2 + 1 olan ikinci dereceden denklem x13 + x23 = 16


aşağıdakilerden hangisidir?
olduğuna göre, a kaçtır?
A) x2 – 4x – 17 = 0 B) x2 – 4x – 9 = 0
A) 1 B) 2 C) 3 D) 4 E) 5
C) x2 – 4x + 17 = 0 D) x2 – 17x – 4 = 0 x13 + x23 = 16 ⇒ (x1 + x2)3 – 3x1x2 (x1 + x2) = 16
⇒ 64 . 3a . 4 = 16
E) x2 + 17x + 4 = 0
⇒ a = 4 bulunur.
a = 2x1 + 1 ve b = 2x2 + 1 olsun
(Cevap D)
a + b = 2(x1 + x2) + 2 = 2 . 1 + 2 = 4
a . b = (2x1 + 1) (2x2 + 1)
= 4x1 . x2 + 2(x1 + x2) + 1
= 4 . (–3) + 2(1) + 1
= –9
x2 – (a + b)x + a . b = 0 ⇒ x2 – 4x – 9 = 0
14. Kökleri x1 ve x2 olan bir ikinci dereceden denklemin kökleri
(Cevap B)
arasında,
10. (a – 2)x2 – (a + 1)x – 7 = 0 x1x2 – 2(x1 + x2) = –5
ikinci dereceden denkleminin kökleri x1 ve x2 dir. 3(x1 + x2) – 2x1x2 = –2

2 x1 + x 2 = 3 bağıntıları vardır.
a−2
Buna göre, bu koşullara uygun ikinci dereceden denklem
olduğuna göre, a kaçtır? aşağıdakilerden hangisidir?
A) 3 B) 4 C) 5 D) 6 E) 7 A) x2 – 19x + 12 = 0 B) x2 + 12x + 19 = 0
3 3
2x1 + x2 = ⇒ x1 + (x1 + x2) = C) x2 – 5x + 12 = 0 D) x2 – 12x + 19 = 0
Çözüm Yayınları

a–2 a–2

a+1 3 E) x2 + 5x + 12 = 0
⇒ x1 + =
a–2 a–2
x1 + x2 = a ve x1 . x2 = b olsun.
3 a+1
⇒ x1 = – = –1 b – 2a = –5
a–2 a–2 ⇒ a = 12 ve b = 19 bulunur.
3a – 2b = –2
x = –1 denklemi sağlar.
x – ax + b = 0 ⇒ x2 – 12x + 19 = 0
2

(a – 2)(–1)2 – (a + 1) (–1) – 7 = 0 ⇒ a = 4 bulunur. (Cevap D)


(Cevap B)

11. 2x2 + (m + 3)x + 12 = 0


15. a(x + 1)2 + b(x + 1) + c = 0
denkleminin kökleri x1 ve x2 dir.
denkleminin çözüm kümesi {–8, 2} olduğuna göre,
x12 · x2 = 12
a(2x – 1)2 + b(2x – 1) + c = 0
olduğuna göre, m kaçtır?
denkleminin çözüm kümesi aşağıdakilerden hangisidir?
A) –15 B) –13 C) –7 D) 5 E) 12
A) {–3, 2} B) {–4, 1} C) {–2, 3}
12
x1 . x1 . x2 ⇒ 12 ⇒ x1 . ( ) = 12
2 D) {–1, 4} E) {–2, 4}
⇒ x1 = 2 x = –8 ⇒ x + 1 = –7
x1 = 2 denklemi sağlar. ⇒ 2x – 1 = –7
8 + 2(m + 3) + 12 = 0 ⇒ m = –13 ⇒ x = –3
x = 2 ⇒ x + 1 = 3
(Cevap B)
84 ⇒ 2x – 1 = 3
⇒x=2
Ç = {–3, 2} bulunur. (Cevap A)

16. 3( x + y ) x + y + 1
12. x2 + 2x + 4 = 0 + =1
x + y − 2 x + y −1
denkleminin bir kökü a dır.
olduğuna göre, x + y toplamının pozitif değeri kaçtır?
(a2 + 2a + 5) · (a2 + 2a + 10)
1 2 3 4
A) B) C) D) 1 E)
ifadesinin değeri kaçtır? 3 3 4 3
x + y = a olsun.
A) 6 B) 7 C) 8 D) 9 E) 10 3a a+1 3a2 – 3a + a2 – a – 2
+ = 1 ⇒
x = a denklemi sağlar. a2 + 2a + 4 = 0 ⇒ a2 + 2a = –4 a–2 a–1 a2 – 3a + 2
(a2 + 2a + 5)(a2 + 2a + 10) = (–4 + 5)(–4 + 10) = 6 (a – 1) (a – 2)
⇒ 3a2 – a – 4 = 0
(Cevap A) ⇒ (3a – 4)(a + 1) = 0
4
⇒ a = veya a = –1
3 (Cevap E)
Karmaşık Sayılar BÖLÜM 04 Test 07
1. (1 – i17) · (1 – i18) · (1 + i19) 5. i2 = –1 ve a, b ∈ R olmak üzere,
işleminin sonucu aşağıdakilerden hangisidir? 2 − 4i = a + bi
1 + i
A) –4i B) –2i C) 2i D) 4i E) 8i
(1 – i)(1 – (–1))(1 + i3) = (1 – i) . 2 . (1 – i) olduğuna göre, a + b toplamı kaçtır?
= (1 – i)2 . 2
A) –4 B) –2 C) 2 D) 4 E) 6
= –2i . 2
= –4i 2 – 4i 2 – 4i – 2i + 4i2
= a + bi ⇒ = a + bi
(Cevap A) 1+i 2
(1 – i)
–2 – 6i
⇒ = a + bi
2
⇒ –1 – 3i = a + bi

a + b = –1 – 3 = –4 bulunur.
(Cevap A)

30
2.  1 + i  6. i2 = –1 olmak üzere,
 1− i 
(1 + i)6
işleminin sonucu aşağıdakilerden hangisidir?
işleminin sonucu aşağıdakilerden hangisidir?
A) –1 B) –i C) 1 D) i E) 1 + i
A) –8i B) –8 C) 6i D) 8 E) 8i
^1 + i h2
15
= G = ` –22i i j = ^ –1h15 = –1
15

^1– i h2 [(1 + i)2]3 = (1 + 2i + i2)3 = (2i)3


(Cevap A)
= 8i3
= 8(–1)
= –8i
Çözüm Yayınları

(Cevap A)

7. i2 = –1 olmak üzere,
1 + 2i + 3
2 1 − i 1 + i
3. i = –1 olmak üzere,
işleminin sonucu aşağıdakilerden hangisidir?
i + i2 + i3 + ... + i50
A) –1 B) i C) 1 D) 1 – i E) 1 + i
toplamının sonucu aşağıdakilerden hangisidir?
(1 + 2i) (1 + i) 3(1 – i) 1 + i + 2i + 2i2 + 3 – 3i
+ =
A) i – 1 B) –1 + i C) –1 – i 2 2 2
2
D) 1 + i E) 1 =
2
i + i2 + i3 + i4 + ... + i45 + i46 + i47 + i48 + i49 + i50
=1
= i – 1 – i + 1 + ... + i – 1 – i + 1 + i + i2 = –1 + i
(Cevap C)
0 0
(Cevap B)

85
8. x2 – 4x + 5 = 0
denkleminin karmaşık sayılarda çözüm kümesi
aşağıdakilerden hangisidir?

A) {2 – i, 2 + i} B) {1 – 2i, 1 + 2i}

C) {3 – i, 3 + i} D) {1 – i, 1 + i}
4. −3 ⋅ −5 ⋅ −15
E) {2 – 3i, 2 + 3i}
işleminin sonucu aşağıdakilerden hangisidir?
∆ = 16 – 4 . 1 . 5 ⇒ ∆ = –4
A) –15i B) –5i C) –3i D) 15i E) 15 ⇒ ñ∆ = ò–4 = 2i
4 – 2i
ñ3 i . ñ5 i . ò15 i = i3 . ò152 x1 = ⇒ x1 = 2 – i
2
= –15i 4 + 2i
x2 = ⇒ x2 = 2 + i
(Cevap A) 2
Ç = {2 – i, 2 + i}
(Cevap A)
Test 07 1. A 2. A 3. B 4. A 5. A 6. A 7. C 8. A 9. B 10. D 11. C 12. C 13. C 14. C 15. A 16. B

9. i2 = –1 olmak üzere, 13. i2 = –1 olmak üzere,


x2 + 5x + m – 1 = 0 f(x) = x3 – 3x2 + 3x – 1

denkleminin köklerinden biri (1 + i) olduğuna göre, m olduğuna göre, f(2 + i) ifadesi aşağıdakilerden hangisine
kaçtır? eşittir?

A) –4 + 3i B) –4 – 7i C) –5 + 4i A) 1 – i B) –1 + i C) –2 + 2i

D) 1 – 2i E) 1 – i D) –1 + 2i E) 2 + 2i
x1 + x2 = –5 ⇒ 1 + i + x2 = –5 f(x) = (x – 1)3 ⇒ f(2 + i) = (2 + i – 1)3
⇒ x2 = –6 – i ⇒ f(2 + i) = (1 + i)3
⇒ f(2 + i) = (1 + i)2(1 + i)
m – 1 = x1 . x2 ⇒ m – 1 = (1 + i) (–6 – i)

⇒ f(2 + i) = 2i(1 + i) = –2 + 2i
⇒ m – 1 = –5 – 7i
(Cevap C)
⇒ m = –4 – 7i
(Cevap B)

14. i2 = –1 olmak üzere,


(z – 2) · (4 + 2i) = 14 – 8i
2 + 2i
10. z = olduğuna göre, z karmaşık sayısı aşağıdakilerden
2 2
hangisidir?
olduğuna göre, z4 aşağıdakilerden hangisidir?
A) 3 – 4i B) 2 – 3i C) 4 – 3i
A) i B) 2 C) 1
D) 1 – 3i E) 2 + 3i
D) –1 E) 1 − 2i 14 – 8i (14 – 8i)(4 – 2i)
z –2= =
z = ñ2 (1 + i) ⇒ z4 = ( ñ2 )4(1 + i)4
4 + 2i 20
2 2 (4 – 2i) 56 – 28i – 32i + 16i2
=
1 20
Çözüm Yayınları

⇒ z4 = . (2i)2
4 40 – 60i
=
1 20
⇒ z4 = . 4i2 z – 2 = 2 – 3i
4
z = 4 – 3i (Cevap C)
⇒ z4 = –1
(Cevap D) 15. 12 = –1 olmak üzere,

z = 3 + 4i

olduğuna göre,
11. z 3 − 8i = 3 + 4i
2 2
z − 2iz − 4 z+z
 
olduğuna göre, z karmaşık sayısı aşağıdakilerden  z − z 
hangisidir?
ifadesinin sonucu aşağıdakilerden hangisidir?
A) 4 + 2i B) 5 + 3i C) 3 + 2i
9 3 3i 9i 9
A) − B) − C) D) E)
D) 3 + 6i E) 3 + 5i 16 4 4 16 16
z3 + 8i3 (z + 2i)(z2 – 2iz – 4) (3 + 4i + 3 – 4i)
]2 = ` 8i j = ` 4i j
6 2 3 2
= = 3 + 4i [
z2 – 2iz – 4 z2 – 2iz – 4 3 + 4i – (3 – 4i)
⇒ z + 2i = 3 + 4i 9 9
= =–
⇒ z = 3 + 2i 16i2 16
(Cevap A)
(Cevap C)
86
16. x3 + mx2 + 7x – 5 = 0
denkleminin köklerinden biri 1 dir.

Buna göre, bu denklemin karmaşık sayı olan diğer iki


12. Köklerinden biri 3 – 2i olan reel (gerçek) katsayılı ikinci kökünden biri aşağıdakilerden hangisidir?
dereceden denklem aşağıdakilerden hangisidir?
A) –1 + 1 B) 1 – 2i C) –1 + 2i
A) x2 – 4x + 9 = 0 B) x2 + 2x + 3 = 0
D) 1 – i E) 2 – i
C) x2 – 6x + 13 = 0 D) x2 + 6x + 13 = 0 x = 1 için 1 + m + 7 – 5 = 0 ⇒ m = –3
Denklem: x3 – 3x2 + 7x – 5 = 0
E) x2 – 5x + 6 = 0 x3 – 3x2 + 7x – 5 x – 1 x2 – 2x + 5 = 0
x1 = 3 – 2i ⇒ x2 = 3 + 2i x3 –x2 ∆ = 4 – 4 . 5 = –16
x2 – 2x + 5
x1 + x 2 = 6 –2x2 + 7x – 5 ñ∆ = 4i
x1 . x2 = 13 2 – 4i
–2x2 + 2x x1 = = 1 – 2i
x2 – (x1 + x2)x + x1 . x2 = 0 ⇒ x2 – 6x + 13 = 0 5x – 5 2
2 + 4i
5x – 5 x2 = = 1 + 2i
(Cevap C) 2 (Cevap B)
0
İkinci Dereceden Fonksiyonlar BÖLÜM 04 Test 08
1. y = x2 – 7x – 18 5. y = x2 + bx + c
parabolü A(a, 0), B(b, 0) ve C(0, c) noktalarından geçtiğine parabolünün tepe noktası (2, –1) olduğuna göre,
göre, a + b + c toplamı kaçtır? b + c toplamı kaçtır

A) –29 B) –11 C) 25 D) 27 E) 30 A) –2 B) –1 C) 3 D) 4 E) 7
x = 0 için y = –18 ⇒ c = –18 b
y = 0 için x2 – 7x = 0 ⇒ (x + 2) (x – 9) = 0 – = 2 ⇒ b = –4
2
⇒ x = –2 veya x = 9 x = 2 için y = –1 olmalıdır.
a = –2, b = 9
–1 = 4 – 4 . 2 + c ⇒ c = 3 bulunur.
a + b + c = –2 + 9 – 18 = –11 bulunur.
(Cevap B) b + c = –4 + 3 = –1 dir.
(Cevap B)
2. f(x) = x2 – (m + 1)x – 3
g(x) = –x2 + 2x + n
6. y
parabolleri x eksenini aynı noktalarda kestiklerine göre, y = x2
m + n toplamı kaçtır?
A
A) –6 B) –5 C) –4 D) 4 E) 6
1 –(m + 1) –3
= = B
–1 2 n D
–m – 1 1 x
= ⇒m+1=2

2 –1
⇒ m = 1 ve
E C
–3 1
= ⇒ n = 3 bulunur.
n –1
y = –x2 + 1
m + n = 1 + 3 = 4 tür.
(Cevap D)
Aşağıdaki noktalardan hangisi
3. y
y ≤ x2
y = cx2 2
y = bx
Çözüm Yayınları

y ≤ –x2 + 1

sistemini sağlayan bölgede bulunur?


x
0 A) A B) B C) C D) D E) E
Aranan bölge eğrilerin alt bölgelerinin kesisimi olur.
(Cevap C)
2
y = ax

Yukarıda grafikleri verilen y = ax2, y = bx2 ve y = cx2


parabolleri için aşağıdakilerden hangisi doğrudur?

A) a < b < c B) a < c < b C) b < a < c

D) b < c < a E) c < a < b


a < 0, b < c (kollar açıldıkça boş katsayı küçülür.)
7. f: R → R, f(x) = –2x2 + 8x + m – 3
Buna göre, a < b < c
(Cevap A) fonksiyonunun grafiği x eksenine teğet olduğuna göre, m
kaçtır?

A) –11 B) –5 C) 3 D) 5 E) 11
4. y ∆ = 0 ⇒ 64 –4 . (–2) . (l – 3) = 0

⇒ m = –5
87
6 (Cevap B)

x
–1 3

8. f(x) = x2 – 2x + m – 3
Şekilde grafiği verilen parabolün tepe noktasının x
fonksiyonunun alabileceği en küçük değer –3 olduğuna
eksenine uzaklığı kaç birimdir?
göre, m kaçtır?
13 15 17
A) B) 7 C) D) 8 E) A) –2 B) –1 C) 1 D) 2 E) 3
2 2 2
b –2
y = a(x +1)(x – 3), x = 0 için y = 6 dır. – =– , f(1) = –3 ⇒
2a 2.1
6 = a(1)(–3) ⇒ a = –2 bulunur.
f(1) = –3 ⇒ 1 – 2 + m – 3 = –3
Parabolün denklemi y = –2(x + 1)(x – 3) olur.
–1 + 3 ⇒m=1
Tepe noktasının apsisi = 1 ve ordinatı x = 1 için y = –2(2) (–2) = 8 dir.
2 (Cevap D) (Cevap C)
Test 08 1. B 2. D 3. A 4. D 5. B 6. C 7. B 8. C 9. E 10. E 11. C 12. B 13. C 14. B

9. y 12. y = x2 + 5x – 1 parabolü ile y = x – 3 doğrusunun kesim


noktaları A ve B dir.

Buna göre, [AB] doğru parçasının orta noktasının ordinatı


y = (x – 3)2
kaçtır?

A) –6 B) –5 C) –1 D) 3 E) 4
C B
x2 – 5x – 1 = x – 3 ⇒ x2 + 4x + 2 = 0
x1 + x 2 –4
x [AB] doğru parçasının orta noktasının apsisi = = –2 dir.
O A 2 2
y = x – 3 doğrusunda x yerine –2 yazılarak ordinatı –5 bulunur.

Yukarıdaki şekilde y = (x – 3)2 fonksiyonunun grafiği (Cevap B)

verilmiştir.

Buna göre, OABC dikdörtgeninin alanı kaç birimkaredir?

A) 24 B) 36 C) 40 D) 48 E) 54
Tepe noktasının apsisi 3 tür. |OA| = 6 birim olur. x = 0 için y = 9 dur. |OC| = 9 birimdir.

A(OABC) = 6 . 9 = 54 birimkare bulunur.


13. y
(Cevap E)

y = x2 – 6x + m

x
O A B

10.
Çözüm Yayınları

y Şekilde grafiği verilen parabolün denklemi

T y = x2 – 6x + m

ve parabolün x ekseninin kesim noktaları A ile B dir.


C B
|OB| = 5|OA| olduğuna göre, m kaçtır?

A) 3 B) 4 C) 5 D) 6 E) 7
O A A noktasının apsisi x1, B noktasının apsisi x2 olsun. |OB| = 5 . |OA| ⇒ x2 = 5x1
x1 + x2 = 6 ⇒ x1 + 5x1 = 6
⇒ x1 = 1 ve x2 = 5 bulunur.
Şekildeki y = –2x2 + 8x + 10 parabolünün tepe noktası T dir. m = x1 . x2 = 1 . 5 = 5 bulunur.

Buna göre, OABC dikdörtgeninin alanı kaç birimkaredir? (Cevap C)

A) 12 B) 15 C) 16 D) 18 E) 20
b –8
– = ⇒ |OA| = 2 birim ve x = 0 için y = 10 birimdir.
2a 2 . (–2)
A(OABC) = 2 . 10 = 20 birimkare
(Cevap E) 14. y

88 D C(k, 7)

x
A B

y = a(x – 4)2 + 16
ABCD dikdörtgeninin C(k, 7) ve D köşeleri y = a(x – 4)2 + 16
11. y = x + k doğrusu, y = x2 – x – 2 parabolüne teğet
parabolü üzerindedir.
olduğuna göre, bu doğrunun y eksenini kestiği noktanın
ordinatı kaçtır? Buna göre, A(ABCD) kaç birimkaredir?

A) –5 B) –4 C) –3 D) 1 E) 2 A) 36 B) 42 C) 45 D) 48 E) 56
x2 – x – 2 = x + k ⇒ x2 – 2x – 2 – k = 0 Orijinin koordinatları denklemi sağlar.
∆ = 0 olmalıdır. x = 0 için a(x – 4)2 + 16 = 0 ⇒ a(–4)2 = –16
⇒ a = –1 bulunur.
4 – 4 . 1 . (–2 – k) = 0 ⇒ k = –3 bulunur. –(a – 4)2 = 16 = 7 ⇒ x1 = 1 ve x2 = 7
y = x – 3 doğrusu, x = 0 için y eksenini kestiği noktanın ordinatı –3 tür. |AB| = x2 – x1 = 6 dır.
A(ABCD) = 6 . 7 = 42 birimkaredir. (Cevap B)
(Cevap C)
İkinci Dereceden Fonksiyonlar BÖLÜM 04 Test 09
1. y 5. y

x
A B
x
C –1 O 5

y = –x2 + px + q

Şekilde verilen y = –x2 + (m + 1)x – m parabolü x eksenini A


ve B noktalarında, y eksenini C noktasında kesmektedir. Şekildeki parabolün denklemi y = –x2 + px + q olduğuna
göre, f(1) kaçtır?
|AB| = 4 olduğuna göre, C noktasının ordinatı kaçtır?
A) 6 B) 7 C) 8 D) 9 E) 10
A) –6 B) –5 C) –4 D) –3 E) –2 p = x1 + x2 = –1 + 5 = 4 ve
|x2 – x1| = 4 ⇒ ñ∆ = 4 q = –x1 . x2 = –(–1) . 5 = 5 olur.
|a| f(x) = –x2 + 4x + 5 ⇒ f(1) = 8
^m + 1h - 4. (–1) (–m)
2
(Cevap C)
⇒ 1 =4
⇒ |m – 1| = 4
⇒ m – 1 = 4 veya m – 1 = –4
⇒ m – 5 veya m = –3

m = 5 için y = –x2 + 6x – 5 parabolu y eksenini y = –5 noktasında keser.


(Cevap B)

2. y = x2 – 4x + c
parabolü x eksenine teğet olduğuna göre, c kaçtır?

A) –4 B) –2 C) 3 D) 4 E) 8 6. y = x2 – 4x – 5
∆ = 0 ⇒ 16 – 4 . 1 . c = 0
Çözüm Yayınları

⇒c=4
parabolüne teğet ve y = 2x – 3 doğrusuna paralel olan
(Cevap D)
doğrunun denklemi aşağıdakilerden hangisidir?

A) y = 2x – 14 B) y = 2x – 9 C) y = 2x – 5

D) y = 2x – 4 E) y = 2x + 3
3. y
y = 2x – 3 doğrusuna paralel olan doğrusunun denklemi y = 2x + k biçimindedir.
y = ƒ(x) x2 – 4x – 5 = 2x + k ⇒ x2 – 6x – 5 – k = 0
∆ = 0 ⇒ 36 – 4(–5 – k) = 0
⇒ k = –14 bulunur.
(Cevap A)
x
–1 O 5

–10

7. y
y = f(x) fonksiyonunun grafiği şekildeki parabol eğrisidir.

Buna göre f(x) in alabileceği en küçük değer kaçtır?

A) –20 B) –18 C) –16 D) –15 E) –14


y = a(x + 1)(x – 5)
x = 0 için –10 = a(1)(–5) ⇒ a = 2 bulunur. x
89
Parabolun denklemi y = 2(x + 1)(x – 5) olur.
–1 + 5
Tepe noktasının apsisi = 2 dir. y = ƒ(x)
2
x = 2 için y = 2(3)(–3) = –18 (En küçük)
(Cevap B) Tepe noktası birinci bölgede bulunan şekildeki

f(x) = ax2 + bx + c
4. y = x2 + bx + 15
parabolü için aşağıdakilerden hangisi yanlıştır?
parabolünün simetri ekseni x = 4 doğrusu olduğuna göre,
tepe noktasının ordinatı kaçtır? A) a < 0 B) b2 – 4ac > 0 C) a . b < 0

A) –3 B) –2 C) –1 D) 4 E) 5 D) a · c > 0 E) b · c > 0
6 6
– = 4 ⇒ b = –8 a < 0, – > 0 ⇒ b > 0, c > 0
2 2a
2
y = x – 8x + 15 parabolünün tepe noktasının apsisi 4 tür. Parabol x eksenini farklı iki noktada kestiğinden ∆ > 0 ⇒ b2 –4ac > 0 dır.
x = 4 için y = 16 – 32 + 15 ⇒ y = –1 bulunur. a ve c ters işaretli olduklarından a . c > 0 yanlıştır.
(Cevap C) (Cevap D)
Test 09 1. B 2. D 3. B 4. C 5. C 6. A 7. D 8. E 9. B 10. B 11. E 12. B 13. B 14. C

8. y 12. y = –2x2 + mx + 4 parabolü ile y = x – 3 doğrusu A ve B


noktalarında kesişiyor.
6 [AB] doğru parçasının orta noktasının apsisi –1 olduğuna
göre, m kaçtır?

3 5
A) –5 B) –3 C) D) E) 1
x 2 2
–1 0 3 –2x2 + mx + 4 = x – 3 ⇒ 2x2 + (1 – m)x – 7 = 0
x1 + x 2
y = ƒ(x) = 1 ⇒ x + x2 = 2
2 1–m
Yukarıdaki şekilde y = f(x) parabolünün grafiği verilmiştir. ⇒
2
=2

⇒ m = –3 bulunur.
Buna göre, f(4) kaçtır?
(Cevap B)
A) –1 B) –3 C) –4 D) –6 E) –10
f(x) = a(x + 1) (x – 3)
f(0) = 6 ⇒ 6 = a(a) (–3)
⇒ a = –2
f(4) = –2 –2(5)(1) ⇒ f(4) = –10
(Cevap E) 13. y

9. f(x) = x2 – (m – 4)x – 4m y = (ƒog)(x)

parabolünün tepe noktası y ekseni üzerinde olduğuna


göre, tepe noktasının ordinatı kaçtır? x
–1 0 3
A) –20 B) –16 C) –14 D) 12 E) 16
m–4=0⇒m=4
f(x) = x2 – 16 bulunur. f(0) = –16 dır. –3
(Cevap B)
Çözüm Yayınları

10. y y = (fog)(x) fonksiyonunun grafiği yukarıdaki parabol eğrisidir.

f(x) = 2x + 1 olduğuna göre, g(4) kaçtır?


g(x) =x2 + bx + c
A) 1 B) 2 C) 3 D) 5 E) 8
O
x (fog)(x) = a(x + 1)(x – 3) ⇒ –3 = a(1)(–3)
⇒a=1
–4 (fog)(x) = (x + 1)(x – 3) ⇒ f(g(x)) = (x + 1)(x – 3)
x2 – 2x –4
2g(x) + 1 = x2 – 2x – 3 ⇒ g(x) =
ƒ(x) = –(x – 1)2 2
⇒ g(4) = 2
Yukarıda grafiği verilen f(x) ve g(x) parabolleri birbirini tepe (Cevap B)
noktasında kesmektedir.

Buna göre, g(0) değeri kaçtır?

7 5 3
A) − B) –3 C) − D) –2 E) − 14. y
2 2 2
f(x) = –(x – 1)2 parabolünün tepe noktası (1, 0) dır.
g(1) = 0 ⇒ 1 + b + c = 0 E T
⇒ b + c = –1 bulunur. C
A
g(x) parabolünün tepe noktasınınkoordinatları f(x) parabolünü sağlar.
–(x – 1)2 = 4 ⇒ x = –1 veya x = 3
90 x
g(x) parabolünün tepe noktası (–1, –4) tür. O B D
x = –1 için 1 – b + c = –4 ⇒ –b + c = –5
b + c = –1
–b + c = –5
⇒ b = 2 ve c = –3, g(0) = 3 bulunur.
(Cevap B) y = –x2 + 6x – 5
y = –x2 + 6x – 5
11. Denklemi
parabolünün tepe noktası T ve B(2, 0) olmak üzere, AOBC ve
y = x2 + ax + a – 1 EODT birer dörtgendir.
olan parabolün simetri ekseni x = 2 doğrusu olduğuna
Buna göre, taralı bölgenin alanı kaç birimkaredir?
göre, x eksenini kestiği noktalar arasındaki uzaklık kaç
birimdir? A) 3 B) 4 C) 6 D) 8 E) 12
A) 2 B) 3 C) 4 D) 5 E) 6 b –6
– = = 3,
6 2a 2 . (–1)
– = 2 ⇒ a = –4 x = 3 için y = 4 bulunur.
2
y = x2 – 4x – 5 = 0 ⇒ (x + 1)(x – 5) = 0 x = 2 için y = 3 olur.
⇒ x1 = –1, x2 = 5
A(ODTE) – A(OBCA) = 3 . 4 – 2 . 3 = 6
x2 – x2 = 5 – (–1) = 6 (Cevap C)
(Cevap E)
İkinci Dereceden Fonksiyonlar BÖLÜM 04 Test 10
1. y 5. y

y = ƒ(x)

x x
–1 O 4 0 3

A(4, –4)

y = f(x) fonksiyonunun grafiği şekildeki parabol eğrisidir.


Şekildeki taralı bölgeyi aşağıdaki eşitsizlik sistemlerinden
Parabolün x eksenini kestiği noktaların apsisleri –1 ve 4, hangisi gösterir?
f(1) = 12 olduğuna göre, f(5) kaçtır?
A) y ≥ – x B) y ≤ – x C) y ≥ – x
A) –12 B) –8 C) –6 D) –5 E) –3
f(x) = a(x + 1)(x – 4) y ≤ x2 – 2x y ≥ x2 – 3x y ≤ – x2 + 4x
f(1) = 12 ⇒ a(2)(–3) = 12
⇒ a = –2 D) y ≤ – x E) y ≥ – x
f(x) = –2(x + 1)(x – 4) ⇒ f(5) = –2(6)(1) 2
y ≥ – x + 3x y ≤ – x2 + 3x
⇒ f(5) = –12 Orijinden ve A(4, –4) noktasından geçen doğrunun denklemi y = –x tir. Parabolün
(Cevap A) denklemi y = ax(x – 3) biçiminde ve x = 4 için y = –4 tür.
a . 4 . (4 –3) = –4 ⇒ a = –1 bulunur.
2. f: R → R, f(x) = x2 – 2x – 15 Taralı bölge doğrunun üstü ve parabolün iç bölgesidir.

fonksiyonunun görüntü kümesi aşağıdaki aralıklardan (Cevap E)

hangisidir?

A) (–∞, –2] B) (–∞, 15] C) [–9, ∞)

D) [–16, ∞) E) [15, ∞)
Çözüm Yayınları

Tepe noktasının ordinatı


4ac – b2 4 . 1 . (–15) – (4)
– = –16 6. f(x) = ax2 + 2x – 3
4a 4.1
ve parabolün kolları yukarı doğru olduğndan görüntü kümesi [–16, ∞) olur. fonksiyonunun alabileceği en büyük değer 4 olduğuna
(Cevap D) göre, f(–7) kaçtır?

A) –30 B) –27 C) –24 D) –20 E) 12


3. n nin hangi değeri için y = x + n doğrusu,
Parabolün kolları aşağı doğru olduğunda (a < 0) alabileceği en büyük değer tepe
y = x2 – 3x + 1 noktasının ordinatıdır.
4ac – b2 4a(–3) – 4
parabolüne teğet olur? 4a
= 4 ⇒
4a
=4
1
A) –4 B) –3 C) –2 D) 1 E) 3 ⇒a=– bulunur.
x2 – 3x + 1 = x + n ⇒ x2 – 4x + 1 – n = 0 7
1 2
f(–7) = – . (–7) + 2(–7) –3 = –24
∆ = 0 ⇒ 16 – 4 . 1 . (1 – n) = 0 7
(Cevap C)
⇒ n = –3 bulunur.
(Cevap B)

4. y 7. y
y = ƒ(x)

k
B
91
x x
O 2 A –1

Şekilde grafiği verilen parabolün denklemi Şekildeki grafik f(x) = x2 + 3x + c fonksiyonuna aittir.
y = x2 – 3x + m2 + 4m – 2
Buna göre, |AB| uzunluğu kaç birimdir?
olduğuna göre, k kaçtır?
A) 2 2 B) 3 C) 3 2 D) 4 E) 4 2
1 3 5
A) B) 1 C) D) 2 E) f(–1) = 0 ⇒ 1 – 3 + c = 0
2 2 2
⇒c=2
x = 0 için y = m2 + 4m – 2 = k dır.
x2 + 3x + 2 = 0 ⇒ (x + 1)(x + 2) = 0
Parabolün denklemi y = x2 – 3x + k olur.
⇒ x = –1 veya x = –2
x = 2 için y = 0 dır. 4 – 6 + k = 0 ⇒ k = 2 bulunur.
A(–2, 0) ve B(0, 2) olmak üzere,
(Cevap D)
|AB| = ^ –2h2 + ^ –2h2 = ñ8 = 2ñ2 birim
(Cevap A)
Test 10 1. A 2. D 3. B 4. D 5. E 6. C 7. A 8. B 9. D 10. C 11. B 12. D 13. B 14. C

8. x liraya alınan bir mal y liraya satılmaktadır. 11. f(x) = x2 + (a + 3)x + 3b – 1


x ile y arasında, parabolü, x = –4 apsisli noktada y = 4 doğrusuna teğet
olduğuna göre, b kaçtır?
y = –x2 + 9x + 24
A) 9 B) 7 C) 5 D) –3 E) –5
bağıntısı bulunduğuna göre, bu alışverişten sağlanan kâr
Parabolün tepe noktası T(–4, 4) olur.
en çok kaç lira olabilir? b –(a + 3)
– = = –4 ⇒ a = 5
2a 2.1
A) 32 B) 40 C) 45 D) 50 E) 64 x = –4 için y = 4 olduğundan
Kâr = y – x = –x2 + 9x + 24 – x
f(x) = x2 + 8x + 3b – 1 ⇒ f(–4) = 4
= –x2 + 8x + 24
⇒ 16 – 32 + 3b – 1 = 4
Parabolün kolları aşağı doğru olduğundan alabileceği en büyük değer tepe noktasının
⇒ b = 7 bulunur.
ordinatıdır. (Cevap B)
4ac – b2 4 . (–1) . 24 – 64 12. y = –x2 + 2x parabolünün x + y – 6 = 0 doğrusuna paralel
= = 40
4a 4 . (–1)
(Cevap B)
teğetinin değme noktası A(a, b) olduğuna göre, a + b
toplamı kaçtır?

3 7 9
A) B) C) 2 D) E) 3
2 4 4
Parabolün y = –x + 6 doğrusuna paralel teğeti y = –x + k şeklindedir.
–x2 + 2x = –x + k ⇒ x2 – 3x + k = 0
9. y ∆ = 0 ⇒ 9 – 4 . 1 . k = 0
y = x2 – 4x – 5 9
⇒ k = bulunur.
4
(Cevap D)

O 13. y
x
B
y = x2 + k
A
Çözüm Yayınları

A
Yukarıdaki şekilde verilenlere göre, A(OAB) kaç x
O
birimkaredir?

23 25 y = –x2 + 4x + 5
A) 11 B) C) 12 D) E) 13
2 2
y = x2 + k parabolü, y = –x2 + 4x + 5 parabolüne A
y = 0 için x2 – 4x – 5 = 0 ⇒ (x + 1)(x – 5) = 0
noktasında teğet olduğuna göre, A noktasının
⇒ x = –1 veya x = 5
koordinatları toplamı kaçtır?
Buna göre, |OB| = 5 birimdir.
x = 0 için y = –5 ve |OA| = 5 birimdir. A) 8 B) 9 C) 10 D) 11 E) 12
5 . 5 25 x2 + k = –x2 + 4x + 5 ⇒ 2x2 – 4x + k – 5 = 0
A(OAB) = = birimkare bulunur.
2 2 ∆ = 0 ⇒ 16 – 4 . 2 . (k – 5) = 0
(Cevap D) ⇒ k = 7 bulunur.
2x2 – 4x + 2 = 0 ⇒ x2 – 2x + 1 = 0
10. y ⇒ (x – 1)2 = 0
⇒x=1
x = 1 için y = 1 + 4 + 5 = 8 bulunur. A noktasının koordinatları toplamı 1 + 8 = 9 bulunur.
(Cevap B)

x 14.
A O C y
B y = x2
92
D C B y=3
d
D A
Şekilde y = x2 + x – 6 parabolü ile d doğrusunun grafiği
verilmiştir. x
O
|OB| = |BD| olduğuna göre, AOD üçgeninin alanı kaç
x=1
birimkaredir?

A) 12 B) 13 C) 18 D) 20 E) 26 Şekildeki ABCD dikdörtgeninin alanı kaç birimkaredir?


2
x + x – 6 = 0 ⇒ (x + 3)(x – 2) = 0 A) 1 B) 2 C) 4 D) 6 E) 8
⇒ x = –3 veya x = 2 bulunur. x = 1 için y = x2 = 1 bulunur.
|OA| = |–3| = 3 birimdir. |AB| = 3 – 1 = 2 birim ve
x = 0 için y = –6 olduğundan |OB| = |–6| = 6 birimdir. |OD| = 2 . 6 = 12 birim olur. |AD| = 1 + 1 = 2 biridir.
3 . 12
A(AO D) = = 18 birimkaredir. A(ABCD) = 4 birimkare bulunur.
2
(Cevap C) (Cevap C)
İkinci Dereceye Dönüştürülebilen Denklemler BÖLÜM 04 Test 11
1. x4 – 6x2 + 8 = 0 5. x2 – |x + 12| = 0
denkleminin çözüm kümesi aşağıdakilerden hangisidir? denkleminin çözüm kümesi aşağıdakilerden hangisidir?

A) {2, 4} B) {–2, 4} C) {–4, 2} A) {–4, 3} B) {–3, 4} C) {–3, 2}

D) {–2, − 2, 2, 2 } E) {–2, –1, 1, 2} D) {–1, 4} E) {3, 4}


x + 12 < 0 ⇒ x < –12 iken:
x2 = t olsun. t2 – 6t + 8 = 0 ⇒ (t – 2)(t – 4)
x2 + (x + 12) = 0 ⇒ ∆ < 0 (greçek kök yoktur.)
⇒ t = 2 veya t = 4
x + 12 ≥ 0 ⇒ x ≥ –12 iken:
x2 = 2 ⇒ x = –ñ2 veya x = ñ2
x2 – x – 12 = 0 ⇒ (x + 3)(x – 4) = 0
x2 = 4 ⇒ x = –2 veya x = 2
⇒ x = –3 veya x = 4 bulunur.
Ç = {–2, –ñ2, ñ2, 2}
Ç = {–3, 4}
(Cevap D)
(Cevap B)

6. 3 + 4x + 9 = x
2. (x2 – x – 3)2 – 2(x2 – x – 3) – 3 = 0
denkleminin çözüm kümesi aşağıdakilerden hangisidir?
denkleminin köklerinin çarpımı kaçtır?
A) {4} B) {7} C) {10}
A) –18 B) –12 C) 12 D) 15 E) 18
D) {0, 10} E) {4, 10}
x2 – x – 3 = t olsun. t2 – 2t – 3 = 0 ⇒ (t + 1)(t – 3) = 0
⇒ t = –1 veya t = 3 ó4x+9 = x – 3 ⇒ 4x + 9 = x2 – 6x + 9

x2 – x – 3 = –1 ⇒ x2 – x – 6 = 0 ⇒ x2 – 10x = 0

⇒ x1 . x2 = –2 ve ⇒ x(x – 10) = 0

x2 – x – 3 = 3 ⇒ x2 – x – 6 = 0
⇒ x = 0 veya x = 10
Çözüm Yayınları

⇒ x3 . x4 = –6 bulunur.
x = 0 denklemi sağlamaz. Ç = {10}

x1 . x2 . x3 . x4 = (–2) . (–6) = 12 dir. (Cevap C)

(Cevap C)

3. (x2 – 1)3 = (x2 – 1)


7. x2 – 2|x| – 48 = 0
denkleminin köklerinin toplamı kaçtır?
denkleminin çözüm kümesi aşağıdakilerden hangisidir?
A) –3 B) –2 C) –1 D) 0 E) 1
A) {–8, 8} B) {–6, 6} C) {–8, 6}

x2 – 1 = t olsun t3 = t ⇒ t3 – t = 0 D) {–8, 6, 8} E) {–8, –6, 8}


⇒ t(t – 1)(t + 1) = 0 |x| = t olsun. t2 –2t – 48 = 0 ⇒ (t + 6)(t – 8) = 0
⇒ t = 0, t = 1, t = –1 ⇒ t = –6 veya t = 8
t = 0 ⇒ x2 – 1 = 0 |x| = –6 olamaz.
⇒ x = –1 veya x = 1 |x| = 8 ⇒ x = –8 veya x = 8
t = 1 ⇒ x2 – 1 = 1 (Cevap A)
⇒ x = –ñ2 ve x = ñ2
t = –1 ⇒ x2 – 1 = –1
⇒x=0
Köklerin toplamı –1 + 1 – ñ2 + ñ2 + 0 = 0 dır.
93
(Cevap D)

3
8. x+ x−2 = 2
4. (x2 – 3x)2 – 14(x2 – 3x) + 40 = 0
denkleminin çözüm kümesi aşağıdakilerden hangisidir?
denkleminin köklerinin toplamı kaçtır?
A) {6} B) {11} C) {6, 11}
A) 3 B) 4 C) 5 D) 6 E) 9
D) {–6} E) {0, 6}
x2 – 3x = t olsun. t2 – 14t + 40 = 0 ⇒ (t – 4)(t – 10) = 0
⇒ t = 4 veya t = 10 x + óx – 2 = 23 ⇒ óx – 2 = 8 – x
x2 – 3x = 4 ⇒ x2 – 3x – 4 = 0 ⇒ x – 2 = 64 – 16x + x2
⇒ x1 + x2 = 3 ve ⇒ x2 – 17x + 66 = 0
x2 – 3x = 10 ⇒ x2 – 3x – 10 = 0 ⇒ (x – 6)(x – 11) = 0
⇒ x3 + x4 = 3 bulunur. ⇒ x = 6 veya x = 11
x1 + x2 + x3 + x4 = 3 + 3 = 6 dır. x = 11 denklemi sağlamaz.
(Cevap A)
(Cevap D)
Test 11 1. D 2. C 3. D 4. D 5. B 6. C 7. A 8. A 9. B 10. A 11. B 12. C 13. B 14. B 15. D 16. D

9. 3
( x + 2)2 − 23 x + 2 − 3 = 0 13. 6x + 7 = x + 2

denkleminin çözüm kümesi aşağıdakilerden hangisidir? denkleminin çözüm kümesi aşağıdakilerden hangisidir?

A) {–1, 27} B) {–3, 25} C) {–1, 3} A) {–3, 1} B) {–1, 3} C) {–3}

D) {–1, 9} E) {–3, 9} D) {1} E) {–3, 1, 1, 3}


6x + 7 = x2 + 4x + 4 ⇒ x2 – 2x – 3 = 0
3
x + 2 = t = t olsun. t2 – 2t – 3 = 0 ⇒ (t + 1)(t – 3) = 0
⇒ (x + 1)(x – 3) = 0
⇒ t = –1 veya t = 3
⇒ x = –1 veya x = 3
3
x + 2 = –1 ⇒ x + 2 = –1
Her iki kök denklemi sağlar.
⇒ x = –3 ve
(Cevap B)
3
x + 2 = 3 ⇒ x + 2 = 27
⇒ x = 25 bulunur.
(Cevap B)

14. a bir sabit sayı olmak üzere,


10. 2x − 5 + x = 4 (2a − 1)2 6a − 3
2
− +2 =0
x x
denkleminin çözüm kümesi aşağıdakilerden hangisidir?

A) {3} B) {5} C) {7} denklemini sağlayan x değerlerinin toplamı 3 olduğuna


göre, a kaçtır?
D) {3, 5} E) {3, 7}
3 5
A) 1 B) C) 2 D) E) 3
ó2x–5 = 4 – x ⇒ 2x – 5 = 16 – 8x + x2 2 2
⇒ x2 – 10x + 21 = 0 (2a – 1) – 3(2a – 1)x + 2x2 = 0
⇒ (x – 3)(x – 7) = 0 3(2a – 1)
x1 + x2 = 3 ⇒ =3
⇒ x = 3 veya x = 7 2
Çözüm Yayınları

⇒ 3(2a – 1) = 6
x = 7 denklemi sağlamaz.
(Cevap A) ⇒ 2a – 1 = 2
3
⇒ a = bulunur.
2
(Cevap B)

11. x2 + 9 + 15 =8
x2 + 9
15. |x – 1| · |x + 3| = 3
denkleminin çözüm kümesi aşağıdakilerden hangisidir?
denkleminin kaç farklı reel (gerçek) kökü vardır?
A) {–3, 0, 3} B) {–4, 0, 4} C) {–5, 0, 5}
A) 0 B) 1 C) 2 D) 3 E) 4
|(x – 1)(x – 3)| = 3 ⇒ x2 + 2x – 3 = –3 veya x2 + 2x – 3 = 3
D) {–4, 5} E) {–3, 4} ⇒ x2 + 2x = 0 veya x2 + 2x – 6 = 0
15 Denklemlerinin çözümünden 4 kök bulunur. Bulunan köklerin hepsi denklemi sağlar.
x 2 + 9 = t olsun. t + = 8 ⇒ t2 – 8t + 15 = 0
t
(Cevap E)
⇒ (t –3)(t – 5) = 0

⇒ t = 3 veya t = 5

x 2 + 9 = 3 ⇒ x2 + 9 = 9

94 ⇒ x = 0 ve

x 2 + 9 = 5 ⇒ x2 + 9 = 25
⇒ x = –4 veya x = 4 bulunur. 2
(Cevap B)  x  + 2x = 15
16.  
 x−3 x−3

denkleminin çözüm kümesi aşağıdakilerden hangisidir?


12. 4x − 3 = 8x + 1 − 2
denkleminin çözüm kümesi aşağıdakilerden hangisidir? A) {–5, 3} B) {32 , 72} C) {–3, 5}

A) {1} B) {3} C) {1, 3}

D) {–1, 3} E) {–3, 1}
D) {52 , 92} E) { }
5, 5
3
x = 1 ve x = 3 denklemi sağlar. x
= t olsun. t2 + 2t – 15 = 0 ⇒ (t + 5)(t – 3) = 0
Ç = {1, 3} x–3
(Cevap C) ⇒ t = –5 veya t = 3
x 5 x 9
= –5 ⇒ x = ve = 3 ⇒ x = bulunur.
x–3 2 x–3 2
(Cevap D)
İkinci Dereceye Dönüştürülebilen Denklemler BÖLÜM 04 Test 12
1. x 4 − 5x 2 + 4 = 0 5. 1 + 5 − 36 = 0
x+2 ( x − 2) 2 x − 2
denkleminin çözüm kümesi aşağıdakilerden hangisidir? denkleminin çözüm kümesi aşağıdakilerden hangisidir?
A) {1, 4}


B) {1, 2}

C) {–2, –1, 1, 2} D) {–2, –1, 1}


A) −9,
1
4
{ } {
1 1
B) − ,
9 4
1
}
C) − , 4
9 { }
E) {–1, 1, 2}
x2 = t olsun. t2 – 5t + 4 = 0 ⇒ (t – 1)(t – 4) = 0
D) { }
1, 1
9 4
E) {
17 , 9
9 4 }
⇒ t = 1 veya t = 4 1
= t olsun. t2 + 5t – 36 = 0 ⇒ (t + 9)(t – 4) = 0
x–2
x2 = 1 ⇒ x = –1 veya x = 1 ve
⇒ t = –9 veya t = 4
x2 = 4 ⇒ x = –2 veya x = 2 bulunur. 1 17
x = –2 paydayı sıfır (ifadeyi tanımsız) yaptığından alınmaz. = –9 x = ve
x–2 9
Ç = {–1, 2, 2} 1 9
(Cevap E) =4⇒x= bulunur.
x–2 4
(Cevap E)

6. x4 – 40x2 + 144 = 0
2
2. (x – 2) – 5(x – 2) + 6 = 0
denkleminin çözüm kümesi aşağıdakilerden hangisidir?
denkleminin kökler çarpımı kaçtır?
A) {–4, 4, 6} B) {–6, –4, 4, 6}
A) 6 B) 10 C) 15 D) 18 E) 20
C) {–6, –2, 2, 6} D) {–3, –2, 2, 3}
x – 2 = t olsun. t2 – 5t + 6 = 0 ⇒ (t – 2)(t – 3)
⇒ t = 2 veya t = 3 E) {–2, 2, 9}
x – 2 = 2 ⇒ x = 4 ve x2 = t olsun. t2 – 40t + 144t = 0 ⇒ (t – 4)(t – 36) = 0
Çözüm Yayınları

x – 2 = 3 ⇒ x = 5 bulunur. Çarpımları 20 dir. ⇒ t = 4 veya t = 36


(Cevap E) x2 = 4 ⇒ –2 veya x = 2
x2 = 36 ⇒ x = –6 veya x = 6
(Cevap C)

3. x4 – 4x2 + 3 = 0 2
7.  x  − 3 x  + 2 = 0
   
denkleminin çözüm kümesi aşağıdakilerden hangisidir?  x − 1  x − 1

A) {1, 3} B) {1, 4} C) {3, 4} denkleminin çözüm kümesi aşağıdakilerden hangisidir?

A) {2} B) {3} C) {4}


D) {− 3, 3 } E) {− 3, − 1, 1, 3 }
2 2
x = t olsun. t – 4t + 3 = 0 ⇒ (t – 1)(t – 3) = 0 D) {2, 4} E) {3, 4}
⇒ t = 1 veya t = 3 1
= t olsun. t2 – 3t + 2 = 0 ⇒ (t – 1)(t – 2) = 0
x2 = 1 ⇒ x = –1 veya x = 1, x–1
⇒ t = 1 veya t = 2
x2 = x = –ñ3 veya x = ñ3 bulunur.
1
(Cevap E) = 1 ⇒ x – 1 = x
x–1
⇒ – 1 = 0 olamaz. 95
x
= 2 ⇒ x = 2
x–1
(Cevap A)

8. |x2 – 8x + 7| – |8x – 8| = 0
4. (x2 – 7x + 10)2 + 2(x2 – 7x + 10) = 0 denkleminin köklerinin toplamı kaçtır?

denkleminin kökler çarpımı kaçtır? A) 9 B) 10 C) 12 D) 15 E) 18


|x – 1| . |x – 7| – 8|x – 1| = 0 ⇒ |x – 1| . (|x – 7| – 8) = 0
A) 63 B) 81 C) 100 D) 120 E) 135
⇒ |x – 1| = 0 veya |x – 7| – 8 = 0
(x2 – 7x + 10)(x2 – 7x + 10 + 2) = 0
|x – 1| = 0 ⇒ x = 1 ve
(x – 2)(x – 5)(x – 3)(x – 4) = 0
|x – 7| = 8 ⇒ x – 7 = –8 veya x – 7 = 8
x = 2, x = 5, x = 3, x = 4 bulunur.
⇒ x = –1 veya x = 15 bulunur.
Çarpımları 2 . 5 . 3 . 4 = 120
Toplamları 1 + (–1) + 15 = 15 tir. (Cevap D)
(Cevap D)
Test 12 1. E 2. E 3. E 4. D 5. E 6. C 7. A 8. D 9. A 10. B 11. C 12. C 13. B 14. A 15. A 16. C

9. 5x − 1 = x + 1 13. 5x + 6 + 3x − 2 = 6

denkleminin çözüm kümesi aşağıdakilerden hangisidir? denkleminin çözüm kümesi aşağıdakilerden hangisidir?

{41}
A) {1} B) C) {32}


A) {1}

D) {1, 2}
B) {2}

E) {2, 3}
C) {3}

D) {–1} E) {− }
1 Bu soruda cevaptan gitme daha kolay olur. x = 2 denklemi sağlar.
2 (Cevap B)
(ó5x–1)2 = (ñx + 1)2 ⇒ 5x – 1 = x + 2ñx + 1
⇒ 4x – 2 = 2ñx
⇒ 2x – 1 = ñx
⇒ 4x2 – 4x + 1 = x
⇒ 4x2 – 5x + 1 = 0
⇒ (4x – 1)(x – 1) = 0

1
⇒ x = veya x = 1
4
1
x = denklemi sağlamaz.
4
(Cevap A)
14. (x – 2)2 – |x – 2| – 6 = 0
10. x2 = 2 · |x| + 3
denkleminin çözüm kümesi aşağıdakilerden hangisidir?
denkleminin köklerinin çarpımı kaçtır?
A) {–1, 5} B) {–2, 3} C) {–1, 6}
A) –10 B) –9 C) –4 D) 3 E) 6
D) {2, 6} E) {3, 6}
|x| = t olsun. t2 = 2t + 3 ⇒ t2 – 2t – 3 = 0
|x – 2| = t olsun. t2 – t – 6 = 0 ⇒ (t + 2)(t – 3) = 0
⇒ (t + 1)(t – 3) = 0
⇒ = t = –2 veya t = 3
⇒ t = –1 veya t = 3
|x – 2| = –2 olamaz.
|x| = –1 olamaz.
|x – 2| = 3 ⇒ x – 2 = –3 veya x – 2 = 3
|x| = 3 ⇒ x = –3 veya x = 3 bulunur. Çarpımları –9 dur.
⇒ x = –1 veya x = 5
(Cevap B)
Çözüm Yayınları

(Cevap A)

15. 2 · 4x – 17 · 2x + 8 = 0
denkleminin çözüm kümesi aşağıdakilerden hangisidir?
11. 2x 2 − 2x + 1 = 2x − 3
A) {–1, 3} B) {–3, 1} C) {–2, 1}
denkleminin çözüm kümesi aşağıdakilerden hangisidir?
D) {–1, 4} E) {1, 3}
A) {–4} B) {1} C) {4}
2x = t olsun. 2t2 – 17t + 8 = 0 ⇒ (2t – 1)(t –8) = 0
1
D) {–1, 4} E) {1, 4} ⇒ t = veya t = 8
2
2x2 – 2x + 1 = (2x – 3)2 ⇒ 2x2 – 2x + 1 = 4x2 – 12x + 9 1
2x2 = ⇒ 2x = 2–1
⇒ 2x2 – 10x + 8 = 0 2
⇒ x = –1 ve
⇒ x2 – 5x + 4 = 0
2x = 8 ⇒ 2x = 23
⇒ (x – 1)(x – 4) = 0
⇒ x = 3 bulunur.
⇒ x = 1 veya x = 4
(Cevap A)
x = 1 denklemi sağlanamaz.
(Cevap C)

96
16. x 2 − 2x = x 2 − 2x − 3 + 3
denkleminin kökler toplamı kaçtır?

A) 1 B) 2 C) 4 D) 6 E) 9
12. 2x + 1 = 2x 2 + 2x + 5
2 2
x – 2x – 3 – x – 2x – 3 = 0
denkleminin çözüm kümesi aşağıdakilerden hangisidir? x 2 – 2x – 3 = t olsun. t2 – t = 0 ⇒ t(t –1) = 0
A) {–2} B) {–1} C) {1} ⇒ t = 0 veya t = 1
x 2 – 2x – 3 = 0 ⇒ x2 – 2x – 3 = 0
D) {–2, 1} E) {–1, 2} ⇒ x1 + x2 = 2 ve
2 2 2
(2x + 1) = 2x + 2x + 5 ⇒ 4x + 4x + 1 = 2x + 2x + 5
x 2 – 2x – 3 = 1 ⇒ x2 – 2x –4 = 0
⇒ 2x2 + 2x – 4 = 0
⇒ x3 + x4 = 2 bulunur.
⇒ x2 + x – 2 = 0
x1 + x2 + x3 + x4 = 4 elde edilir.
⇒ (x + 2)(x – 1) = 0
(Cevap C)
⇒ x = –2 veya x = 1
x = –2 denklemi sağlamaz. (Cevap C)
İkinci Dereceden Bir Bilinmeyenli Eşitsizlikler BÖLÜM 04 Test 13
1. (x – 3) (x – 7) ≤ 5 4 > x −1
5.
x+2
eşitsizliğini sağlayan kaç farklı x tam sayısı vardır?
eşitsizliğini sağlayan en büyük x tam sayısı kaçtır?
A) 6 B) 7 C) 8 D) 9 E) 10
x2 – 10x + 21 ≤ 5 ⇒ x2 – 10x + 16 ≤ 0 A) –2 B) –1 C) 1 D) 2 E) 3
4 4 – x2 + x – 2x + 2
⇒ (x – 2)(x – 8) ≤ 0 – x + 1 > 0 ⇒ >0
x+2 x+2
⇒2≤x<8
–x2 – x + 6
Eşitsizliği sağlayan 7 tam sayı vardır. ⇒ >0
x+2
(Cevap B)
–x2 – x + 6 = 0 ⇒ x2 + x – 6 = 0 x + 2 = 0 ⇒ x = –2
⇒ x = –3 veya x = 2

x –3 –2 2
+ 0 – + 0 –
Eşitsizliği sağlayan en büyük tam sayı 1 dir.
(Cevap C)

1 5
2. <0 6. ≤ −1
x 2 − 3 x − 10 x 2 + 6x
eşitsizliğini sağlayan x tam sayılarının toplamı kaçtır? eşitsizliğini sağlayan en küçük x tam sayısı ile
A) 6 B) 7 C) 8 D) 9 E) 10 en büyük x tam sayısının toplamı kaçtır?
x2 – 3x – 10 < 0 ⇒ (x + 2)(x –5) < 0 A) –7 B) –6 C) –5 D) –4 E) –3
⇒ –2 < x < 5 5 5 + x2 + 6x
+1≤0⇒ ≤0
Eşitsizliği sağlayan x tam sayılarının toplamı –1 + 0 + 1 + 2 + 3 + 4 = 9 dur. x2 + 6x x2 + 6x

(Cevap D)
x2 + 6x + 5 = 0 ⇒ x = –1 veya x = –5
x2 + 6x = 0 ⇒ x = –6 veya x = 0
x –6 –5 –1 0
Çözüm Yayınları

+ – 0 + 0 – +
sağlar sağlar

x in alabileceği en küçük tam sayı değeri –5, en büyük tam sayı değeri –1 ve toplamları
–6 dır.
(Cevap B)

x 2 − 2x ≥ 0 7. (m + 1)x2 + (2m + 2)x + 2m – 5 = 0


3.
x+4
denkleminin farklı iki gerçek kökü olduğuna göre,
eşitsizliğinin çözüm kümesi aşağıdakilerden hangisidir? aşağıdakilerden hangisi doğrudur?
A) (–4, 1] B) [–1, 4] C) (–4, 2] A) m < – 1 B) –1 < m < 6 C) –2 < m < 3
D) (–∞, –4) E) (–4, 0] ∪ [2, ∞) D) –6 < m < 1 E) m > 6
x2 + 2x = 0 ⇒ x = 0 veya x = 2, x + 4 = 0 ⇒ x = –4 ∆ < 0 ⇒ (2m + 2)2 – 4(m + 1)(2m – 5) > 0
⇒ –3m2 + 20m + 24 > 0
x –4 0 2
– + 0 – 0 + ⇒ m2 – 5m – 6 < 0
sağlar sağlar ⇒ (m + 1)(m – 6) < 0
Ç = (–4, 0] ∪ [2, ∞) ⇒ –1 < m < 6
(Cevap E) (Cevap B)

97

2
4. (3 − x )( x − 2) ≥ 0
3
( x + 5) 8. m nin kaç farklı tam sayı değeri için,

eşitsizliğini sağlayan x tam sayılarının toplamı kaçtır? x2 + mx + 4 = 0

A) –5 B) –4 C) –3 D) 2 E) 3 denkleminin reel (gerçek) kökü yoktur?

x –5 2 3
(x = 2 çift katlı köktür.)
A) 5 B) 6 C) 7 D) 8 E) 9
– + 0 + 0 –
∆ < 0 ⇒ m2 – 4 . 1 . 4 < 0

Eşitsizliği sağlayan x tam sayılarının toplamı: –4 – 3 – 2 – 1 + 0 + 1 + 2 + 3 = –4 ⇒ m2 – 16 < 0

(Cevap B) ⇒ (m + 4)(m – 4) < 0


⇒ –4 < m < 4
m ∈ {–3, –2, –1, 0, 1, 2, 3} (Cevap C)
Test 13 1. B 2. D 3. E 4. B 5. C 6. B 7. B 8. C 9. B 10. B 11. D 12. C 13. D 14. B 15. D 16. C

9. Bir tam sayının karesi, bu tam sayının 2 katının 8 fazlasından 13. ∀ ∈ R için,
küçüktür.
x2 – (m – 2)x + 9 > 0
Buna göre, bu koşula uyan tam sayıların toplamı kaçtır?
olduğuna göre, m nin alabileceği kaç farklı tam sayı
A) 4 B) 5 C) 6 D) 7 E) 8 değeri vardır?
x2 < 2x + 8 ⇒ x2 – 2x – 8 < 0
A) 8 B) 9 C) 10 D) 11 E) 12
⇒ (x + 2)(x – 4) < 0
a > 0 ve ∆ < 0 olmalıdır.
⇒ –2 < x < 4
a = 1 olduğundan a > 0 dır.
İstenen koşulu sağlayan tam sayıların toplamı: –1 + 0 + 1 + 2 + 3 = 5 tir.
∆ < 0 ⇒ (m – 2)2 – 4 . 9 < 0
(Cevap B)
⇒ m2 – 4m – 3 < 0
⇒ (m + 4)(m – 8) < 0
⇒ –4 < m < 8
(Cevap D)

10. 3 x ⋅ ( x − 3 )3
≤0 2 x − 32 ≤ 0
x+5 14.
x−3
eşitsizliğini sağlayan x tam sayılarının toplamı kaçtır?
eşitsizliğini sağlayan x tam sayılarının toplamı kaçtır?
A) –7 B) –4 C) 1 D) 3 E) 4
x–3 A) 6 B) 9 C) 12 D) 15 E) 16
3x > 0 olduğundan ≤ 0 olmalıdır.
x+5
2x – 32 = 0 ⇒ 2x – 25 = 0
x –5 3 ⇒x=5
–5 < x ≤ 3 bulunur.
+ – 0 + x 3 5
2x – 32 – – 0 +
Eşitsizliği sağlayan x tam sayılarının toplamı: –4 – 3 – 2 – 1 + 0 + 1 + 2 + 3 = –4 tür. x–3 – + +
Çözüm Yayınları

(Cevap B) Bölüm + – 0 +

Eşitsizliği sağlayan x tam sayılarının toplamı 4 + 5 = 9 dur.


(Cevap B)

11. x 2 − 2x − 15 ≤ 0
| x−2|
eşitsizliğinin çözüm kümesi aşağıdakilerden hangisidir? 15. 3x2 + (m – 2)x + m2 + 2m – 3 = 0

A) (–∞, –3] B) [–3, 2) C) (2, 5] denkleminin ters işaretli iki gerçek kökü bulunduğuna
göre, aşağıdakilerden hangisi doğrudur?
D) [–3, 5] – {2} E) [5, ∞)
x –3 5
A) m < –3 B) –1 < m < 3 C) 1 < m < 3
x≠2
x2 – 2x – 15 + – 0 +
D) –3 < m < 1 E) m > 1
x1 < 0 < x2 ⇒ x1 . x2 < 0
Ç = [–3, 5] – {2}
m2 + 2m – 3
(Cevap D) ⇒ <0
3
⇒ (m + 3)(m – 1) < 0
⇒ –3 < m < 1
(Cevap D)
98

12. f(x) = x2 – 2mx + 2m + 3


parabolü x eksenini kesmediğine göre, aşağıdakilerden
hangisi doğrudur?
16. x+5 + x+5 = 0
A) –3 < m < 1 B) –1 < m < 0 C) –1 < m < 3 x−3 x−3
D) m < 0 E) m > 3 eşitliğini sağlayan x tam sayılarının toplamı kaçtır?
2
∆ < 0 ⇒ 4m – 4 . 1 . (2m + 3) < 0 A) –16 B) –15 C) –12 D) –7 E) –5
⇒ 4m2 – 8m – 12 < 0 x+5 x+5 x+5
x–3 =–x–3 ⇒ x–3 #0
⇒ m2 – 2m – 3 < 0
⇒ –5 ≤ x < 3
⇒ (m + 1)(m – 3) < 0
–5 –4 – 3 – 2 – 1 + 0 + 1 + 2 = –12
⇒ –1 < m < 3
(Cevap C)
(Cevap C)
İkinci Dereceden Bir Bilinmeyenli Eşitsizlikler BÖLÜM 04 Test 14
5. (m – 2)x2 + (3m – 1)x + m + 3 = 0
1. ( x − 5 ) 2 ( x 2 − x − 2)
≤0
x+2 denkleminin ters işaretli iki gerçek kökü bulunduğuna
eşitsizliğini sağlayan kaç tane pozitif x tam sayısı vardır? göre, aşağıdakilerden hangisi doğrudur?

A) 3 B) 4 C) 5 D) 6 E) 7 A) –3 < m < 2 B) –2 < m < 3 C) m < –3


x2 – x – 2 = 0 ⇒ (x + 1)(x – 2) = 0 D) 2 < m < 3 E) m > 2
⇒ x = –1 veya x = 2 m+3
x1 . x2 < 0 ⇒ <0
(x – 5)2 = ⇒ x = 5 çift katlı köktür. m–2

x –2 –1 2 5 ⇒ –3 < m < 2
– + 0 – 0 + 0 + (Cevap A)

Eşitsizliği sağlayan pozitif tam sayılar x = 1, x = 2 ve x = 5 olmak üzere 3 tanedir.


(Cevap A)

2. (1 − x )3 ⋅ x 2
>0 6. x+5 ≤ x
x2 − 1
x−3
eşitsizliğini sağlayan en büyük x tam sayısı kaçtır? eşitsizliğinin çözüm kümesi aşağıdakilerden hangisidir?
A) –4 B) –3 C) –2 D) 1 E) 5 A) (–∞, –1) B) (3, 5]
^1 – x h .x 2

^ x – 1h^ x + 1h
20 C) (–1, 5] – {3} D) [–1, 3) ∪ [5, ∞)
x = 0 ve x = 1 çift katlı köktür. E) (3, ∞)
x –1 0 1 m+5 x + 5 – x2 + 3x
+ – 0 – – –x ≤ 0 ⇒ ≤0
m–3 m–3
Çözüm Yayınları

Eşitsizliği sağlayan en büyük x tam sayısı –2 dir. –x2 + 4x + 5


⇒ ≤0
(Cevap C) x–3
–(x – 5)(x + 1)
⇒ ≤0
x–3

x –1 3 5
+ 0 – + 0 –
Ç = [–1, 3) ⇒ [5, ∞)
(Cevap D)

3. ( x − 1) 4 x −1 − x +1 < 0
2
<0 7.
x − 2x − 15 x +1 x −1
eşitsizliğini sağlayan x tam sayılarının toplamı kaçtır? eşitsizliğini sağlayan en küçük pozitif x tam sayısı kaçtır?
A) 5 B) 6 C) 7 D) 8 E) 9 A) 1 B) 2 C) 3 D) 4 E) 5
x–1 x –3 1 5 (x – 1)2 – (x + 1)2 –4x
< 0 + – 0 – + <0⇒ <0
(x + 3)(x – 5) (x + 1)(x –1) (x + 1)(x –1)
x = 1 çift katlı köktür. –3 < x < 5 x –1 0 1
+ – 0 + –
Eşitsizliği sağlayan x tam sayılarının toplamı –2 – 1 + 0 + 2 + 3 = 6 dır.
(Cevap B) Eşitsizliği sağlayan en küçük pozitif tam sayı 2 dir.
(Cevap B)

99

8. (m – 2)x2 – 4x + m + 5 = 0

4. ( x + 5)( x − 4) ≥ 0 denkleminin ters işaretli iki gerçek kökü bulunduğuna


4x − x 2 göre, aşağıdakilerden hangisi doğrudur?
eşitsizliğini sağlayan x tam sayılarının toplamı kaçtır? A) m < –5 B) –5 < m < 2 C) –2 < m < 5
A) –15 B) –9 C) –5 D) 5 E) 10 D) 2 < m < 5 E) m > 2
(x + 5)(x – 4) m+5
≥ 0 x = 4 çift katlı köktür. x1 . x2 < 0 ⇒ <0
x(4 – x) m–2
x –5 0 4 ⇒ –5 < m < 2
+ 0 – – (Cevap B)
Eşitsizliği sağlayan x tam sayılarının toplamı –5 – 4 – 3 – 2 – 1 = –15 tir.
(Cevap A)
Test 14 1. A 2. C 3. B 4. A 5. A 6. D 7. B 8. B 9. C 10. C 11. B 12. E 13. E 14. C 15. A 16. B

9. x2 – (m + 3)x + 16 = 0 13. y = x + n doğrusu,


denkleminin gerçek kökleri olmadığına göre, y = x2 + 3x + 2
aşağıdakilerden hangisi doğrudur?
parabolünü iki noktada kestiğine göre, aşağıdakilerden
A) m < –11 B) –11 < m < 0 C) –11 < m < 5 hangisi doğrudur?

D) 5 < m < 11 E) m > 11 A) n < –2 B) n > –1 C) –1 < n < 1


2
∆ < 0 ⇒ (m + 3) – 4 . 1 . 16 < 0 D) 0 < n < 1 E) n > 1
⇒ (m + 3 – 8)(m + 3 + 8) < 0 x2 + 3x + 2 = x + n ⇒ x2 + 2x + 2 – n = 0
⇒ (m – 5)(m + 11) < 0 ∆ > 0 olmalıdır. 4 – 4(2 – n) > 0 ⇒ n > 1
⇒ –11 < m < 5 (Cevap E)
(Cevap C)

10. x2 − x − 6 < 0
| 3x − 6 |

eşitsizliğini sağlayan kaç farklı x tam sayısı vardır?

A) 1 B) 2 C) 3 D) 4 E) 5 14. (2 x − 32)( x 3 + 8)
≤0
(x + 2)(x – 3) | x 2 − 1|
<0
3|x – 2|
(x + 2)(x – 3) < 0 ve x – 2 ≠ 0 olmalıdır.
eşitsizliğini sağlayan x tam sayılarının toplamı kaçtır?
–2 < x < 3 ve x ≠ 2 bulunur. A) 9 B) 10 C) 12 D) 14 E) 15
Eşitsizliği sağlayan –1, 0, 1 olmak üzere 3 farklı tam sayı vardır.
x –2 5
(Cevap C) 2 x – 25 – – 0 +
x3 + 8 – 0 + +
Bölüm + – +
Çözüm Yayınları

–2 ≤ x ≤ 5 ve x ≠ ∓ 1 olmalıdır. Toplamları –2 + 0 + 2 + 3 + 4 + 5 = 12 bulunur.


(Cevap C)

11. x2 + x + 2 ≤ 0
| 2x − 1 | −5

eşitsizliğini sağlayan kaç tane x tam sayısı vardır?

A) 3 B) 4 C) 5 D) 6 E) 8
x2 + x + 2 = 0 denklemi için ∆ < 0 olduğundan x2 + x + 2 < 0 dır.
15. 1− | 2 − x |
|2x – 1| – 5 < 0 olmalıdır. ≤0
5− | x − 2 |
|2x – 1| < 5 ⇒ –5 < 2x – 1 < 5
⇒ –2 < x < 3 eşitsizliğini sağlayan x tam sayılarının toplamı kaçtır?
Eşitsizliği sağlayan –1, 0, 1, 2 olmak üzere 4 tam sayı vardır.
A) 16 B) 19 C) 20 D) 21 E) 22
(Cevap B) 1–t
|x – 2| = t olsun. ≤0⇒1<t<5
5–t
1 ≤ |x – 2| < 5 ⇒ 1 ≤ 2 – x < 5 veya 1 ≤ x – 2 < 5
12. f(x) = (m – 2)x2 – (1 – m)x + m – 1 ⇒ –3 < x ≤ 1 veya 3 ≤ x < 7
fonksiyonu veriliyor. Toplamları: –2 –1 + 0 + 1 + 3 + 4 + 5 + 6 = 16
(Cevap A)
∀x ∈ R için f(x) > 1
100
olduğuna göre, aşağıdakilerden hangisi doğrudur?

5 <m<2 1
A) B) < m < 3 C) 2 < m < 3
3 3

3 16. x − 4 ≤ x + 16
D) m < E) m > 3
5 x −2
f(x) > 1 ⇒ f(x) –1 > 0 eşitsizliğini sağlayan kaç farklı x tam sayısı vardır?
⇒ (m – 2)x2 + (m – 1)x + m – 2 > 0
A) 8 B) 9 C) 10 D) 12 E) 16
m – 2 > 0 ⇒ m > 2 ve ∆ < 0 olmalıdır.
(ñx – 2)(ñx + 2) ≤ óx+16 ⇒ óx + 2 ≤ óx+16
∆ < 0 ⇒ (m – 1)2 – 4(m – 2)(m – 2) < 0
ñx – 2
⇒ x + 4ñx + 4 ≤ x + 16
⇒ 3m2 – 14m + 15 > 0
⇒ ñx ≤ 3
⇒ (3m – 5)(m – 3) > 0
5 ⇒ x ≤ 9 bulunur.
⇒ m < veya m > 3
3 Ayrıca ñx – 2 ≠ 0 ⇒ x ≠ 4 olmalıdır.
m > 3 sağlar.
(Cevap E) 0, 1, 2, 3, 5, 6, 7, 8, 9 olmak üzere 9 tane x tam sayısı vardır. (Cevap B)
İkinci Dereceden Bir Bilinmeyenli Eşitsizlikler BÖLÜM 04 Test 15
1. (x2 – x – 2) (9 – x2) > 0
5. ( x − 3 ) 4 ( x + 4 )5
≤0
eşitsizliğinin çözüm kümesi aşağıdakilerden hangisidir? x −1

A) (–3, 1) ∪ (2, 3) B) (–3, –1) ∪ (2, 3) eşitsizliğinin çözüm kümesi aşağıdakilerden hangisidir?

C) (–3, 1) D) (–3, –1] ∪ [2, 3) A) (–∞, –4] B) (–4, 3] C) (1, 3]

E) (2, 3) D) [–4, 1) ∪ {3} E) (1, ∞)


(x – 3)2(x + 4)
(x + 1)(x – 2)(3 – x)(3 + x) > 0 ≤ 0, 3 çift katlı köktür.
x–1
x –3 –1 2 3
– 0 + 0 – 0 + 0 – x –4 1 3
sağlar sağlar + 0 – + 0 –
Ç = [–4, 1) ∪ {∞}
Ç = (–3, –1) ∪ (2, 3)
(Cevap D)
(Cevap B)

6. 12 > 1
2. ( x − 1)( x 2 − 3 x + 2) x2 + x
≤0
x2 − x − 6
eşitsizliğini sağlayan en küçük x tam sayısı kaçtır?
eşitsizliğini sağlayan pozitif tam sayıların toplamı kaçtır?
A) –5 B) –4 C) –3 D) 1 E) 2
A) 2 B) 3 C) 4 D) 6 E) 9 12 12 – x2 – x
– 1 > 0 ⇒ >0
(x – 1)(x – 1)(x – 2) x2 + x x(x + 1)
≤ 0 x = 1 çift katlı köktür.
(x + 2)(x –3) –(x + 4)(x – 3)
⇒ >0
x –2 1 2 3 x(x + 1)
– + 0 + 0 – +
x –4 –1 0 3
Çözüm Yayınları

x = 1 ve x = 2 olabilir. Toplamları 3 tür. – 0 + – + 0 –


(Cevap B) Eşitsizliği sağlayan en küçük x tam sayısı –3 tür.
(Cevap C)

7. x <1
x −1 x

3. ( 4 − x 2 )( x 2 − 2x )2 eşitsizliğinin çözüm kümesi aşağıdakilerden hangisidir?


≤0
x −1
A) (–1, 1) B) (0, 1) C) (–∞, 0)
eşitsizliğini sağlayan kaç tane negatif x tam sayısı vardır?
D) (1, 2) E) (1, ∞)
A) 1 B) 2 C) 3 D) 4 E) 5
x 1 x2 – x + 1
(2 – x) . (2 + x) . x2 . (x – 2)2 –(x + 2) . x2 . (x – 2)3 – < 0 ⇒ <0
≤0⇒ x–1 x x(x – 1)
x–1 x–1
2
x – x + 1 = 0 denkleminde ∆ < 0 ve kök yoktur.
x –2 0 1 2
+ 0 – 0 – + 0 – x 0 1 Ç = (0, 1)
+ – +
(Cevap B)
–2 ve –1 olmak üzere 2 tanedir.
(Cevap B)

101

8. 2x − 3 ≤ 1
x 2 + 3 x − 10 ≤ 0 x+2
4.
1− x eşitsizliğinin çözüm kümesi aşağıdakilerden hangisidir?
eşitsizliğinin çözüm kümesi aşağıdakilerden hangisidir? A) (–∞, –2) B) [–5, 2) C) (–2, 5]
A) [–5, –1) B) [–5, 0] C) [–5, 2] D) (–2, 3] E) (5, ∞)
D) (1, 2] E) [–5, 1) ∪ [2, ∞) 2x – 3 2x – 3 – x – 2
–1 ≤ 0 ⇒ ≤0
x+2 x+1
(x + 5)(x – 2)
≤0 x–5
1–x
⇒ ≤0
x –5 1 2 x+2
+ 0 – + 0 – ⇒ –2 < x ≤ 5 bulunur.
Ç = (–2, 5] tir.
Ç = [–5, 1) ∪ [2, ∞) (Cevap E)
(Cevap C)
Test 15 1. B 2. B 3. B 4. E 5. D 6. C 7. B 8. C 9. E 10. C 11. D 12. E 13. D 14. C 15. A

9. x2 – (m – 1)x – m = 0 13. m < 0 olmak üzere,


denkleminin farklı iki negatif kökü olduğuna göre, mx2 + (m – 4)x – 3 = 0
aşağıdakilerden hangisi doğrudur?
denkleminin kökleri p ve q dur.
A) m < –2 B) 1 < m < 2 C) –1 < m < 1
|p| < |q|
D) 0 < m < 1 E) m < 0 ve m ≠ –1
olduğuna göre, aşağıdakilerden hangisi doğrudur?
c b
∆ < 0, > 0, – < 0 olmalıdır. A) p < 0 < q B) p < q < 0 C) 0 < p < q
a a
2 2
∆ = (m – 1) – 4 . 1 . (–m) ⇒ ∆ = (m + 1) < 0 dır. D) q < p < 0 E) 0 < q < p
m 0 1 –3
c x1 . x 2 = ⇒ x1 . x2 > 0 (Kökler aynı işaretli)
= –m – 0 – + m
a
b –(m – 4)
– =m–1 – + 0 + x1 + x 2 = ⇒ x1 . x2 < 0 (Her iki kök negatif)
a m
m < 0 ve m ≠ –1 olmalıdır. |p| < |q| ⇒ q < p < 0 dır.
(Cevap E) (Cevap D)

10. |x2 – x + 2| ≤ 8
eşitliğini sağlayan kaç tane x tam sayısı vardır?

A) 4 B) 5 C) 6 D) 7 E) 8
x2 – x + 2 = 0 denkleminde ∆ < 0 olduğunda x2 – x + 2 < 0 dır.
14. y
x2 – x + 2 ≤ 8 ⇒ x2 – x – 6 ≤ 0
⇒ (x + 2)(x – 3) ≤ 0
⇒ –2 ≤ x ≤ 3
y = f(x)

Eşitsizliği –2, –1, 0, 1, 2, 3 olmak üzere 6 tane tam sayı sağlar.


(Cevap C)
x
Çözüm Yayınları

–3 0 3 5

Şekildeki grafiğe göre,


11. f(x) = (m – 2)x2 – (2 – m)x – 5 f (x)
≤0
veriliyor. x
− 1

∀x ∈ R için, f(x) < 0 eşitsizliğinin çözüm kümesi aşağıdakilerden hangisidir?

olduğuna göre, aşağıdakilerden hangisi doğrudur? A) (–∞, –2] B) [–2, 5]

A) m < –18 B) m > 2 C) 2 < m < 18 C) [–3, 1) ∪ [3, 5] D) (1, 3]

D) –18 < m < 2 E) m > – 18 E) [5, ∞)


m – 2 < 0 ⇒ m < 2 ve ∆ < 0 olmalıdır. x –3 1 3 5
∆ < 0 ⇒ (m – 2)2 – 4 . (m – 2) . (–5) < 0 f(x) – 0 + + 0 – 0 +
x–1 – – 0 + + +
⇒ m2 + 16m – 36 < 0
Bölüm – 0 – + 0 – 0 +
⇒ (m + 18)(m – 2) < 0
(Cevap D) Ç = [–3, 1) ∪ [3, 5]
(Cevap C)

102

12. y = x + 2
doğrusu,

y = x2 + mx + 3

parabolünü farklı iki noktada kestiğine göre, 15. x−2 = 2−x


aşağıdakilerden hangisi doğrudur? x+3 x+3

A) m < –3 B) –3 < m < 1 C) –1 < m < 3 eşitliğini sağlayan kaç tane x tam sayısı vardır?

D) –2 < m < 1 E) m < –1 ∨ m > 3 A) 5 B) 6 C) 7 D) 8 E) 9


x2 + mx + 3 = x + 2 ⇒ x2 + (m – 1)x + 1 = 0 x–2
≤ 0 ⇒ –3 < x ≤ 2
∆ < 0 ⇒ (m – 1)2 – 4 . 1 . 1 > 0 x+3
⇒ m2 + 2m – 3 > 0 Eşitsizliği; –2, –1, 0, 1, 2 olmak üzere 5 tam sayı sağlar.
⇒ (m + 1)(m – 3) > 0 (Cevap A)
⇒ m < –1 veya m > 3 (Cevap E)
İkinci Dereceden Bir Bilinmeyenli Eşitsizlikler BÖLÜM 04 Test 16
1. x2 ≤ 3x + 10 x2 + x ≥ 2
5.
eşitsizliğini sağlayan x tam sayılarının toplamı kaçtır? x+3

A) 8 B) 9 C) 10 D) 11 E) 12 eşitsizliğinin çözüm kümesi aşağıdakilerden hangisidir?

x2 – 3x – 10 ≤ 0 ⇒ (x + 2)(x – 5) ≤ 0 A) (–∞, –3) B) (–3, 0] C) (–3, 3]


⇒ –2 ≤ x ≤ 5
D) (–3, –2] ∪ [3, ∞) E) (–3, ∞)
–2 – 1 + 0 + 1 + 2 + 3 + 4 + 5 = 12
x2 + x x2 – x – 2x – 6
(Cevap E) – 2 ≥ 0 ⇒ ≥0
x+3 x+3
x2 – x – 6
⇒ ≥0
x+3
(x + 2)(x – 3)
⇒ ≥0
x+3
x –3 –2 3
– + 0 – 0 +
Ç = (–3, –2] ∪ [3, ∞)
(Cevap D)
5
2. ( x − 2)( x + 3)
≤0
( x − 1)6 2 ≤ 1
6.
eşitsizliğinin çözüm kümesi aşağıdakilerden hangisidir? x−2 x−7

A) (–∞, –3] B) [–3, 1) C) [–3, 2] – {1} eşitsizliğini sağlayan kaç farklı x doğal sayısı vardır?

D) (1, 2] E) [2, ∞) A) 5 B) 6 C) 7 D) 8 E) 9
2 1 2x – 14 – x + 2
(x – 2)(x + 3) x –3 1 2 – ≤ 0 ⇒ ≤0
≤ 0 x–2 x–7 (x – 2)(x – 7)
(x – 1)2 + 0 – – 0 +
x – 12
⇒ ≤0
Ç = [–3, 2] – {1} (x – 2)(x – 7)
(Cevap C) x 2 7 12
– + – 0 +
Çözüm Yayınları

Eşitsizliği sağlayan doğal sayılar 0, 1, 8, 9, 10, 11, 12 olmak üzere 7 tanedir.


(Cevap C)

7. Bir tam sayının karesi, aynı sayının 12 fazlasından küçüktür.


3. x≤ 6
x −1 Buna göre, bu koşula uyan sayıların toplamı kaçtır?
eşitsizliğini sağlayan kaç tane x doğal sayısı vardır? A) 3 B) 4 C) 5 D) 6 E) 7
A) 2 B) 3 C) 4 D) 5 E) 6 x2 < x + 12 ⇒ x2 – x – 12 < 0
6 x2 – x – 6
⇒ (x + 3)(x – 4) < 0
x– ≤ 0 ⇒ ≤0
x–1 x–1
⇒ –3 < x < 4
(x + 2)(x – 3) Toplamları –2 – 1 + 0 + 1 + 2 + 3 = 3 tür.
⇒ ≤0
x–1 (Cevap A)
x –2 1 3

– 0 + – 0 +

Eşitsizliği sağlayan x = 2 ve x = 3 olmak üzere iki doğal sayı vardır.


(Cevap A)
8. | x − 2 | −3
≤0
x+6 103
eşitsizliğinin çözüm kümesi aşağıdakilerden hangisidir?

A) (–∞, –6) B) (–6, –1]


4. x 2 − 3x − 4 ≤ 0 C) (–6, 5] D) (–∞, –6) ∪ [–1, 5]
x3 − 8
E) (–∞, –6) ∪ [5, ∞)
eşitsizliğini sağlayan pozitif x tam sayılarının toplamı
|x – 2| – 3 = 0 ⇒ x – 2 = –3 veya x – 2 = 3
kaçtır?
⇒ x = –1 veya x = 5
A) 7 B) 8 C) 9 D) 10 E) 12 x –6 –1 5
(x + 1)(x – 4) |x – 2| – 3 + + 0 – 0 +
≤0
(x – 2)(x2 + 2x + 4) x+6 – 0 + + +
x –1 2 4 Bölüm – + – +

– 0 + – 0 +
Ç = (–∞, –6) ∪ [–1, 5]
Eşitsizliği sağlayan x doğal sayılarının toplamı 3 + 4 = 7 dir.
(Cevap D)
(Cevap A)
Test 16 1. E 2. C 3. A 4. A 5. D 6. C 7. A 8. D 9. A 10. D 11. B 12. A 13. D 14. E 15. A

( − x + 3)( x + 4) 13. (x – 2)2 – 4 · |x – 2| + 3 ≤ 0


9. ≥0
| x − 2 | +1 eşitsizliğini sağlayan x tam sayılarının toplamı kaçtır?
eşitsizliğinin çözüm kümesi aşağıdakilerden hangisidir? A) 6 B) 9 C) 10 D) 12 E) 15
A) [–4, 3] B) [0, 3] C) [1, 3] |x – 2| = t olsun. t2 – 4t + 3 ≤ 0 ⇒ (t – 1)(t – 3) ≤ 0
⇒1≤t≤3
D) [2, 4] E) [–4, 1]
1 ≤ |x – 2| ≤ 3 ⇒ 1 < –(x – 2) ≤ 3 veya 1 ≤ x – 2 ≤ 3
Payda pozitif olduğundan payın işaretine bakılır. ⇒ –1 ≤ x ≤ 1 veya 3 ≤ x ≤ 5
(–x + 3)(x + 4) ≥ 0 x –4 3 Eşitsizliği sağlayan x tam sayılarının toplamı: (–1 + 0 + 1) + (3 + 4 + 5) = 12 dir.
+ 0 – 0 –
(Cevap D)
Ç = [–4, 3]
(Cevap A)

10. ∀x ∈ R için,
x2 – 2x + m – 5 > 0 14. y
y = f(x)
olduğuna göre, m nin alabileceği en küçük tam sayı
değeri kaçtır?

A) 4 B) 5 C) 6 D) 7 E) 8 x
a = 1 pozitif olduğundan ∆ < 0 olmalıdır. –3 0 1
∆ < 0 ⇒ 4 – 4 . 1 . (m – 5) < 0
⇒m>6
m nin alabileceği en küçük tam sayı değeri 7 dir.
(Cevap D)
Çözüm Yayınları

y = f(x) fonksiyonunun grafiği şekildeki parabol eğrisi


olduğuna göre,
f (x)
2
≤0
x − 3x − 4
11. y = x + m doğrusu, eşitsizliğinin çözüm kümesi aşağıdakilerden hangisidir?
y= x2 –x–2 A) [–3, 1] B) (–1, 1] C) (–1, 4)
parabolünü farklı iki noktada kestiğine göre, D) [–3, 4) E) [–3, –1) ∪ [1, 4)
aşağıdakilerden hangisi doğrudur?
x –3 –1 1 4
A) m < 0 B) m > – 3 C) –5 < m < –3 f(x) + 0 – – 0 + +
x2 – 3x – 4 + + 0 – – 0 +
D) m > 0 E) m > 3
Bölüm + 0 – + 0 – +
x2 – x – 2 = x + m ⇒ x2 – 2x – 2 – m = 0
∆ < 0 ⇒ 4 – 4 . 1 . (m – 5) < 0 Ç = [–3, –1) ∪ [1, 4)
⇒ m > –3 (Cevap E)
(Cevap B)

104

12. x2 – (m – 2)x – 3 = 0
15. (m – 2)x2 + 3x + m + 7 = 0
denkleminin kökleri x1 ve x2 dir.
denkleminin kökleri x1 ve x2 dir.
|x1| > x2
x1 < 1 < x2
olduğuna göre, aşağıdakilerden hangisi doğrudur?
olduğuna göre, m nin alabileceği tam sayı değerlerinin
A) m < 2 B) 2 < m < 3 C) 3 < m < 4 toplamı kaçtır?

D) 2 < m < 9 E) m > 4 A) –5 B) –2 C) 1 D) 3 E) 5


c
negatif olduğundan denklemin ters işaretli iki gerçek kökü vardır. a . f(1) < 0 ⇒ (m – 2) . (m – 2 + 3 + m + 7) < 0
a b ⇒ (m – 2)(2m + 8) < 0
x1 < 0 < x2 ve |x1| > x2 ⇒ – < 0
a ⇒ –4 < m < 2
⇒m–2<0
m nin alabileceği tam sayı değerlerinin toplamı –3, –2 – 1 + 0 + 1= –5 tir.
⇒m<2
(Cevap A)
(Cevap A)
İkinci Dereceden Bir Bilinmeyenli Eşitsizlikler BÖLÜM 04 Test 17
5. x2 – (m – 2)x – 3 = 0
1. ( x − 2)( x 2 − x − 2)
≤0
( x − 5 )3 denkleminin kökleri x1 ve x2 dir.
eşitsizliğini sağlayan x tam sayılarının toplamı kaçtır?
x12 + 2x1x 2 + x 2 2 ≤ 5

A) 6 B) 7 C) 9 D) 10 E) 12
(x – 2)(x – 2)(x + 1) (x – 2)2(x + 1) olduğuna göre, m nin alabileceği değerlerin kümesi
≤0⇒ ≤0 aşağıdakilerden hangisidir?
x–5 x–5

A) [–7, 3] B) [–5, 1] C) [–3, 5]


x –1 2 5
+ 0 – 0 – + D) [–3, 7] E) [1, 5]
|x1 + x2| ≤ 5 ⇒ |m – 2| ≤ 5
Eşitsizliği sağlayan x tam sayılarının toplamı: –1 + 0 + 1 + 2 + 3 + 4 = 9 dur. ⇒ –5 ≤ m – 2 ≤ 5
(Cevap C) ⇒ –3 ≤ m ≤ 7
Ç = [–3, 7]
(Cevap D)

2. x 2 (1 − x 2 )
≤0
x 2 + 2x + 1

eşitsizliğini sağlayan x tam sayılarının toplamı kaçtır? 6. 3 x ≤ 2


x+3
A) –2 B) –1 C) 0 D) 1 E) 2
eşitsizliğini sağlayan x tam sayılarının toplamı kaçtır?
x2(1 – x)(1 + x)
≤0
(x + 1)2 A) 8 B) 9 C) 11 D) 15 E) 18
3x 3x – 2x – 6
x –1 0 1 – 2 ≤ 0 ⇒ ≤0
– + 0 + 0 – x+3 x+3

(... – 3 – 2) + 0 + (1 + 2 + 3 + ...) = 1 x–6
⇒ ≤0
Çözüm Yayınları

x+3
(Cevap D)

⇒ –3 < x ≤ 6
Eşitsizliği sağlayan x tam sayılarının toplamı –2 – 1 + 0 + 1 + 2 + 3 + 4 + 5 + 6 = 18
(Cevap E)

7. 1 ≤ 1
x−2 x+3
3 2
3. x + x − 4 x − 4 ≤ 0
x−6 eşitsizliğin çözüm kümesi aşağıdakilerden hangisidir?

eşitsizliğini sağlayan x tam sayılarının toplamı kaçtır? A) (–∞, –3) B) (–3, 2) C) (–2, 3)

A) 3 B) 5 C) 6 D) 8 E) 11 D) [–5, 3) E) (2, ∞)
x2(x + 1) – 4(x + 1) (x + 1)(x –2)(x + 2) 1 1 x+3–x+2
≤0⇒ ≤0 – ≤0⇒ ≤0
x–6 x–6 x–2 x+3 (x – 2)(x + 3)
x –2 –1 2 6 5
+ 0 – 0 + 0 – + ⇒ ≤0
(x – 2)(x + 3)

(... – 3 – 2) + 0 + (1 + 2 + 3 + ...) = 1 x –3 2
+ – +
(Cevap E)
Ç = (–3, 2)
(Cevap B)
105

8. (m + 2)x2 + (2m – 3)x + m – 7 = 0

4. ( x − 3) 4 ( x + 4) denkleminin kökleri x1 ve x2 dir.


≤0
( x − 1)3
x1 < 0 < x2
eşitsizliğinin çözüm kümesi aşağıdakilerden hangisidir?
olduğuna göre, m nin alabileceği değerlerin kümesi
A) (–∞, –4) B) [–4, 3] C) [–4, 1) ∪ {3} aşağıdakilerden hangisidir?

D) (1, 3] E) [3, ∞) A) (–7, –2) B) (–7, 2) C) (–2, 7)


2
(x – 3) (x + 4) D) (2, 7) E) (2, 3)
≤0
x–1 m–7
x1 . x2 < 0 ⇒ <0
x –4 1 3 m+2
+ 0 – + 0 + ⇒ –2 < m < 7
Ç = [–4, 1) ∪ {3} Ç = (–2, 7)
(Cevap C) (Cevap C)
Test 17 1. C 2. D 3. E 4. C 5. D 6. E 7. B 8. C 9. C 10. B 11. E 12. E 13. D 14. E 15. C

9. x2 – (m – 4)x + 2m – 18 = 0 1− | x − 2 |
13. ≤0
denkleminin kökleri x1 ve x2 dir. 5 −|2−x|

x1 < 0 < x2 ve |x1| < |x2| eşitsizliğini sağlayan x tam sayılarının toplamı kaçtır?

olduğuna göre, aşağıdakilerden hangisi doğrudur? A) 10 B) 12 C) 15 D) 16 E) 18


|x – 2| = t olsun. |2 – x| = |x – 2| = t olur.
A) m < –4 B) –4 < m < –2 C) 4 < m < 9
t 1 5
D) m > 9 E) 4 < m < 7 1–t 1–t
≤0 + 0 – +
c b 5–t 5–t
< 0 ve – > 0 olmalıdır. 1≤t<5
a a
1 ≤ t < 5 ⇒ 1 ≤ |x – 2| < 5
m 4 9
⇒ 1 ≤ x – 2 < 5 veya –5 < x – 2 ≤ –1
2m – 18 – 0 – +
⇒ 3 ≤ x < 7 veya –3 < x ≤ 1
m–4 – + 0 +
Eşitsizliği sağlayan x tam sayılarının toplamı
4<m<9
3 + 4 + 5 + 6 – 2 – 1 + 0 + 1 = 16 dır.

(Cevap C) (Cevap D)

14. f(x) = ax2 + bx + c


10. − x 2 + 3 x + 10 ≥ 0
| x − 2 | +1 fonksiyonu x1 < x2 olmak üzere, x eksenini x1 ve x2 apsisli
eşitsizliğini sağlayan x tam sayılarının toplamı kaçtır? noktalarda kesmektedir.

A) 10 B) 12 C) 15 D) 16 E) 20 a · f(1) < 0
İfadenin paydası pozitif olduğundan payına bakılır.
f(1) · f(2) < 0
–x2 + 3x + 10 ≥ 0 ⇒ –(x + 2)(x – 5) ≥ 0
x –2 5 –2 ≥ x ≥ 5 olduğuna göre, aşağıdakilerden hangisi doğrudur?
– 0 + 0 –
A) x1 < x2 < 1 B) x1 < 0 < x2 C) 2 < x1 < x2
Çözüm Yayınları

Eşitsizliği sağlayan x tam sayılarının toplamı –2 –1 + 0 + 1 + 2 + 3 + 5 = 12


(Cevap B) D) 1 < x1 < 2 < x2 E) x1 < 1 < x2 < 2
1 sayısı kökler arasında olacağından 2 sayısı kökler dışında olur.
x1 < 1 < x2 < 2 dir.
(Cevap E)

11. y = 2x + k doğrusu
y = x2 + kx + 2

parabolünü farklı iki noktada kestiğine göre,


aşağıdakilerden hangisi doğrudur?
15. y y = f(x)
A) k < 2 B) –2 < k < 0 C) 0 < k < 2 y = g(x)

D) –2 < 0 < k E) k < –2 veya k > 2


x2 + kx + 2 = 2x + k ⇒ x2 + (k –)x + 2 – k = 0
∆ > 0 ⇒ (k – 2)2 – 4(2 – k) > 0 x
–3 0 2 5
⇒ k2 – 4 > 0
⇒ k < –2 veya k > 2
(Cevap E)

106 Yukarıda f(x) ve g(x) fonksiyonlarının grafikleri verilmiştir.

Buna göre,

12. ∀x ∈ R için, f(x) · g(x + 2) < 0

x2 + 1 eşitsizliğinin çözüm kümesi aşağıdakilerden hangisidir?


>0
mx 2 − 6x + m − 8 A) (0, 3) B) (1, 4)

olduğuna göre, aşağıdakilerden hangisi doğrudur? C) (–3, –2) ∪ (2, 5) D) (3, 6)

A) –9 < m < 0 B) –8 < m < 0 C) 0 < m < 4 E) (4, 7)


x –3 –2 2 5
D) 0 < m < 9 E) m > 9
f(x) – 0 + + 0 – 0 +
mx2 + 6x + m – 8 > 0
g(x + 2) – – 0 + + +
m > 0 ve ∆ < 0 olmalıdır.
Çarpım + – + – +
∆ < 0 ⇒ 36 – 4m(m – 8) < 0
⇒ m < – 1 veya m > 9 Ç = (–3, –2) ∪ (2, 5)

(Cevap E) (Cevap C)
İkinci Dereceden Bir Bilinmeyenli Eşitsizlikler BÖLÜM 04 Test 18
1. x3 − 1 5. x ≤ 1
≤0
x2 − 4 x − 12 x+4 x−2

eşitsizliğinin çözüm kümesi aşağıdakilerden hangisidir? eşitsizliğinin çözüm kümesi aşağıdakilerden hangisidir?

A) (–∞, 1] B) (–2, 6) A) (–∞, –4) B) [–1, 2] C) [–1, 2)

C) (–∞, –2) ∪ [1, 6) D) [2, 6) D) (–4, –1] ∪ (2, 4] E) [3, ∞)


2
x 1 x – 2x – x – 4
E) (6, ∞) – ≤ 0 ⇒ ≤0
x+4 x–2 (x + 4)(x – 2)
2
(x – 1)(x + x + 1)
≤ 0, x2 + x + 1 = 0 (∆ < 0) (x + 1)(x – 4)
(x + 2)(x – 6) ⇒ ≤0
(x + 4)(x – 2)
x –2 1 6
– + 0 – + x –4 –1 2 4
+ – 0 + – 0 +
Ç = [–∞, –2) ∪ [1, 6)
Ç = (–4, –1] ∪ (2, 4] (Cevap D)
(Cevap C)
6. (m – 2)x2 – (3m – 1)x – 4 = 0
denkleminin kökleri x1 ve x2 dir.
4 2
x − 4x ≤ 0 x1 < 2 < x2
2.
x2 + 1
olduğuna göre, aşağıdakilerden hangisi doğrudur?
eşitsizliğinin çözüm kümesi aşağıdakilerden hangisidir?
A) –5 < m < 0 B) 0 < m < 2
A) [–2, 2] B) [–4, 1] C) [–1, 4]
C) m < –5 ∨ m > 2 D) –5 < m < 2
D) [–4, 4] E) [2, ∞)
x2(x – 2)(x + 2)
E) m > 0
≤0 a . f(2) < 0 ⇒ (m – 2)(4m – 8 – 6m + 2 – 4) < 0
x2 + 1
⇒ (m – 2)(–2m – 10) < 0

x –2 0 2 x –5 2
Ç = [–2, 2]
a . f(2) – 0 + 0 –
Çözüm Yayınları

– 0 + 0 – 0 +
(Cevap A)
m < –5 veya m > 2 olmalıdır.
(Cevap C)

7. x2 – (m2 – 4)x + n – 5 = 0

(2 − x )5 ( x − 1) 4 denkleminin kökleri x1 ve x2 dir.


3. ≤0
9 − x2 x1 < 0 < x2
eşitsizliğinin çözüm kümesi aşağıdakilerden hangisidir? |x1| = x2
A) (–3, 2] ∪ (3, ∞) B) (–∞, –3) olduğuna göre, m + n toplamının alabileceği en büyük
C) (–3, 1) D) [1, 2] tam sayı değeri kaçtır?

E) (–∞, –3) ∪ [2, 3) ∪ {1} A) 4 B) 5 C) 6 D) 7 E) 8


(2 – x)(x – 1)2 |x1| = x2 ⇒ –x1 = x2
≤0
(3 – x)(3 + x) ⇒ x1 + x 2 = 0
⇒ m2 – 4 = 0
x –3 1 2 3 ⇒ m = –2 veya m = 2
– + 0 + 0 – +
c
Ç = (–∞, –3) ∪ [2, 3) ∪ {1} < 0 ⇒ n – 5 < 0 m = 2 ve n = 4 için
a
(Cevap E) ⇒ n < 5 m + n = 6 olur.
(Cevap C) 107

8. (m – 1)x2 – 5x + m – 1 = 0
denkleminin kökleri x1 ve x2 dir.
4. −x 2 + 3x − 2 ≥ 0
x2 − 4 x1 < 2 < x2
eşitsizliğinin çözüm kümesi aşağıdakilerden hangisidir? olduğuna göre, aşağıdakilerden hangisi doğrudur?
A) (–2, 1] B) (–∞, –2) C) [1, 2) A) m < –1 B) –1 < m < 0 C) 0 < m < 1
D) (–2, 2) E) (2, ∞) D) 1 < m < 3 E) m > 3
–(x2 – 3x + 2) –(x – 1)(x – 2) a . f(2) < 0 ⇒ (m – 1)(4m – 4 – 10 + m – 1) < 0
≥0⇒ ≥0
(x – 2)(x + 2) (x – 2)(x + 2) ⇒ (m – 1)(5m – 15) < 0

x 1 3
x –2 1 2 a . f(2) + 0 – 0 +
Ç = (–2, 1]
– + 0 – –
(Cevap A) 1 < m < 3 olmalıdır. (Cevap D)
Test 18 1. C 2. A 3. E 4. A 5. D 6. C 7. C 8. D 9. E 10. C 11. E 12. A 13. A 14. B 15. B 16. C

9. x2 – 2mx + m – 3 = 0
13. x 2 − 4 x − 12 ≤ 0
denkleminin her iki kökü de pozitif olduğuna göre, x−3
aşağıdakilerden hangisi doğrudur?
eşitsizliğinin çözüm kümesi aşağıdakilerden hangisidir?
A) m < 0 B) 0 < m < 3 C) –1 < m < 2
A) (–∞, –2] ∪ {6} B) [–2, 3] C) (3, 6]
D) m > 0 E) m > 3
D) [–2, 6] E) [6, ∞)
c b
∆ > 0, > 0, – < 0 olmalıdır. x –2 3 6
a a
(x + 2)(x – 6) + 0 – – 0 +
∆ = 4m2 – 4(m – 3) ⇒ ∆ = 4(m2 – m + 3) > 0 dır.
x–3 – – 0 + +
m 0 3
c Bölüm + 0 Tanımsız 0 +
=m–3 – – 0 +
a
Ç = (–∞, 2] ∪ {6}
b
– = 2m – 0 + + (Cevap A)
a
m > 3 bulunur.
(Cevap E)

10. | x − 3 | +2x 14. |x2 – 2x – 15| = –x2 + 2x + 15


≤0
x +1
eşitliğini sağlayan x tam sayılarının toplamı kaçtır?
eşitsizliğinin çözüm kümesi aşağıdakilerden hangisidir?
A) 8 B) 9 C) 10 D) 12 E) 15
A) (–∞, –3] B) (–∞, –1) C) [–3, –1)
x2 – 2x – 15 ≤ 0 ⇒ (x + 3)(x – 5) ≤ 0
D) (–1, 1] E) [–3, 1] – {–1} ⇒ –3 ≤ x ≤ 5
–x + 3 + 2x x+3 Eşitliği sağlayan x tam sayılarının toplamı: – 3 – 2 – 1 + 0 + 1 + 2 + 3 + 4 + 5 = 9
x < 3 için ≤ 0 ⇒ ≤0
x+1 x+1 (Cevap B)
⇒ –3 ≤ x < 1
x – 3 + 2x 3x – 3
Çözüm Yayınları

x ≥ 3 için ≤ 0 ⇒ ≤0
x+1 x+1
⇒ –1 < x ≤ 1 olamaz (x ≥ 3)
Ç = [–3, 1)
(Cevap C)
15. ∀x ∈ R için,

11. ∀x ∈ R için, x 2 − (m + 2)x + m + 2


2
<0
x2 + (m – 2)x + 4 > 0 − x + 3 x − 5

olduğuna göre, aşağıdakilerden hangisi doğrudur? olduğuna göre, m nin alabileceği gerçek sayı değerlerinin
kümesi aşağıdakilerden hangisidir?
A) –4 < m < 2 B) –6 < m < 0 C) –3 < m < 2
A) (–3, 1) B) (–2, 2) C) (–1, 3)
D) –3 < m < 0 E) –2 < m < 6
∆ < 0 ⇒ (m – 2)2 – 4 . 1 . 4 < 0
D) (–4, 0) E) (0, 4)
–x2 + 3x – 5 = 0 denklemi için ∆ < 0 olduğundan –x2 + 3x – 5 < 0 dır.
⇒ m2 – 4m – 12 < 0
x2 – (m + 2)x + m + 2 > 0 olmalıdır.
⇒ (m + 2)(m – 6) < 0
∆ < 0 ⇒ (m + 2)2 – 4(m + 2) < 0
⇒ –2 < m < 6
⇒ (m + 2)(m – 2) < 0
(Cevap E)
⇒ –2 < m < 2
(Cevap B)

12. x2 + (m – 1)x ≤ 0
108
eşitsizliğinin çözüm kümesi [0, 4] aralığı olduğuna göre,

x2 – mx – 4 < 0
eşitsizliğinin çözüm kümesi aşağıdakilerden hangisidir? 16. f(x) = x2 – (m – 2)x – 6
A) (–4, 1) B) (–3, 0) C) (–2, 3) parabolü x eksenini (1, 3) aralığında sadece bir noktada
kestiğine göre, m nin alabileceği değerlerin kümesi
D) (0, 3) E) (1, 4)
aşağıdakilerden hangisidir?
x2 + (m – 1)x = 0 denkleminin kökleri
x1 = 0 ve x2 = 4 tür. A) (–3, 1) B) (–2, 3) C) (–3, 3)
–(m – 1) = x1 + x2 ⇒ –(m – 1) = 4
D) (1, 3) E) (2, 3)
⇒ m = –3 bulunur. f(1) . f(3) < 0 ⇒ [1 – (m – 2) – 6][9 – 3(m – 2) – 6]
x2 – mx – 4 < 0 ⇒ x2 + 3x – 4 < 0 ⇒ (1 – m + 2 – 6)(9 – 3m + 6 – 6)
⇒ (x + 4)(x – 1) < 0 ⇒ (–m –3)(–3m + 9)
⇒ –4 < x < 1 x –3 3
f(1) . f(3) + 0 – 0 +
Ç = (–4, 1) (Cevap A)
–3 < m < 3 olmalı (Cevap C)
Denklem ve Eşitsizlik Sistemleri BÖLÜM 04 Test 19
1. x + 2y = 3 −x 3 ≤ 0
5.
x2 – 4y2 = 21 x+2

x2 − 4 < 0
olduğuna göre, x + y toplamı kaçtır?
eşitsizlik sisteminin çözüm kümesi aşağıdakilerden
A) 3 B) 4 C) 5 D) 6 E) 7
2 2
–x – 4y = 21 ⇒ (x – 2y)(x + 2y) = 21 hangisidir?
⇒ (x – 2y) . 3 = 21 A) (–2, 0) B) [0, 2) C) (–2, 2)
⇒ x – 2y = 7
x + 2y = 3
⇒ x = 5 ve y = –1 bulunur.
D) (–∞, –2) E) (2, ∞)
x – 2y = 7
x + y = 4 tür. x –2 0 2
–x3
(Cevap B) – + 0 – –
x+2
2. x – y = 10
x2 – 4 + 0 – – +
x2 + y2 + 4x + 6y = 40
denklem sisteminin çözüm kümesi aşağıdakilerden Ç = [0, 2)
hangisidir? (Cevap B)

A) {(4, –6), (1, –9)} B) {(5, –5), (0, –10)}

C) {(6, –4), (2, –8)} D) {(3, –7), (9, –1)}

E) {(7, –3), (8, –2)}


x – y = 10 ⇒ x = y + 10
(y + 10)2 + y2 + 4(y + 10) + 6y = 40 ⇒ y2 + 15y + 50 = 0 6. |x + y – 2| + |x3 + y3 – 26| = 0
⇒ (y + 5)(y + 10) = 0
olduğuna göre, x · y çarpımı kaçtır?
⇒ y = –5 veya y = –10
y = –5 için x = 5 ve A) –6 B) –3 C) 0 D) 3 E) 6
y = –10 için x = 0 bulunur. x + y – 2 = 0 ⇒x+y=2
Ç = {(5, –5), (0, –10)} (Cevap B) x3 + y3 – 26 = 0 ⇒ (x + y)(x2 – xy + y2) = 26
⇒ x2 – xy + y2 = 13
Çözüm Yayınları

3. 5x – y = 1
x2 + 2xy + y2 = 4
x · y = 6 x2 – xy + y2 = 13

––––––––––– –
–––––
denklem sisteminin çözüm kümesi aşağıdakilerden
3xy = –9 ⇒ xy = –3
hangisidir?
(Cevap B)
 6     3 
A)  , 5  , ( −1, − 6) B) ( −2, − 3),  − , − 4  
 5     2 

 7. x2 – 4x – 5 ≤ 0
 3   3 
C) (3, 2),  , 8   D) ( −3, 2),  , − 4  
  4   2  x2 – 5x ≤ 0

  3  eşitsizlik sisteminin çözüm kümesi aşağıdakilerden


E) (1, 6),  − , 4  
  2  hangisidir?
5x – y = 1 ⇒ y = 5x – 1
x . y = 6 ⇒ x(5x – 1) = 6 A) [–1, 5] B) [–1, 0] C) [0, 5]
⇒ 5x2 – x – 6 = 0
D) [–1, 4] E) [4, 4]
⇒ x = 6 veya x = –1
5 x –1 0 5
x = 6 için y = 5 ve x = –1 için y = –1 için y = –6 bulunur.
5 x2 – 4x – 5 + 0 – – 0 +
Ç = {( 6 , 5), (–1, –6)}
x2 – 5x + + 0 – 0 +
5
(Cevap A)
4. 3x2 – y2 = 2 Ç = [0, 5]
x2 – 2y = –1 (Cevap C) 109
denklem sisteminin çözüm kümesi aşağıdakilerden
hangisidir?
A) {(–1, 1), (1, 1), (–3, 5), (3, 5)} 8. |x + 3| ≤ |x – 5|
B) {(–1, –1), (–1, 1), (–3, 5), (–3, –5)} |x + 3| = x + 3
C) {(1, 1), (–3, 3), (–3, 5)} sistemini sağlayan x tam sayılarının toplamı kaçtır?
D) {(–1, –1), (–1, 3), (1, –3)}
A) –7 B) –5 C) –3 D) 3 E) 5
E) {(–1, 1), (–3, 5), (1, 5)}
x2 – 2y = –1 ⇒ x2 = 2y – 1 (x + 3)2 ≤ (x – 5)2 ⇒ x2 + 6x + 9 ≤ x2 – 10x + 25
2 2 2
3x – y = 2 ⇒ 3(2y – 1) – y = 2 ⇒ 16x ≤ 16
⇒ 6y – 3 – y2 = 2 ⇒x≤1
⇒ y2 – 6y + 5 = 0 |x + 3| = x + 3 ⇒ x + 3 ≥ 0
⇒ y = 1 veya y = 5
⇒ x ≥ –3
y = 1 için x2 = 1 ⇒ x = –1 veya x = 1
–3 < x < 1 bulunur. Sistemi sağlayan x tam sayılarının toplamı –3 – 2 – 1 + 0 + 1 = –5 tir.
y = 5 için x2 = 9 ⇒ x = –3 veya x = 3 bulunur.
(Cevap B)
Ç = {(–1, 1), (1, 1), (–3, 5), (3, 5)} (Cevap A)
Test 19 1. B 2. B 3. A 4. A 5. B 6. B 7. C 8. B 9. D 10. B 11. A 12. C 13. A 14. D 15. D 16. C

9. x2 – 3x < 0 13. (m – 5)x2 – 4mx + m + 2 = 0


(2x + 5) (x – 2) ≥ 0 denkleminin kökleri x1 ve x2 dir.

eşitsizlik sisteminin çözüm kümesi aşağıdakilerden x1 < 0 < x2


hangisidir? |x1| < x2

5 5
A)  −∞, −  B)  − , 0  C) (0, 2] olduğuna göre, aşağıdakilerden hangisi doğrudur?
 2  2 
A) –2 < m < 0 B) –2 < m < 5 C) m < 5
D) [2, 3) E) (3, ∞)
D) 0 < m < 5 E) m > 5
x –5 c m+2
2 0 2 3 <0⇒ < 0 ⇒ –2 < m < 5
a m–5
x(x – 3) + + – – 0 +
b 4m
(2x + 5)(x – 2) + 0 – – 0 + + – >0⇒ > 0 ⇒ m < 0 veya m > 5
a m–5
Ç = [2, 3) –2 < m < 0 bulunur.

(Cevap D) (Cevap A)

14. x2 + 1 > 0
x +1

10. x2 – (2m – 8)x + n – 3 = 0 x−3 ≤ 0
x+3
denklemimin kökleri x1 ve x2 dir.
eşitsizlik sisteminin çözüm kümesi aşağıdakilerden
|x1| = |x2| hangisidir?

x1 · x2 < 0 A) (–∞, –3) B) (–3, –1) C) (–3, 1)

olduğuna göre, m + n toplamının alabileceği en büyük D) (–1, 3] E) [3, ∞)


tam sayı değeri kaçtır? x –3 –1 3
l – – + +
A) 5 B) 6 C) 7 D) 8 E) 9
ll + – – 0 +
x1 ve x2 ters işaretli olduğundan
Çözüm Yayınları

x1 + x2 = 0 ⇒ 2m – 8 = 0 Ç = (–1, 3]
⇒m=4 (Cevap D)
x1 . x2 < 0 ⇒ n – 3 < 0
⇒ n < 3 (n nin alabileceği en büyük değer 2 olur.) 15. (m – 1)x2 + 2(m + 1)x + m – 3 = 0
m + n = 6 bulunur. denkleminin birbirinden farklı iki pozitif kökü olduğuna
(Cevap B) göre, m nin alabileceği değerlerin kümesi aşağıdakilerden
hangisidir?
11. x2 ≤ x + 6  1
A) (–∞, –1) B)  −1,  C) (1, 3)
x2 > 6x – 5  3

eşitsizlik sisteminin çözüm kümesi aşağıdakilerden 1 


D)  , 1 E) (3, ∞)
hangisidir? 3 
c b 1
∆ > 0, > 0 ve – > 0 eşitsizliklerini birlikte sağlayan bölge ( , 1) aralığıdır.
A) [–2, 1) B) [–2, 5) C) [–2, 3] a a 3
(Cevap D)
D) (1, 3] E) (1, 5) 16. y
x –2 1 3 5 y = f(x)
x2 – x – 6 + 0 – – 0 + +
2
x2 – 6x + 5 + + 0 – – 0 +

Ç = [–2, 1) x
110 –1 0 3 5
(Cevap A)

y = g(x)
12. x2 + 4x – 5 ≤ 0
Yukarıdaki şekilde y = f(x) ve y = g(x) fonksiyonlarının
9 – x2 ≥ 0 grafikleri verilmiştir.
eşitsizlik sisteminin çözüm kümesi aşağıdakilerden Buna göre,
hangisidir? f(x) · g(x) ≥ 0
eşitsizliğini sağlayan kaç tane x tam sayısı vardır?
A) (–∞, –5] B) [–5, –3] C) [–3, 1]
A) 3 B) 4 C) 5 D) 6 E) 7
D) [1, 3] E) [3, ∞) x –1 0 3 5
x –5 –3 1 3 f(x) + + 0 – – 0 +
(x + 5)(x – 1) + 0 – – 0 + + g(x) – + + – –
0 0
(x – 3)(3 + x) – – 0 + + 0 – f(x).g(x) – 0 + 0 – 0 + 0 –

x → –1, 0, 3, 4, 5 (5 tane) (Cevap C)


(Cevap C)
Denklem ve Eşitsizlik Sistemleri BÖLÜM 04 Test 20
1. x2 + y2 = 50 5. x2 ≤ x + 6
x–y=0 x2 ≤ 6x – 5
denklem sisteminin çözüm kümesi aşağıdakilerden eşitsizlik sisteminin çözüm kümesi aşağıdakilerden
hangisidir? hangisidir?
A) {(–5, 5), (5, –5)} B) {(–3, 3), (3, 3)} A) [–3, 1] B) [–1, 3] C) [1, 3]
C) {(–4, –4), (4, 4)} D) {(–5, –5), (5, 5)} D) [1, 5] E) [2, 6]
E) {(–6, –6), (6, 6)} x –2 1 3 5
x–y=0⇒x=y x2 – x – 6 + 0 – – 0 + +
x2 + y2 = 50 ⇒ x2 + x2 = 50
x2 – 6x + 5 + + 0 – – 0 +
⇒ 2x2 = 50
⇒ x = –5 veya x = 5 Ç = {1, 3}
x = –5 için y = –5 ve x = 5 için y = 5 bulunur. (Cevap C)
(Cevap D)

2. x2 + y2 = 52
3x – 2y = 0 6. |x + 2| – 2 = x

denklem sisteminin çözüm kümesi aşağıdakilerden |x – 5| + x = 5


hangisidir? sistemini sağlayan x tam sayılarının toplamı kaçtır?
A) {(–2, –3), (2, 3)} B) {(–2, 3), (2, –3)}
A) 11 B) 12 C) 13 D) 14 E) 15
C) {(3, –2), (–3, 2)} D) {(–4, 6), (4, –6)} |x + 2| = x + 2 ⇒ x + 2 ≥ 0 ⇒ x ≥ –2
|x – 5| = –x + 5 ⇒ x – 5 ≤ 0 ⇒ x ≤ 5
E) {(–4, –6), (4, 6)}
3x –2 ≤ x ≤ 5
3x – 2y = 0 ⇒ y =
2 –2 – 1 + 0 + 1 + 2 + 3 + 4 + 5 = 12
3x
x2 + y2 = 52 ⇒ x2 + ( )2 = 52 (Cevap B)
Çözüm Yayınları

2
9x2
⇒ x2 + = 52
4
⇒ 13x2 = 52 . 4
⇒ x = 4 veya x = –4
x = 4 için y = 6 ve x = –4 için y = –6 Ç = {(–4, –6), (4, 6)}
(Cevap E) 7. 1 >0
x+3

x ≤0
2 2
3. x – y = 5 x−2
x2 + y2 = 13 eşitsizlik sisteminin çözüm kümesi aşağıdakilerden
hangisidir?
olduğuna göre, aşağıdakilerden hangisi x + y toplamı
olamaz? A) (–∞, –3) B) (–3, 0] C) [0, 2)

A) –5 B) –3 C) –1 D) 1 E) 5 D) (–3, 2) E) (2, ∞)
x2 – y 2 x –3 0 2
⇒ 2x2 = 18 1
x2 + y 2
⇒ x2 = 9 – 0 + + +
x+3
⇒ x = –3 veya x = 3 x
+ + 0 – 0 +
x2 + y2 = 13 ⇒ 9 + y2 = 13 x–2
⇒ y = –2 veya y = 2
x + y = –3 olamaz. Ç = [0, 2)
(Cevap B) (Cevap C) 111
4. 4x2 + y2 = 100
4x – y2 = –20
8. x2 – y = 1
denklem sisteminin çözüm kümesi aşağıdakilerden
4x + y = –5
hangisidir?
denklem sisteminin çözüm kümesi aşağıdakilerden
A) {(–5, 0), (4, –6), (4, 6)} B) {(–5, 1), (–4, 6), (4, 6)}
hangisidir?
C) {(0, 5), (–6, 4), (6, 4)} D) {(0, 4), (2, –6), (2, 6)}
A) {(–3, 2)} B) {(–2, 3)} C) {(2, 3)}
E) {(–3, 0), (4, –2), (4, 2)}
4x2 + 4x = 80 ⇒ x2 + x – 20 = 0 D) {(–2, 3), (2, 3)} E) {(2, –3), (2, 3)}
⇒ x = –5 veya x = 4 x2 + 4x + 4 = 0 ⇒ (x + 2)2 = 0
x = –5 içi y2 = 0 ⇒ y = 0 ⇒ x = –2
x = 4 için 4 . 16 + y2 = 100 ⇒ y2 = 36 x = –2 için 4 . (–2) + y = –5 ⇒ y = 3 bulunur.
⇒ y = –6 veya y = 6 Ç = {(–2, 3)}
(Cevap A) (Cevap B)
Test 20 1. D 2. E 3. B 4. A 5. C 6. B 7. C 8. B 9. D 10. C 11. C 12. C 13. C 14. C 15. B 16. D

9. x −1 ≤ 0 13. x2 – 3x – 4 < 0
x−4
–x2 + 7x – 10 ≥ 0
x 2 − 2x ≥ 0
eşitsizlik sisteminin çözüm kümesi aşağıdakilerden
eşitsizlik sisteminin çözüm kümesi aşağıdakilerden hangisidir?
hangisidir?
A) (–∞, –1) B) (–1, 2] C) [2, 4)
A) (–∞, 0] B) [0, 2] C) [0, 4)
D) (4, 5] E) [5, ∞)
D) [2, 4) E) (4, ∞) x –1 2 4 5
x 0 1 2 4 x2 – 3x – 4 + 0 – – 0 + +
x–1
+ 0 + – – + –x2 + 7x – 10 – – 0 + + 0 –
x–4
2 – 2x
x + – 0 – 0 + + Ç = [2, 4)
(Cevap C)
Ç = [2, 4)
(Cevap D)

10. 3x2 – y2 = –22


x2 – 9 = 0
denklem sisteminin çözüm kümesi aşağıdakilerden 14. x 2 − 81 ≤ 0
hangisidir? x2 − 9
A) {(–3, –1), (–3, 1), (3, –1), (3, 1)} eşitsizliğini sağlayan kaç farklı x tam sayısı vardır?
B) {(–3, –2), (–3, 2), (3, –2), (3, 2)} A) 8 B) 10 C) 12 D) 14 E) 16
C) {(–3, 7), (–3, –7), (3, 7), (3, –7)} x –9 –3 3 9
x2 – 81 + 0 – – – 0 +
D) {(–3, 5), (–3, –5), (3, –5), (3, 5)}
x2 – 9 + + 0 0 + +
E) {(–3, –4), (–3, 4), (3, –4), (3, 4)}
Bölüm + – – +
x2 = 9 ⇒ x = –3 veya x = 3 Tanımsız

x = –3 içi 3 . (–3)2 – y2 = –22 ⇒ y2 = 49 Eşitsizliği sağlayan (– 9 – 8 – 7 – 6 – 5 – 4) ve (4, 5, 6, 7, 8, 9) olmak üzere 12 tane tam
Çözüm Yayınları

⇒ y = –7 veya y = 7 sayı vardır.

x = 3 için 3 . (3)2 – y2 = –22 ⇒ y2 = 49 (Cevap C)

⇒ y = –7 veya x = 7
(Cevap C)

11. x−3 < 0


x−5 15. x−7 < 0
x+4
x 2 − 6x + 8 ≤ 0
( x − 3)( x 2 − 9)
>0
eşitsizlik sisteminin çözüm kümesi aşağıdakilerden 5−x
hangisidir?
eşitsizlik sisteminin çözüm aralığında kaç tam sayı
A) (–∞, 2] B) [2, 3) C) (3, 4] vardır?
D) [4, 5) E) (5, ∞) A) 5 B) 6 C) 7 D) 8 E) 9
x 2 3 4 5 x –4 –3 3 5 7
x–3 x–7
+ + 0 – – 0 + + – – – – 0 +
x–5 x+4
(x – 2)(x – 4) + 0 – – 0 + + (x – 3)2(x + 3)
5–x – – 0 + 0 + – –

Ç = [3, 4]
(Cevap C) Çözüm aralığında –2, –1, 0, 1, 2, 4 olmak üzere 6 tam sayı vardır.
112 12. (m – 1)x2 – (m – 2)x + m = 0 (Cevap B)
denkleminin kökleri x1 ve x2 dir.
x1 < 0 < x2
|x1| < |x2|
16. x2 – 2(m + 2)x – m + 4 = 0
olduğuna göre, aşağıdakilerden hangisi doğrudur?
A) m < –1 B) –1 < m < 0 C) 0 < m < 1 denkleminin pozitif iki kökü olduğuna göre, m için
aşağıdakilerden hangisi doğrudur?
D) 1 < m < 2 E) m > 1
c b A) m < –5 B) –5 < m < –2 C) –2 < m < 0
< 0 ve – > 0 olmalıdır.
a a
m 0 1 2
D) 0 < m < 4 E) m > 4
m c b
+ 0 – + + ∆ > 0, > 0, – > 0 olmalıdır. Çözüm yapıldığında 0 < m < 4 bulunur.
m–1 a a
m–2 (Cevap D)
m–1 + + – 0 +

0 < m < 1 bulunur. (Cevap C)


BİRE BİR BÖLÜM 04 Test 21
1. a bir gerçek sayı olmak üzere, 5. P(x) = x2 – 4x + m
ax2 – 16x + 16 = 0 Q(x) = x2 + 3x + n

denkleminin köklerinden biri diğerinin 3 katıdır. polinomları veriliyor.

Buna göre, a kaçtır? Bu iki polinom ortak bir köke sahip ve P(x) polinomunun
16
x2 = 3x1 ⇒ x1 + x2 = kökleri eşit olduğuna göre, m + n toplamı kaçtır?
a
16 A) –6 B) –5 C) –4 D) 2 E) 5
x1 + 3x1 =
a ∆ = 16 – 4 . l . m = 0 ⇒ m = 4
16 4
4x1 = ⇒ x1 = P(x) = 0 ⇒ (x – 2)2 = 0 ⇒ x = 2
a a
4 4 16 64 Q(2) = 0 ⇒ 4 + 6 + n = 0
a( )2 – 16( ) + 18 = 0 ⇒ a . – + 16 = 0
a a a2 a ⇒ n = –10
16 64 m + n = 4 – 10 = –6
⇒ – + 16 = 0
a a (Cevap A)
48
⇒ 16 =
a
⇒a=3 (Cevap 3)

2. b ve c sıfırdan farklı gerçek sayılar olmak üzere,


6. (3x – 2) (x + 2) + (3x – 2) (x – 3) = 0
x2 + 2bx + 3c = 0
eşitliğini sağlayan x gerçek sayılarının toplamı kaçtır?
denkleminin kökleri b ve c dir.
7 4 3 5
A) 1 B) C) D) E)
Buna göre, b + c toplamı kaçtır? 6 3 2 3
(3x – 2)[x + 2 + x – 3] = 0 ⇒ (3x – 2)(2x – 1) = 0
A) –12 B) –9 C) –6 D) 3 E) 6 2 1
⇒x= Vx=
b . c = 3c ⇒ b = 3 3 2
2 1 7
b + c = –2b ⇒ 3 + c = –6 Toplamları + = dır.
3 2 6
⇒ c = –9 (Cevap B)
b + c = 3 – 9 = –6
Çözüm Yayınları

(Cevap C)

7. x2 – 4x – 2 = 0
3. k pozitif bir gerçek sayı olmak üzere, denkleminin kökleri p ve q dur.
2
2x – 5x – k = 0
Buna göre, aşağıdaki denklemlerden hangisinin kökleri
denkleminin bir kökü k olduğuna göre, diğer kökü kaçtır? 1 ve 1
dur?
1 1 1 1 p q
A) –1 B) − C) − D) − E) −
2 3 6 8 A) 2x2 – 4x – 1 = 0 B) 2x2 + 4x + 1 = 0
2k2 – 5k – k = 0 ⇒ 2k2 – 6k = 0
⇒ 2k(k – 3) = 0 C) x2 + 2x – 4 = 0 D) x2 – 2x – 4 = 0
⇒k=0vk=3 E) 2x2 + 4x – 1 = 0
2x2 – 5x – 3 = 0 ⇒ (x – 3)(2x + 1) = 0 1 1
1 x= ⇒p=
p x
⇒x=–
2 1 1
(Cevap B) ( )2 – 4 . – 2 = 0 ⇒ 1 – 4x – 2x2 = 0
x x
⇒ 2x2 + 4x – 1 = 0
(Cevap E)
113
4. k bir pozitif gerçek sayı olmak üzere,
4x2 – 6kx – 7 = 0

denkleminin kökleri farkı 4 olduğuna göre, k kaçtır?


4 3
A) 1 B) 2 C) 4 D) 6 E) 9 8. 2 + − 2 = 0
x x
ñ∆ 36k2 – 4 . 4 . (–7)
= =4 denklemini sağlayan x gerçek sayılarının toplamı
|a| 4
kaçtır?
36k2 + 16 . 7 = 16
A) –4 B) –3 C) –2 D) 2 E) 3
36k2 = 162 – 16 . 7
2x2 + 4x – 3 = 0
36k2 = 16(16 – 7)
4
36k2 = 16 . 9 x1 + x2 = – = –2
2
4k2 = 16 ⇒ k2 = 4 (Cevap C)
⇒k=2 (Cevap B)
Test 21 1. (3) 2. C 3. B 4. B 5. A 6. B 7. E 8. C 9. (10) 10. A 11. C 12. C 13. C 14. A 15. (5) 16. E

9. x2 – ax + 9 = 0 1 + 6 +9 = 0
13.
denkleminin kökleri x1 ve x2 dir. x2 x
olduğuna göre, x kaçtır?
1 + x =4
2
x1 1 1 1
A) − B) − C) − D) 3 E) 6
9 6 3
olduğuna göre, a kaçtır?
1 + 6x + 9x2 = 0
1 + óx1x2 = 4òx1 (1 + 3x)2 = 0
1 + 3 = 4òx1 1
3xH = 0 ⇒ x = –
3
òx1 = 1 (Cevap C)
x1 = 1
1 – a + 9 = 0 ⇒ a = 10
(Cevap 10)

10. (x – 1) (x + 1) (x – 3) = (x – 2) (x + 2) (x – 5)
denklemiyle aşağıdaki denklemlerden hangisinin çözüm 14. (m + 1)x2 + 4x + m – 4 = 0
kümesi aynıdır?
denkleminin ters işaretli iki gerçek kökü olduğuna göre,
A) 2x2 + 3x – 17 = 0 B) x2 + 3x – 15 = 0 m nin alabileceği tam sayı değerlerinin toplamı kaçtır?
C) 2x2 + 3x – 11 = 0 D) 2x2 – 3x – 9 = 0 A) 6 B) 7 C) 8 D) 9 E) 10
c m–4 m –1 4
E) 2x2 – 3x – 7 = 0 = < 0
c
a m+1 0 0
(x2 – 1)(x – 3) = (x2 – 4)(x – 5) + – +
a
x3 – 3x2 – x + 3 = x3 – 5x2 – 4x + 20
m nin alabileceği tam sayı değerlerinin toplamı 0 + 1 + 2 + 3 = 6 dır.
2x2 + 3x – 17 = 0
(Cevap A)
(Cevap A)
Çözüm Yayınları

11. (m – 2)x2 + 2x + m2 – 9 = 0
15. x pozitif bir gerçek sayı olmak üzere,
denkleminin kökleri x1 ve x2 dir.
 2 1   x  x2 + 1
x1 < 0 < x2 x − 2  =
 x   3x + 9  x
olduğuna göre, m nin alabileceği değerler kümesi
olduğuna göre, x kaçtır?
aşağıdakilerden hangisidir?
x4 – 1 x x2 + 1
. =
A) (–∞, 2) B) (–3, 3) x2 3x + 9 x

C) (–∞, –3) ∪ (2, 3) D) (2, ∞) x2 – 1


= 1 ⇒ x2 – 1 = 3x + 9
3x + 9
E) (–3, 2) ∪ (3, ∞) ⇒ x2 – 3x – 10 = 0
⇒ (x – 5)(x + 2) = 0
c m2 – 9 x –3 2 3
= < 0 c
a m–2 ⇒x=5
– 0 + – 0 +
a (Cevap 5)
Ç = (–∞, –3) ∪ (2, 3)
114 (Cevap C)
16. Her x gerçek sayısı için,
2x2 + ax + 12 = (x + 4) (bx + c)

olduğuna göre, a + b + c toplamı kaçtır?

A) 12 B) 13 C) 14 D) 15 E) 16
12. x pozitif bir gerçek sayı ve
2x2 + ax + 12 = bx2 + cx + 4bx + 4c
x4 – 7x2 = 18
= bx2 + (c + 4b)x + 4c
olduğuna göre, x kaçtır? b = 2, 4c = 12 ⇒ c = 3
a = c + 4b ⇒ a = 3 + 8 = 11
A) 1 B) 2 C) 3 D) 4 E) 5
a + b + c = 16 bulunur.
t2 – 7t – 18 = 0 (Cevap E)
(t + 2)(t – 9) = 0
x2 = 9 ⇒ x = 3 (x > 0)
(Cevap C)
BİRE BİR BÖLÜM 04 Test 22
1. Analitik düzlemde y = 3x2 parabolü ile y = k doğrusunun 5. b ve c gerçek sayılar olmak üzere,
kesiştiği iki nokta arasındaki uzaklık 4 birim olduğuna
P(x) = x2 + bx + c
göre, k kaçtır?
polinomunun bir kökü 2 – 4i karmaşık sayısıdır.
A) 6 B) 9 C) 12 D) 15 E) 18
Buna göre, P(–1) kaçtır?
y y = 3x2
y=k A) 15 B) 16 C) 20 D) 25 E) 30
x1 = 2 – 4i
x2 = 2 + 4i

–2 0 2 x1 + x2 = 4, x1 . x2 = 20
p(x) = x2 – 4x + 20
x = 2 için y = 3 . 22 = 12 p(–1) = 1 + 4 + 20 = 25
(Cevap C) (Cevap D)

2. y = x2 – 2(a – 1)x + a2 6. a, b ∈ R ve i2 = –1 olmak üzere,

parabolü y = 2 doğrusuna teğet olduğuna göre, a kaçtır? 4 − 2i = a + bi


1 − i
3 3
A) –2 B) − C) 1 D) E) 2
2 2 olduğuna göre, a + b toplamı kaçtır?
x2 – 2(a – 1)x + a2 = 2
A) 3 B) 4 C) 5 D) 6 E) 7
x2 – 2(a – 1)x + a2 – 2 = 0
(4 – 2i)(1 + i) 4 + 4i – 2i – 2i2
4(a – 1)2 – 4(a2 – 2) = 0 =
2 2
a2 – 2a + 1 – a2 + 2 = 0
6 + 2i
–2a = –3 =
3 2
a= = 3 + i = a + bi
2
Çözüm Yayınları

(Cevap D) a + b = 3 + 1 = 4 tür.
(Cevap B)

7. z = a + bi (b ≠ 0)
3. i2 = –1 olmak üzere,
w = c + di
2
(1 − i) ⋅  2 + 2 4
karmaşık sayıları için z + w toplamı ve z · w çarpımı birer
 i
gerçek sayı olduğuna göre,
işleminin sonucu aşağıdakilerden hangisidir?
I. z2 + w2 gerçek sayıdır.
A) –32i B) –16i C) 16 D) 32 E) 32i
II. z – w gerçek sayıdır.
1 z
[(1 – i)2]2 . [2(1 +
)]2 = (–2i)2 . 22 . (1 – i)2 III.
gerçek sayıdır.
i w
= 4i2 . 4 . (–2i)
= –16 . (–2i)
ifadelerinden hangileri doğrudur?
= 32i A) Yalnız I B) I ve II C) I ve III
(Cevap E)
D) II ve III E) I, II ve III
z + w gerçek sayı olduğundan z2+ w2 sayısı gerçek sayı olur. Yalnız l doğrudur.
(Cevap A)
115
4. z karmaşık sayısının eşleniği z ile gösterildiğine göre,
z = 2 – 3i
z
karmaşık sayısı için ifadesi aşağıdakilerden 8. a ve b gerçek sayı ve i2 = –1 olmak üzere,
z+1
hangisine eşittir?
(1 – 2i) (2 – i) (3 – i) (2 + i) (3 + i) = a + bi
1 5 1 5 1 5
A) − − i B) − + i C) − i olduğuna göre, a + b toplamı kaçtır?
6 6 6 6 6 6
D) 3 - 2i E) 3 + 2i A) –100 B) –50 C) 0 D) 50 E) 100
2 – 3i 2 – 3i (2 – 3i)(3 – 3i) (1 – 2i)(2 – i)(2 + i)(3 – i)(3 + i) = a + bi
= =
2 + 3i + 1 3 + 3i 18 (1 – 2i)(5)(10) = a + bi ⇒ 50 – 100i = a + bi
3 – 3i a = 50, b = –100 ve a + b = –50 bulunur.
6 – 6i – 9i + 9i2 –3 – 15i 1 5
= = =– – i (Cevap B)
18 18 6 6
(Cevap A)
Test 22 1. C 2. D 3. E 4. A 5. D 6. B 7. A 8. B 9. C 10. A 11. B 12. (4) 13. (5) 14. B 15. B 16. (90)

9. y 13. 2a – 3b + 5c = 11
y = f(x)
a – 6b + 10c = 7

olduğuna göre, a kaçtır?


2 –4a + 4b – 10c = –22
a – 6b + 10c = 7

–––––––––––––– ––––––
x –3a = –15
2
a = 5 bulunur.
(Cevap 5)
y = f(x) fonksiyonunun grafiği, şekildeki gibi x eksenine
(2, 0) noktasında teğet olan ve (0, 2) noktasından geçen
paraboldür.

Buna göre, f(–2) kaçtır?

A) 4 B) 6 C) 8 D) 10 E) 12
y = a(x – 2)2, x = 0 için y = 2
1
a(–2)2 = 2 ⇒ a =
2
1 2 1 16
f(–2) = (–2 – 2) = (–4)2 = = 8 bulunur. 14. 2a + b = –1
2 2 2
(Cevap C) 2b + c = 10
10. (2x – 3) (4x2 – 9) < 0
2c + a = 6
eşitsizliğinin çözüm kümesi aşağıdaki açık aralıklardan
hangisidir? olduğuna göre, a kaçtır?
3  − 3 , 0  3 3
A)  −∞, −  B)   C)  − ,  A) –3 B) –2 C) –1 D) 1 E) 2
 2  2   2 2 3(a + b + c) = 15 ⇒ a + b + c = 5
a+b–c = 5 a+b+c = 5
3 9 3
D)  − ,  E)  , ∞  2a + b = –1 2a + c = 10
 2 4 2  –
––––––––– –––––
Çözüm Yayınları


––––––––– –––––
(2x – 3)2(2x + 3) < 0 c–a=6 a–b = –5
3 3 c = a + 6 b = a + 5
– a + b + c = 5 ⇒ a + (a + 5) + (a + b) = 5
x 2 2
⇒ a = –2 bulunur.
– 0 + 0 +
(Cevap B)
(Cevap A)

x y 15. x(2 – x) > 0


11. − + z = 1
2 2 ⋅ 3 ⋅ 5 2 ⋅ 3 2 ⋅ 5 2 2 ⋅ 3 2 30 (x + 6) (x – 3) < 0
olduğuna göre, 3x – 2y + 5z ifadesinin değeri kaçtır?
eşitsizlik sisteminin çözüm kümesi (a, b) açık aralığı
A) 4 B) 6 C) 8 D) 9 E) 12 olduğuna göre, a – b farkı kaçtır?
x y z 1
– + = = 3x – 2y + 5z = 6 A) –3 B) –2 C) –1 D) 1 E) 2
22 . 3. 5 2 . 32. 5 22 . 32 2.3.5
x –6 0 2 3
(3) (2) (5) (6) (Cevap B)
x(2 – x) – – 0 + 0 – –
)x + 6)(x – 3) + 0 – – – 0 +

(a, b) = (0, 2) ⇒ a – b = 0 – 2 = –2
(Cevap B)

116 12. x, y ve z gerçek sayıları için


x·y=6

x · z = 14
16. y = x2 parabolü ile y = –2x + 3 doğrusu arasında kalan
2x + 3y + z = 16
sınırlı bölgenin sınırları üzerindeki (x, y) noktaları için
olduğuna göre, x kaçtır? x2 + y2 ifadesinin alabileceği en büyük değer kaçtır?
6 14 y x2 = –2x + 3
2x + 3 . + = 16
x x x2 + 2x – 3 = 0
18 14
2x + + = 16 (x + 3)(x – 1) = 0
x x
x = –3 v x = 1
2x2 + 18 + 14 = 16x
2x2 – 16x + 32 = 0 3
x = –3 için y = 9
2
x – 8x + 16 = 0 ⇒ (x – 4) = 0 ⇒ x = 4 2 x
–3 0 1 3 x2 + y2 = 9 + 81 = 90
(Cevap 4) 2 (Cevap 90)
BİRE BİR BÖLÜM 04 Test 23
1 ( x 2 − 1)( x 2 + 4)
1. f(x) = mx – 2 + 5. <0
x x2 − 4
fonksiyonu veriliyor.
eşitsizliğinin çözüm kümesi aşağıdakilerden hangisidir?
Buna göre, ∀x > 0 için f(x) ≥ 0 koşulunu sağlayan
en küçük m değeri kaçtır? A) (–∞, –2) B) (–2, 1) C) (–1, 2)
1 3 D) (–2, –1) ∪ (1, 2) E) (2, ∞)
A) B) 1 C) D) 2 E) 3
2 2
x –2 –1 1 2
mx2 – 2x + 1
≥ 0 ve x > 0 + 0 – + – 0 +
x
2
mx – 2x + 1 ≥ 0 olmalıdır Ç = (–2, –1) ∪ (1, 2)
m > 0 ve ∆ ≤ 0
(Cevap D)
4 – 4m . 1 ≤ 0 ⇒ –4m ≤ –4

⇒m≥1 (Cevap B)

2. f (x) = 3 4 − x 2
fonksiyonunun gerçek sayılarda en geniş tanım kümesi 6. 2x2 – 5x – 4 = 0
T ve görüntü kümesi G olduğuna göre T ∩ G kümesi
denkleminin kökleri x1 ve x2 dir.
aşağıdaki aralıklardan hangisidir?
Buna göre, 1 + 1 toplamı kaçtır?
A) [–2, 2] B) [0, 2] C) [2, 4] 1 − x1 1− x2
D) [0, 4] E) [0, 6] 1 1 1 1 1
T = [–2, 2] G = [0, 6] T ∩ G = [0, 2]
A) B) C) D) E)
7 6 5 4 3
(Cevap B) 1 – x2 + 1 – x 1 2 – (x1 + x2)
=
(1 – x1)(1 – x2) 1 – x2 – x1 + x1x2
3. x2 – (m + 3)x + m – 3 = 0 2– 5
2
= =1
denkleminin kökleri x1 ve x2 dir. 1– 5 – 4 7
2 2
1 + 1 >2 (Cevap A)
Çözüm Yayınları

x1 x 2

olduğuna göre, m nin alabileceği değerler kümesi
aşağıdaki aralıklardan hangisidir?
A) (–∞, 3) B) (0, 3) C) (3, 9)

D) (9, 12) E) (9, ∞) 7. Her x gerçek sayısı için,


x1 + x 2 m+3
– 2 > 0 ⇒ –2>0 x2 + 2x + a > 3
x1 . x 2 m–3
m + 3 – 2m + 6 olduğuna göre, aşağıdakilerden hangisi doğrudur?
⇒ >0
m–3
A) a < –4 B) –4 < a < –2 C) –2 < a < 0
m 3 9 –m + 9
>0
– 0 + 0 – m–3 D) 0 < a < 4 E) a > 4
x2 + 2x + a – 3 > 0
3<m<9 ∆ < 0 ⇒ 4 – 4 . 1 . (a – 3) < 0
(Cevap C) ⇒ a > 4 bulunur.
4. y (Cevap E)
T
5

1
117
x
–2 0
8. a ≠ –1 olmak üzere,
(a + 1)x2 – 2(a + 3)x + 21 = 0
Şekilde grafiği verilen parabolün tepe noktası T(–2, 5) ve y
eksenini kestiği nokta (0, 1) dir. denkleminin kökleri eşit olduğuna göre, a nın alabileceği
değerler toplamı kaçtır?
Bu parabolün denklemi y = ax2 + bx + c olduğuna göre,
∆ = 4(a + 3)2 – 4(a + 1)21 = 0
a + b + c toplamı kaçtır?
(a + 3)2 – (a + 1) . 21 = 0
A) –6 B) –5 C) –4 D) 2 E) 3 a2 + 6a + 9 – 21a – 21 = 0
y = a(x + 2)2 + 5, x = 0, y = 1 a2 – 15a – 16 = 0
a(a)2 + 5 = 1 ⇒ 4a – 4 ⇒ a = –1 a1 + a2 = 15 bulunur.
y = –(x + 2)2 + 5 ⇒ –x2 – 4x – 4 + 5 (Cevap 15)
⇒ –x2 – 4x + 1
a + b + c = –1 – 4 + 1 = –4
(Cevap C)
Test 23 1. B 2. B 3. C 4. C 5. D 6. A 7. E 8. 15 9. C 10. E 11. A 12. B 13. 20 14. A 15. 6 16. D

9. y
x 2 − 8x + 7 < 0
y = x2 + bx + c 13.
( x + 3)2
eşitsizliğini sağlayan x tam sayılarının toplamı kaçtır?
x (x – 1)(x – 7)
–1 0 A <0
(x + 3)2
x –3 1 7
–3 + + 0 – 0 +

1 < x < 7 ⇒ 2 + 3 + 4 + 5 + 6 = 20
Şekilde grafiği verilen parabolün denklemi (Cevap 20)

y = x2 + bx + c dir.

Buna göre, A noktasının apsisi kaçtır? 14. y


y = f(x)
5 7
A) 2 B) C) 3 D) E) 4 2
2 2
x= – 1 için y = 0 1 – b + c = 0 ⇒ –b + c = –1
2
x = 0 için y = –3 c = –3 –b – 3 = –1 ⇒ b = –2 x
0 4
x2 – 2x – 3 = 0 ⇒ (x + 1)(x –3) = 0
A noktasının apsisi 3 tür.
(Cevap C) d
–4

Yukarıdaki şekilde y = f(x) parabolü ve d doğrusu


10. x−2 + x−2 = 0 gösterilmiştir.
x−3 x−5
Buna göre, taralı bölge aşağıdaki eşitsizlik sistemlerinden
denkleminin kökleri x1 ve x2 olduğuna göre, x1 + x2
hangisinin çözüm kümesidir?
toplamı kaçtır?
A) y ≥ x2 – 4x B) y ≥ x2 – 4x C) y ≤ x2 – 4
A) –6 B) –4 C) 4 D) 5 E) 6
Çözüm Yayınları

1 1 2x – 8
y ≤ –x + 2 y ≤ x – 2 y≥x–2
(x – 2)( + ) = 0 ⇒ (x – 2) . =0
x–3 x–5 (x – 3)(x – 5) 2 2
D) y ≥ x – 4 E) y ≥ x – 4x
⇒ (x – 2)(2x – 8) = 0
y ≤ –x + 1 y≤x–1
⇒ x = 2 veya x = 4 d doğrusunun alt bölgesi ve parabolün iç bölgesi taralı bölgeyi gösterir.
Toplamları 2 + 4 = 6 dır. (Cevap A)
(Cevap E)

11. x2 + ax + b = 0 denkleminin bir kökü 4


x2 + cx + d = 0 denkleminin bir kökü –7 dir.

Bu iki denklemin diğer kökleri eşit olduğuna göre,


15. x(4 – x) > 0
a – c farkı kaçtır?
(2x + 5) (x – 2) > 0
A) –11 B) –3 C) 1 D) 3 E) 11
Ortak kök x2 olsun. eşitsizlik sisteminin çözüm kümesi (a, b) açık aralığı
4 + x2 = –a olduğuna göre, a + b toplamı kaçtır?
⇒ –4 – x2 = a
–7 + x2 = –c
–7 + x2 = –c 5
– –
––––––––– –––– –
–11 a–c x 2 0 2 4
x(4 – x) – – 0 + + 0 –
(Cevap A)
(2x + 5)(x – 2) + 0 – – 0 + +
118 (a, b) = (2, 4) ⇒ a + b = 6 bulunur.
12. x2 – (2 – m)x + m – 3 = 0 (Cevap 6)

denkleminin kökleri x1 ve x2 dir.

x1 < 0 < x2

|x1| > x2 16. 4x – x3 < 0


olduğuna göre, m için aşağıdakilerden hangisi doğrudur? eşitsizliğinin çözüm kümesi aşağıdakilerden hangisidir?
A) m < 2 B) 2 < m < 3 C) 3 < m < 4 A) (–∞, –2) B) (–2, 2) C) (0,2)
D) –3 < m < –2 E) m > 3 D) (–2, 0) ∪ (2, ∞) E) (0,∞)
x1 . x 2 < 0 ⇒ m – 3 < 0 ⇒ m < 3 4x(4 – x2) < 0 ⇒ 4x(2 – x)(2 + x) < 0
⇒2<m<3
x1 + x2 < 0 ⇒ 2 – m < 0 ⇒ m > 2 x –2 0 2
(Cevap B) + 0 – 0 + 0 –
Ç = (–2, 0) ∪ (2, ∞)
(Cevap D)

You might also like